◆ わからない問題はここに書いてね 15 ◆

このエントリーをはてなブックマークに追加
1132人目のともよちゃん
   / ̄   ̄ ヽ
  / ,,w━━━.、)   / ̄ ̄ ̄ ̄ ̄ ̄ ̄ ̄ ̄ ̄ ̄ ̄ ̄ ̄ ̄ ̄ ̄
  ! .fw/f_」」_|_|_i_)  | ここは分からない問題について質問するさくらちゃんスレですわ
  ヽ|:::(6||f;j' ,fj'||)  | 関連スレッドや業務連絡・その他、
 ∠|::i:!::|:|、_ワノ:i、 <  数学記号の書き方の例は >>2-9 の中にありますわ
  .|::|< |::|ヽーノ`l:i;ヽ \_________________
  .ノ:ノ' i:::l `只´|:|i)::)
 (::(:i  |:::|ノ ) j:j|:(

    (⌒, -- 、⌒)     / ̄ ̄ ̄ ̄ ̄ ̄ ̄ ̄ ̄ ̄ ̄ ̄ ̄ ̄ ̄ ̄
  _  Y      Y  _ < 『質問です』って名前で質問を始めてくれると
 ミ \| ・  . ・| / 彡 | 見つけやすくて助かるわー
    @ゝ.  ^  ノ@     \________________

【前のスレッド】
◆ わからない問題はここに書いてね 14 ◆
http://cheese.2ch.net/test/read.cgi/math/1002893257/l50
2132人目のともよちゃん:01/10/27 17:26
3132人目のともよちゃん:01/10/27 17:26
【業務連絡】
■900を超えたら新スレに移行準備.
■旧スレ側 → 終了宣言,新スレへの誘導.
■新スレ側 → 開始宣言と目次,旧スレのリンク,掲示板での数学記号の書き方例,
  業務連絡・その他,旧スレ側の残り問題の移動.
■数学板の要望スレで数学板の注意書き(リンク先)の変更依頼.

【その他】
■単独の質問スレは,このスレか「くだらんスレ」に誘導して下さい.
■誤って過去スレに書き込まれた質問は,最新スレに誘導して下さい.

【数学板削除依頼スレ】
http://teri.2ch.net/test/read.cgi/saku/986384122/ (レス削除)
http://teri.2ch.net/test/read.cgi/saku/987829968/ (スレッド削除)
【ローカルルール等リンク先更新総合スレッド】
http://teri.2ch.net/test/read.cgi/accuse/992178408/
4132人目のともよちゃん:01/10/27 17:27
【掲示板での数学記号の書き方例】
■数の表記
 ●スカラー:a,b,c,...,z, A,B,C,...,Z, α,β,γ,...,ω, Α,Β,Γ,...,Ω, ... (← ギリシャ文字はその読み方で変換可.)
 ●ベクトル:V=[V[1],V[2],...], |V>, V↑, vector(V) (← 混同しない場合はスカラーと同じ記号でいい.通常は縦ベクトルとして扱う.)
 ●テンソル(上下付き1成分表示):T^[i,j,k...]_[p,q,r,...], T[i,j,k,...;p,q,r,...]
 ●行列(1成分表示):M[i,j], I[i,j]=δ_[i,j]
 ●行列(全成分表示):M=[[M[1,1],M[2,1],...],[M[1,2],M[2,2],...],...], I=[[1,0,0,...],[0,1,0,...],...] (← 列(または行ごと)に表示する.)

■演算・符号の表記
 ●足し算:a+b
 ●引き算:a-b
 ●掛け算:a*b, ab (← 通常は"*"を使い,"x"は使わない.)
 ●割り算・分数:a/b, a/(b+c), a/(bc) (← 通常は"/"を使い,"÷"は使わない.)
 ●複号:a±b=a士b, a干b (← "±"は「きごう」で変換可.他に漢字の"士""干"なども利用できる.)
 ●内積・外積・3重積:a・b=(a,b), axb=a∧b=[a,b], a・(bxc)=(axb)・c=det([a,b,c]), ax(bxc)

■関数・数列の表記
 ●関数:f(x), f[x]
 ●数列:a(n), a[n], a_n
 ●平方根:√(a+b)=(a+b)^(1/2) (← "√"は「るーと」で変換可.)
 ●指数・指数関数:a^b, x^(n+1), exp(x+y)=e^(x+y) (← "^"を使う."exp"はeの指数.)
 ●対数・対数関数:log_{a}(b), log(x/2)=log_{10}(x/2), ln(x/2)=log_{e}(x/2) (← 底を省略する場合,"log"は常用対数,"ln"は自然対数.)
 ●三角比・三角関数:sin(a), cos(x+y), tan(x/2)
 ●行列式・トレース:|A|=det(A), tr(A)
 ●絶対値:|x|
 ●ガウス記号:[x] (← 関数の変数表示などと混同しないように注意.)
 ●共役複素数:z~
 ●転置行列・随伴行列:M† (← "†"は「きごう」で変換可.)
 ●階乗:n!=n*(n-1)*(n-2)*...*2*1, n!!=n*(n-2)*(n-4)*...
 ●順列・組合せ:P[n,k]=nPk, C[n.k]=nCk, Π[n,k]=nΠk, H[n,k]=nHk (← "Π"は「ぱい」で変換可.)

■微積分・極限の表記
 ●微分・偏微分:dy/dx=y', ∂y/∂x=y,x, D^(n)f(x) (← "∂"は「きごう」で変換可.)
 ●ベクトル微分:∇f=grad(f), ∇・A=div(A),∇xA=rot(A), (∇^2)f=Δf (← "∇"は「きごう」,"Δ"は「でるた」で変換可.)
 ●積分:∫[0,1]f(x)dx=F(x)|_[x=0,1], ∫[y=0,x]f(x,y)dy, ∬_[D]f(x,y)dxdy, 点[C]f(r)dl (← "∫"は「いんてぐらる」,"∬"は「きごう」で変換可.)
 ●数列和・数列積:Σ_[k=1,n]a(k), Π_[k=1,n]a(k) (← "Σ"は「しぐま」,"Π"は「ぱい」で変換可.)
 ●極限:lim_[x→∞]f(x) (← "∞"は「むげんだい」で変換可.)

■その他
 ●図形:"△"は「さんかく」,"∠"は「かく」,"⊥"は「すいちょく」,"≡"は「ごうどう」,"∽"は「きごう」で変換可.
 ●論理・集合:"⇔⇒∀∃∧∨¬∈∋⊆⊇⊂⊃∪∩"は「きごう」で変換可.
 ●等号・不等号:"≠≒≦≧≪≫"は「きごう」で変換可.

※ ここで挙げた表記法は1例であり,標準的な表記法からそうでないものまで含まれているので,後者の場合使う時にあらかじめことわっておいたほうがいい.
※ 関数等の変数表示や式の括弧は,括弧()だけでなく[]{}を適当に組み合わせると見やすい場合がある.
※ 上記のほとんどの数学記号や上記以外の数学記号は大体「きごう」で順次変換できる.
5132人目のともよちゃん:01/10/27 17:27
━━━━━━━━━━━━━━━━━━━━━━━━━━━━━━

          移転完了しましたわ (o^-')b
        ◆ わからない問題はここに書いてね 15 ◆

━━━━━━━━━━━━━━━━━━━━━━━━━━━━━━

★__________________________.
|              │
│ はにゃ〜ん     |
| γ∞γ~  \    |
│人w/ 从从) )   │
│ ヽ | |┬ イ |〃  │
│ `wハ~ . ノ)    │
│  / \`「 .     │
| 数学板さくらスレ  |
|_________________________│


〃二二ヽ
| |77777〉
| | ゚д゚ノ|  サクラチャンノハタケイヨウデスワ
|⊂   つ
6132人目のともよちゃん:01/10/27 17:27
                          _, -/   _,..-―`─'─-..、_    /
/ ̄ ̄ ̄ ̄ ̄ ̄ ̄ ̄ ̄ ̄\          く ヽ; ー_/:::::::::::::::::::::::::::::::::::::\_/\
| そういうことで、15番目の  |         | o'i /:::::::::::::::::::::::::::::::::::::::::::::::::: \、,o`ー、
| さくらちゃんスレがいよいよ |        ー'7::::::::::::::::::::::::::::::::::::::::::::::::::::::::::::::ヽ=~~/
| 始まりますわ〜        >        /,l:::::::::::::,:::::,:/l:|l:: l:::|l:::ト:::::::i::::::::::::,:::| ̄
\__________/         / /ノ:::::::::;/|'|/|:l |' |:|'!::||:ノノl:/'l:ノl`l::|:;:::|、
                             |/:::/::/:::|'!-十‐ `´ ' -十-'、 ノノ/::::::|
                        _/:::/|_:;!::::| ,.==、     ,.==、´ ',n、::::::|
                _,....-─一 '::::::::::::;|::::::/.;n´ ト-':l       ト-':i` /7|.l::::::|
             _,..-::'::::::::::::::::::::::::::::::::::::/.:::/.:l| ト、  ̄    l>    ̄ ノ//, 〉::::::|
            //::::::::_, -'/::::::::::::::::::::::|:/.:::: r、ヽヽ_ ヽ フ   /   /::::::::::|
  __       〈 ヽ::::/ _/::::::::::::::::::::_//.:::::::::::\    `ヽ _ /  _/:l:::::::::::\
/ ̄\::ヽ_     \ \|/:::::::::::::::::::::/.::::/.:::::::::::::::::.`ー,-、  |.:.::_l   r'i:::::::|:::::::::::::::::\
  ,..-─::::::::ー.、_   )/:::::::::::::::::::::〈.:::::/r' ̄二ニ_ ̄`ー_,!-─i:::|:_`ー〈/\:ト'二 ̄`-、:::\
  /'"/::::::::::::::::::::: ̄: ̄:::::::::::::::/'::::::::|.::/.::|  __,-'\_,ト::;o;:=|::|::;.o;::;=|_/|,-──、_,/::::::::\
   /::/ ̄>::::::::::::::::::/'::::://:::::::::::::|.:|.::::\| ヽ._    |::::::::::::|-|:::::::::::::|'::/   /_,〈-、:::::::::::|
  ,// /.-──-─'/:::://::::::::r'二|/.::::::./ー' |  _/|::::::::::::| |:::::::::::::|/ -、_ノ_,イ ノ::::::/|
ー-'   //      /::::::://::::::::::::::ヽ:::|:::::::::::\_, | ̄::::/:::::::::::::| |::::::::::::::l /:::/ |::::://:::/ ノ
      |'   (   /::::::::::| |:::::::::::/二ノ::::::::::::::::::::.<_|::::::/:::::::::::::: |ノ|::::::::::::::::ヽ:::/,─'::::::':/ /
     \  `ー'::_:ノ|:::: | |:::::::::((ゝ:::::::::::::::::::::::::::;::.ヽ:::::::::::::::::::::|,ゝ|::::::::::::::::::::/:::::::::::::::( _/
           ̄ ,ノ::::::| |:::::::::::::::::::::::::::::::::::::::::::|`ー\::::::::::::::ノ-‐ヽ:::::::::::::/-─i:::::::::::ヽ、
          ー-イ:::/ ヽ::::ト、::::::::::::::::::::::::::::::::〉,-―〈ヽ:::::/ ─=\_:/)__ |::/):::|ヽ::|
           _/ノ  |:::::| |:::::::、::::::::::::::::::::/  _/   ̄ /,   \   ヽ\_// |ノ
             ̄  _ノノ|::::|ヽ:::::|ヽ:::::::::_/ /     / /     \   \_\_
                    \ヽノ:ノ/:::::/  /       / /         \     \ `\_
7132人目の素数さん:01/10/27 17:55
さくら!7番ゲットなんかしないもォォォォン!!
 ̄ ̄ ̄ ̄ ̄∨ ̄ ̄ ̄ ̄ ̄ ̄          パンチラゲットですわ〜
                            ̄ ̄ ̄ ̄∨ ̄ ̄ ̄ ̄ ̄ ̄
   ,   _ ノ)                        ---
  γ∞γ~  \    ホエー              γ ==== ヽ   ハニャーソ♪
  |  / 从从) ) ヘノ)                 |_|||_||_||_| |  \ヘ
  ヽ | | l  l |〃  \          (´´     __||ー. ー |) ゞ / \          (´´
  `从ハ~_ーノ)  ヾ /      (´⌒(´      |0.ハ ワ ~ノ| ヽ ___ /      (´⌒(´
   ⊂ >< /⊂ __/つ≡≡≡(´⌒;;;;≡≡≡  └⊂ >< /⊂ _」__/つ≡≡≡(´⌒;;;≡≡≡
              (´⌒(´⌒;;                    (´⌒(´⌒;;
      ズザーーーーーッ                    ズザザーーーーーッ
8132人目の素数さん:01/10/27 18:08
さっそくですが、ちょっと質問です。

α_0,…,α_nを相異なる複素数、a_0,…,a_nを少なくとも一つが
0でない複素数とするとき、
〜=(a_0)e^((α_0)t)+…+(a_n)e^((α_n)t){!≡}0

証明
〜{!≡}0と仮定すると、〜のt=0におけるテイラー展開の係数は
すべて0。特に

a_0+…+a_n=0
α_0(a_0)+…+α_n(a_n)=0

α_0(a_0)^n-1+…+α_n(a_n)^n-1=0

この同時連立一次方程式の係数行列は±П[i<j](α_i-α_j)≠0
矛盾である。

・・・え?どのへんがどうしてなにに矛盾なのよ?う〜・・・。
たぶんつまらないことと思いますが、さらりと教えて頂きたいです。
よろしくお願いします。
9質問です:01/10/27 18:16
微分の問題なんですが教えてください。

正方形の板一枚から、角から一辺xの正方形4枚を切り取って残った十字型の板から蓋のない直方体の箱を作りたい。
側面積と底面積の和を一定にしたままの時、底面の一片と立体の高さとの比を求めよ。

という問題です、、答えは1:2です。
10132人目の素数さん:01/10/27 18:22
>>8
>この同時連立一次方程式の係数行列は±П[i<j](α_i-α_j)≠0
>矛盾である。
係数行列の行列式のまちがい。(Vandermondeの行列式)
もしこれ全部0だと両辺に係数行列の逆行列をかけるとa_i=0になる。
11だから質問ですって:01/10/27 18:31
線形代数の問題です。
”交代行列の階数は常に偶数である。”
の証明方法を教えてください。
偉そうにしてもかまいませんから、
解らないなら判らないって言ってください、さすれば諦めますから、、
問題

答えが1:2となり
微分を使って解ける問題になるように
>>9を修正せよ。(難易度★★★)
13132人目の素数さん:01/10/27 18:44
>>12
これはむずい。
14132人目の素数さん:01/10/27 20:05
記号萌え〜
15質問します:01/10/27 20:11
(36.5)×(96.2)÷(85.2)×(84.3)+SIN41.2÷TAN58.2=?
俺文型だからわからないので教えて下さい
>>15
スタート→プログラム→アクセサリ→電卓→電卓の種類→関数電卓
17132人目の素数さん:01/10/27 21:29
自然数m、nに対して、 1/m + 1/n < 5/4 をみたすとき 1/m + 1/n の
最大値を求めてください。
18e:01/10/27 21:34
e=2.7..... この続きを100桁くらいまで知っていたらおしえてくれ。
19>18:01/10/27 21:58
In[1]:=
N[E, 101]

Out[1]=
2.7182818284590452353602874713526624977572470936999
595749669676277240766303535475945713821785251664274
20132人目の素数さん:01/10/27 22:02
(r,0)と(0,1-r)を結ぶ線分
rを[0,1]の範囲で動かすと
その境界?に双曲線のような曲線ができますが
1.この曲線の式
2.x>=0, y>=0, とこの曲線が囲む面積
って求められます?
21132人目の素数さん:01/10/27 22:03
>>11
交代行列は対角化可能でその固有値は0か純虚数。
純虚数固有値の固有空間の複素共役が
その複素共役固有値の固有空間になるから。
22132人目の素数さん:01/10/27 22:33
>>21
実数体ならそれでいいけど
もっと一般のたいだったらどうする?
23132人目の素数さん:01/10/27 22:51
おまえら、わけわかんねぇこと回転じゃ根絵よ
日本語でかけ アホ
というわけでこのすれは
■■■■■■■■■■■■終了■■■■■■■■■■■■
24132人目の素数さん:01/10/27 23:04
GL_2(Z/nZ)の位数の求め方誰か教えてください。
25ななし:01/10/27 23:10
誰か教えて下さい。5!=120ってなにですか?
26132人目の素数さん:01/10/27 23:11
5がびっくり仰天したら120にドカン!
27>24:01/10/27 23:14
n^4 から ad-bc=0 の解の数を引けばいいんだね。
この解をどうやって列挙するかだな
28132人目の素数さん:01/10/27 23:15
23て何?誤爆?
29868:01/10/27 23:17
訂正です^^;

微分の問題なんですが教えてください。

正方形の板一枚から、角から一辺xの正方形4枚を切り取って残った十字型の板から蓋のない直方体の箱を作りたい。
側面積と底面積の和を一定にしたまま最大の体積の直方体を作りたい、底面の一片と立体の高さとの比を求めよ。
元の正方形の板の面積は変化しません。
という問題です、、答えは1:2です。
30132人目の素数さん:01/10/27 23:17
5!=163(ひろみ)です
31132人目の素数さん:01/10/27 23:19
5!で郷はちと苦しい
32132人目の素数さん:01/10/27 23:20
>>10
嗚呼・・・やっぱりつまらんことだったわぁ・・・。
お手数おかけしました。ありがとうございます。
33132人目の素数さん:01/10/27 23:27
nの素因数pとしてZ/p^eZでやって
(ad-bc,p)=1のときは分かるのですけど
ad-bc=kp のときが分かりません 
>>29
それなら側面積と底面積の和は一定にならない。
3527:01/10/27 23:33
>>27 はまちがいだった。
ad-bc=0 じゃなくて ad-bc がnと
互いに素でない場合の数を引くんだ。
3629:01/10/27 23:44
ならないんですか?
元の正方形の一片をbとして
立体の底面積は
(b-2x)^2
側面積は(b-2x)x としてやってみたんですが、、、

悪問なんでしょうかねぇ・・・。ごめんなさいわかりません。
こうかいてるんです。
37はなう:01/10/27 23:58
「元の正方形の板の面積は変化しません。」

「正方形の板一枚から、角から一辺xの正方形4枚を切り取って残った十字型の板から蓋のない直方体の箱を作りたい。」
において
xが変化するなら
「側面積と底面積の和を一定にしたまま」
はありえないでしょうに。
38質問です:01/10/27 23:58
RnからRmへの線形変換をAとする。
(1)x∈Rnに対して、|Ax|≦||A|||x|
となることを証明して下さい。
但し、||A||=sup{|Ax|:x∈Rn,|x|≦1}と定義してあります。
Rn、Rmはそれぞれn(m)次元ユークリッド空間です。
(2)λ≧0が
|Ax|≦λxを満たすならば、||A||≦λとなることを証明して下さい。
>蓋のない直方体の箱
つーか正しい問題を推測できる猛者きぼん
側面積と底面積の和が一定という条件の下で
直方体の体積が最大になるのは
底面の一辺と高さの比は2:1。
41132人目の素数さん:01/10/28 00:22
誰かZ/nZ係数の一般線形群の位数の求め方
マジで教えて。
42はなう:01/10/28 00:23
>>39>>29等。
わしゃ「側面積と底面積の和を一定にしたまま」がほんとの問題にはない、に一票。

その時は、元の正方形の一辺をk、体積をf(x)とすると、
f(x)=x(k-2x)^2。で、微分=0になるxを求め、極大値を考えるとx=k/6
このとき高さ=k/6、底辺=k/3になるので、1:2。

って感じかな。
>>40
結局切り取る正方形の一辺xを固定して
元の正方形の一辺bを変数とする、が正解かなあ。
4443:01/10/28 00:27
あーーー違った。ちぇっ(w
>>42
底面の一辺は2k/3。
>>43
それならbを大きくすればいくらでも体積を大きくできる。
46はなう:01/10/28 00:35
>>45おおぅぅ!その通りだ、逝ってきます
では>>40に一票・・
47132人目の素数さん:01/10/28 00:52
2次関数でXが負の領域から正の領域に渡る場合
その2点を結ぶ変化の割合は存在しないことの証明をお願い致します

単調減少の領域から単調増加の領域に結ぶことが
おかしいと考えているのですがどうもぱっとしなくって、、
みなさんのお力をかりたいと思います
48132人目の素数さん:01/10/28 01:53
>>41
n=2 のとき 6 だということだけ確認した。
あんたは n=3 はやってみたかね?
>>47
意味不明。
50132人目の素数さん:01/10/28 02:59
マイナスを発見した人は誰ですか?

インド人ですか?
51132人目の素数さん:01/10/28 03:15
>>47
変化の割合=yの増加量/xの増加量
だよね。
任意の関数y=f(x)上の任意の2点(p,f(p)),(q,f(q))(p≠q)で
変化の割合=(f(q)-f(p))/(q-p)
と計算できるんだけど、質問は何?
45に一票
1:4が解
53 ◆pvySbQO2 :01/10/28 06:00
>>11
a(p,q)≠0とする。
r行にp行の−a(r,q)/a(p,q)倍を足す(r≠p,r≠q)。
r列にp列の−a(q,r)/a(q,p)倍を足す(r≠p,r≠q)。
r行にq行の−a(r,p)/a(q,p)倍を足す(r≠p,r≠q)。
r列にq列の−a(p,r)/a(p,q)倍を足す(r≠p,r≠q)。

この操作でp行,q行,p列,q列は
(p,q),(q,p)の位置以外は0になる。
残りの部分に0でないところがあるなら同じことをしていく。
残りに0以外の所がなくなったら
二つずつ組になっているので階数は偶数になる。
54 ◆pvySbQO2 :01/10/28 07:00
>>17
m,nが共に2以上のとき
1/m+1/n≦1/2+1/2=1。
m,nのうち一方が1のときもう一方は5以上なので
1/m+1/n≦1/1+1/5=6/5。
よって1/m+1/nの最大値は6/5。
55 ◆pvySbQO2 :01/10/28 08:00
>>20
0≦x≦1,0≦y≦1
x(1−r)+yr=r(1−r)(0≦r≦1)となるrがある。
r^2−(x−y+1)r+x=0。
(r−(x−y+1)/2)^2=((x−y+1)/2)^2−x。
0≦(x−y+1)/2≦1,0≦xなので
rが存在する条件は
0≦((x−y+1)/2)^2−x。
0≦(x−y+1)^2−4x。
0≦(x−y)^2−2(x+y)+1。
なので曲線の式は(x−y)^2−2(x+y)+1=0で放物線。

(x−y+1)^2−4x=0。
x−y+1=2x^(1/2)。
y=x+1−2x^(1/2)。
∫_[0,1](x+1−2x^(1/2))dx=1/6。
なので(x−y)^2−2(x+y)+1=0,x=0,y=0で
囲まれた部分の面積は1/6。
56132人目の素数さん:01/10/28 13:35
∫[0,∞]e^{-(x^2)} dx = (√π)/2
この等式を2通りの方法で証明せよ、という問題なのですが
1通りも出てきません・・・。よろしくお願いします。
57132人目の素数さん:01/10/28 15:10
2χ+5=456
5829:01/10/28 15:16
僕の解答は1:4になったんです。

2/3:1/6 になって、底面積と側面積に関する条件がなければ簡単なんですが、、
本当に書いてるんですよ。。。問題間違ってるんですかねぇ・・・。
59132人目の素数さん:01/10/28 15:33
>>53
その証明[[a(p,p),a(p,q)],[a(q,p),a(q,q)]]が正則になること
利用してないか?そもそも標数2なら交代行列=対称行列でランク奇数
なんて簡単につくれるからすくなくともどっかでそのことを証明中
利用するはずだとおもうんだが。
60132人目の素数さん:01/10/28 15:38
すべての自然数は絶対値の異なるいくつかの整数の2乗の和もしくは差
で表されることを示せ。

例)
29=2×2+5×5
56=1×1‐3×3+8×8
61132人目の素数さん:01/10/28 15:44
>>58
>>40さんが正解だろ?

 問題 底面積と側面積の和を一定にたもったまま底面が正方形の
    直方体を変化させるとき体積が最大となる直方体の
    底辺と高さの比を求めよ。

底面の長さx高さyとする。x^2+4xy=k(一定値)とおく。y=(k-x^2)/4x
体積VはV=x^y=-4x^3+4kx。V'の増減表を書いてx=√(k/3)のとき最小。
このときy=2√(k/3)。よってもとめる比は1:2。
>>61
問題はその通りだと思うが、答がめちゃくちゃ。
>>60

任意の自然数nに対して、
 n^2 − (n+1)^2 − (n+2)^2 + (n+3)^2 = 4
だから、
 ある自然数Nが題意のように表せるなら
 N+4も題意のように表せる
ことがわかる。
一方、
 1=1^2
2=4^2−3^2−2^2−1^2
 3=4^2−3^2−2^2
 4=2^2
なので、すべての自然数が題意のように表せることが
帰納的に示される。
64132人目の素数さん:01/10/28 16:39
>>38
|x|=0の時は明らかだから、0≦|x|≦1で考えてみると・・・
上手くいかないなぁ
65132人目の素数さん:01/10/28 16:42
>>62
ほんとだ。あちこちまちがってるよ。逝ってきます。
(n+1)^2−n^2=2n+1。
(n+1)^2−n^2−1^2=2n。
67132人目の素数さん:01/10/28 17:16
>>60
X− n^2 + (n+1)^2
−) X− (n+1)^2 + (n+2)^2

となることと、
 1=1^2
2=4^2−3^2−2^2−1^2
 3=4^2−3^2−2^2
 4=5^2−4^2−2^2−1^2
 5=5^2−4^2−2^2
より、5以降奇数は3を変形、4以降偶数は2を変形することにより順次
生成できることから、数学的帰納法によりすべての自然数は表せる。
68ちむ教の信者:01/10/28 18:18
もれもこっちにきたYO!!
二次関数について、質問があります。
二次関数y=ax^2+bx+cの
頂点が(正の数、負の数)且つ下に凸凹の
時、a、b、cの正負を判断せよ。
これは、a,bは判断できるのですが、どうしてもCが
判断出来ません。なぜならば、aとbどちらが、
大きいかわからないからです。
お願いします。解説を。
>下に凸凹
どおいう意味だ?
70ちむ教の信者の母:01/10/28 18:46
>>68
問題はきちんと正確に書きなさい!
>>60,>>67
すべての奇数2a-1はa^2-(a-1)^2とあらわされ
4以外の偶数2aはa^2-(a-1)^2+1^2とあらわされ
4は2^2とあらわされる。
でよくないか?まちがってる?
72難問:01/10/28 19:26
以下の問題が解けません、分かる方お願いします

食糧供給をめぐって競い合う二つの種のためのモデルは
dx/dt=ax-by
dy/dt=cy-dx
で与えられる。ここでx,yは2種の個体群であり、a,b,c,dは正の数である。
xは
d^2x/dt^2-(a+c)dx/dt+(ac-bd)x=0
をみたすことを示せ。またxは

x=Ae^α1t+B^α2t
の形の解を持つことを導け。ここでαiのうち少なくとも一つは正である。
yに対する解の形も求めよ。
パラメターの値
a=c=2,b=d=1
をもちいて、時刻t=0においてはx=100,y=200であるとし、一方の
種が排除される時間を決定せよ。
>>70
>a^2-(a-1)^2+1^2

a=1のときの2も除外。
2=4^2-3^2-2^2-1^2を補足して解決。
>>71だった
75132人目の素数さん:01/10/28 20:18
>>72
dx/dt=ax-by・・・@
dy/dt=cy-dx・・・A
とすると
@の両辺をtで微分すると
d^2x/dt^2=adx/dt-bdy/dt
Aを@に代入すると
d^2x/dt^2=adx/dt-b(cy-dx)
∴d^2x/dt^2-(a+c)dx/dt+(ac-bd)x=0
また、特性方程式
X^2-(a+c)X+(ac-bd)=0
の判別式
D=(a+c)^2-4(ac-bd)=(a-c)^2+4bc>0
(∵b,c>0)
よってこの特性方程式は2つの実数解を
もつのでその解をα1,α2とすると
x(t)=Ae^α1t+Be^α2t

またa=c=2,b=d=1より
d^2x/dt^2-4dx/dt+3x=0
この特性方程式の解はX=1,3より
x(t)=Ae^t+Be^3t
これを@に代入すると
d(Ae^t+Bet^3)/dt=2(Ae^t+Bet^3)-y
∴y(t)=Ae^t-Be^3t
またx(0)=100,y(0)=200より
A+B=100
A-B=200
これを解くと
A=150,B=-50

x(t)=150e^t-50e^3t
y(t)=150e^t+50e^3t
この式より排除される種はxであり
x(t)=0になるtは
150e^t-50e^3t=0
3e^t=e^3t
log3+t=3t
∴t=(log3)/2

あってる?
76高校生:01/10/28 20:51
教えてください。

「有理数+無理数」=「無理数」を証明せよ。

背理法らしいのですが…。よろしくお願いします。
有理数(p/q)+無理数(x)=有理数(m/n)と仮定
x=(m/n)−(p/q)=略=有理数となって矛盾
78132人目の素数さん:01/10/28 21:00
>>72

dx/dt=ax-by・・・@
dy/dt=cy-dx・・・A
とすると
@の両辺をtで微分すると
d^2x/dt^2=adx/dt-bdy/dt
Aを@に代入すると
d^2x/dt^2=adx/dt-b(cy-dx)
∴d^2x/dt^2-(a+c)dx/dt+(ac-bd)x=0 ―D
ここでDにx=e^mtを代入
代入してからe^mtをtで微分
m^2-m(a+c)+(ac-bd)=0
mを求める
m=(a+c)±√(a+c)^2-4(ac-bd)/2
2つの解を
α1=m=(a+c)+√(a+c)^2-4(ac-bd)/2
α2=m=(a+c)-√(a+c)^2-4(ac-bd)/2
とし
x=Ae^α1t+B^α2t ―E

yに対する解の形

Eを@のxに代入
y=a(Ae^α1t+B^α2t)/b-d(Ae^α1t+B^α2t)/bdt
Ae^α1t+B^α2tをtで微分
y=1/b{e^α1t(Aa-α1A)+e^α2t(Ba-α2B)}  ―F

パラメターの値
a=c=2,b=d=1
をもちいて、時刻t=0においてはx=100,y=200 から
Eにt=0を代入
A+B=100

Fにt=0を代入
A(2-α1)+B(2-α2)=200

ここからどうすればいいかわかりません
あとEはどうしてこうなるのかもよくわかりません
だれか助言を
79はなう:01/10/28 21:00
>>68
「二次関数y=ax^2+bx+cの
頂点の座標を(p,q)とすると、
p>0,q<0で、且つこの関数グラフが下に凸の時、
a、b、cの正負を判断せよ。」
っていう問題でしょう?

問題の式を変形すると、
y=a(x-b/2a)+c-b^2/4a
下に凸なのでa<0、また、
中心の座標は(b/2a,c-b^2/4a)
a<0なのでb/2a>0であるためにはb<0
a<0,b<0のときc-b^2/4a<0であるためにはc<0。ってこと。
80132人目の素数さん:01/10/28 21:05
>>68
cの判定はx=0を代入してグラフを見れば、一発。
y切片でしょ。cって
81132人目の素数さん:01/10/28 21:05
x(1)=1
x(n+1)=√{x(n)+6} (n∈Ν:Nは自然数の集合)

で定められる数列がある。
このときlim(n→∞)x(n)の値を求めよ

この問題でx(n)が有界であるのは、

0<x(n)<3 の時、 0<x(n+1)<3 また、0<x(1)<3 であるから、x(n)は有界。
とやるんでしょうか。それとも、この問題の式を見ただけで一目でわかるんでしょうか

変な質問でもうしわけないです
82高校生:01/10/28 21:18
>>77
=(mq-pn)/nq=有理数ですね。

ありがとうございます。
83小出監督:01/10/28 21:34
質問します。k、μ、M、G、などの単位記号があるけど、他にこう言う単位記号は何が
あるんですか?それは、10の何乗ですか?
例:kは10の3乗

よろしくお願いします。
84132人目の素数さん:01/10/28 21:37
>>38です。
スイマセンが早急に解いていただきたいです。
明日提出なもので・・・
85132人目の素数さん:01/10/28 21:42
∫sin(t)*exp(t)dtみたいな積分ってどうやるのでしょうか?
86132人目の素数さん:01/10/28 21:46
>>85
部分積分を2回すればいいと思う。
同じ項が出てくるはず
87132人目の素数さん:01/10/28 21:50
>>81
>0<x(n)<3 の時、 0<x(n+1)<3 また、0<x(1)<3 であるから、x(n)は有界。
それで完璧。一目では・・・わかる人もいる(w

でも、問題文で「lim(n→∞)x(n)の値を求めよ」とあるということは、
lim(n→∞)x(n)の存在は既知ということなんだろうか?
それならば{x(n)}が有界なのは当然だけど。

ところで、この問題って高校の範囲ではどう解くの?
88132人目の素数さん:01/10/28 21:51
>>38
(1)
x≠0なる任意のxに対して
 |Ax|/|x| = |A(x/|x|)| ≦ ||A||  (∵x/|x|≦1)
だから、x≠0のとき
 |Ax| ≦ ||A|||x| .
x=0のときは自明.
(2)
任意のx∈Rn,|x|≦1に対して、
 |Ax|≦λ|x| ≦λ
だから、supをとって
 ||A||≦λ.
89132人目の素数さん:01/10/28 21:51
>>81
コーシーの判定条件使ったら?
90132人目の素数さん:01/10/28 21:51
>>86
デキターヨ。
ありがとうございました。
91家庭用ゲーム板より質問です。:01/10/28 22:02
【数学的な問題です】

サイコロの最大ダイス目を2とします(つまり1か2しか出ない)。
このとき、マップ上のある場所を通過する際に、ちょうどその場所に
止まる可能性はいくらでしょうか?
ただし、マップは十分長い一本道のループで、城(スタート地点)は
十分離れているものとします。
92132人目の素数さん:01/10/28 22:04
>>81
ありがとうございます。
これから、よく考えてみます
>>91
日本語で書いてください
94132人目の素数さん:01/10/28 22:14
>>91
nの時を考えてみよう
n=1のとき 1/2
n=2のとき 1/2+1/2*1/2=3/4
n=3のとき 1/2*1/2+1/2*1/2+1/2*1/2*1/2=5/8
n=4のとき 1/2*1/2+1/2*1/2*1/2*3+1/2^4=11/16
n=5のとき 1/2*1/2*1/2*3+1/2^4*4+1/2^5=21/32

よってn=nのとき


わからん
95ちむ教の信者:01/10/28 22:20
68です。
凸凹というのは、凸ということです。すいません。わかりにくくて。つまり、
a>0と、いうことかな。
お願いします。

>>70
だから、あんただれだって?
96132人目の素数さん:01/10/28 22:20
>>93

1の目が出る確率が1/2、2の目が出る確率が1/2のサイコロがある。
まず、数直線上の原点に駒を置く。
ここから、
サイコロを振り出た目の数だけ正の方向に駒を動かす
という操作を無限回繰り返す。このとき、座標nに駒が止まる
確率をP(n)とする。
lim_[n→∞] P(n) を求めよ。

>>91 >>94

P(n)= (2/3) - (-1/3) (1/2)^n なので、
lim_[n→∞] P(n) = 2/3
97132人目の素数さん:01/10/28 22:29
十分なんたらっていうのの意図がよく分かりません。

kマス目(k>=1)を通過する際に止まらないのは
k-1マス目に止まっており、2の目がでた場合。

kマス目を通過する際に止まらない確率をa_kとすると
a_k=1/2(1-a_(k-1))
a_k-1/3=-1/2(a_(k-1)-1/3)
a_0=0なので a_k=(-1/3)(-1/2)^k+1/3

求める確率は 2/3+1/3(-1/2)^k
98132人目の素数さん:01/10/28 22:30
しかもこれ書いてるうちに先を越されたし。
99132人目の素数さん:01/10/28 22:33
ループってのが気になるな。
無限に続けるならどのマスに止まる確率も1に近づくことを証明せよってことじゃないの?
10096:01/10/28 22:37
P(n)の計算間違ってた。
>97 の計算が正しくって、
P(n)= (2/3) + (1/3) (-1/2)^n
だ。逝ってきます。
101132人目の素数さん :01/10/28 22:53
>>96
同じ条件でダイス目が3のサイコロの場合、
lim_[n→∞] P(n) はいくつになるのでしょうか・・?
102132人目の素数さん:01/10/28 22:57
>>96
この問題面白いね
今度テストに出そうかな
103132人目の素数さん:01/10/28 23:00
81の問題だけど
x(1)=1
x(n+1)=√{x(n)+6} (n∈Ν:Nは自然数の集合)

で定められる数列がある。
このときlim(n→∞)x(n)の値を求めよ

x(n+1)-3=√{x(n)+6}-3
⇔|x(n+1)-3|≦(1/3)・ |x(n)-3|
∀nに対して成り立つので
|x(n)-3|≦{(1/3)^n-1}・|x(1)-3|(∵0<x_n<3)
ここでn→∞とすると右辺→0ではさみうちよりlim(n→∞)x(n)=3

この解答ってlim(n→∞)x(n)が収束する事は既知としてるの?
よくみかける解答だけど
トポロジー関連の質問です。
トーラスに穴をあけてメビウスの帯を張りつけた曲面のオイラー標数の求め方を教えてください。
あと、射影平面のオイラー標数も教えていただけませんでしょうか?
初歩的な質問で申し訳ないです…
できれば、これ関連の良い参考書か何かあれば教えてください、お願いします。
>>96の便乗です
4〜6の目がそれぞれ1/3の確率で出るサイコロがある。
まず、数直線上の原点に駒を置く。
ここから、
サイコロを振り出た目の数だけ正の方向に駒を動かす
という操作を繰り返す。このとき、座標nに駒が止まる
確率P(n)及びlim_[n→∞] P(n) を求めよ。

楽な求め方ありますかね・・・(;´Д`)
>>104
一般に
χ(A∪B)=χ(A)+χ(B)-χ(A∩B)
以下Mをメビウスの帯、Tをトーラス、SをS^1、DをD^2(に同相な空間)
としT=D∪X,D,S=D∩X≡Sと分解すると
χ(X)=χ(T)-χ(D)+χ(S)=0-1+0=-1
∴χ(X∪M)=χ(X)+χ(M)-χ(X∩M)=-1+0-0=-1
射影平面PをP=M∪D,S=M∩Dと分解して
χ(P)=χ(M)+χ(D)-χ(S)=0+1-0=1
107素数:01/10/29 00:18
何で5次方程式以上は解の公式ないの??
>>33
なぜSL_2(Z/p^eZ)ができてGL_2(Z/p^eZ)ができん?kをpと互いに素な数として
ad-bc≡1 (mod p^e) と ad-bc≡1 (mod p^e) の0≦a,b,c,d≦p^e-1における
解の個数は等しいでしょ?だからSL_2(Z/p^eZ)の位数にZ/p^eZの可逆元の個数
p^e-p^(e-1)をかければおわり。
>>106
どうもありがとうございます。
でも、僕の現在の知識ではちょっと良く分かりません、本当にすみません。
出来れば、球面を三角分割して、面+頂点-辺の方法でご教授願えませんでしょうか?
>>101,>>105
中学生でもわかる大学生にも解けない数学問題集、ピーター、日本評論社
にのってた。一般に1〜kの目のあるさいころでそれぞれのでる確率が
p_iのときf=琶p_i、nにとまる確率をx_nとするとき
limx_n=1/f。
>>109
>>106ででてきた空間(X∪Mなど)の三角分割を具体的にもとめるのはめんどい。
それより>>106にでてきた公式χ(A∪B)=χ(A)+χ(B)-χ(A∩B)を証明するほうが楽。
実際A,B,U=A∪B,I=A∩Bの点、線、面の数をa_0,b_0,u_0,i_0,
a_1,b_1,u_1,i_1,a_2,b_2,u_2,i_2とすると
u_k=a_k+b_k-i_k(k=0,1,2)が成り立つので辺々+−+とたせばいい。
というか君の学校ではχ=点−線+面でならってるの?
それともまだならってないけど予習したいの?
>というか君の学校ではχ=点−線+面でならってるの?
そのようです…

あと、106で出てきたXっていうのはなんですか?
χ(M)=0 χ(X∩M)=0はどうして出すのですか?
あと、D^2っていうのは、球を指しているのですか?
質問ばッかですいません、出来れば貴方が学ばれた参考書を教えていただけませんか?
113132人目の素数さん:01/10/29 01:49
この問題を解いてください。

300Kでのシリコンの伝導体の有効状態密度をNc=2.8×10の19乗/
p3、Nv=1.0×10の19乗/cm3としたとき、真性状態のシリコンのフェルミ
準位の位置、電子と正孔の密度を求めよ。シリコンの禁止帯幅(バンドキャッ
プ)を1.12eVとする。

お願いします。解けません。
114132人目の素数さん:01/10/29 02:16
>>113
こんな紙の上で計算するんじゃなくて
ホール効果でも測定しよう!
115132人目の素数さん:01/10/29 04:47
詳しく教えてください。

正級数蚤_n = ∞ であるとき、S_n=a_1+a_2+・・・+a_n とおいて
(1)蚤_n/S_n (2)蚤_n/(S_n)^2  は収束するか、発散するか?
116名無し:01/10/29 05:16
>>115
(1) は,Σ{a_n/S_n} ですか?それとも (Σa_n)/S_n ですか?
まあ,普通に考えて後者なわけはないと思うんだけど。
以下,前者だと思って解答しますね。
∀k∈Nで,a_k=1とすると,S_n=n
よって, Σ{a_n/S_n}=1/1+1/2+・・・+1/n →∞ (n→∞)
117132人目の素数さん:01/10/29 05:17
フェルマーの定理を証明してっちょ
118132人目の素数さん:01/10/29 06:05
>>116
勿論前者ですが、それは発散する例の一つです。
収束、あるいは発散するならそれを示し、
どちらとも限らないなら例示せよということです
119名無し:01/10/29 06:27
>>118
ああ,そういうこと。
(1)はa_n=1/nとおくと収束するから,『どちらとも言えない』だと思う。
(2)はa_n=1とすると収束するし,a_n=1/(n^2)とおくと発散する気がする。
眠くてよくわかんなくなってきた。ごめん。
120132人目の素数さん:01/10/29 06:38
a_n=1/nだとS_n<1+lognとなって
与式>1/n(1+logn)>(1/2)・1/nlogn
1/nlogn は ∫dx/xlogx=log(logx)→∞(x→∞)
だから、発散しますよ?
>>119
煤i1/n^2)<∞。
122名無し:01/10/29 06:55
>>119
お前の頭,大丈夫か?
>>115
煤ia(n)/S(n)^2)<2/a(1)。
124132人目の素数さん:01/10/29 08:10
(1)はどうも発散しそうだね。
125132人目の素数さん:01/10/29 08:49
(1)はa_k = 1で発散、a_k = 1 / 2^k で収束、よってどちらとも限らない。
(2)は常に収束。なぜならば
(a + b) / { S + (a + b) } <= a / { S + a } + b / { S + b }
より、正の数からなる数列a_kが与えられたとき一つNを取ると、
a_{ N - 1 }, a, b, a_{ N + 1 },... a + b = a_{ N }
となるような別の数列を取ると、上の考察から蚤_n/(S_n)^2は大きくなる。
こうやって適当に区間を分割してゆくと、上の数列の総和はある正の数cを取って
\integral_{c, infty} t / { \integral_{0, t} x dx}^2 dt
= \integral_{c, infty} 4 / t^3 dt
で押さえられる。この時、最後の積分は明らかに収束。(c > 0を使った。)
126125:01/10/29 08:50
間違えた。
(a + b) / { S + (a + b) } <= a / { S + a } + b / { S + b }

(a + b) / { S + (a + b) } <= a / { S + a } + b / { S + a + b }
127132人目の素数さん:01/10/29 09:21
>>125
おいおい、a_n=1/2^n だとS_nが収束して条件に反するぞ?
128125:01/10/29 10:08
スマソ。どっちも大嘘。修行しなおしてきます。
129132人目の素数さん:01/10/29 10:28
この問題を解いてください。

300Kでのシリコンの伝導体の有効状態密度をNc=2.8×10の19乗/
p3、Nv=1.0×10の19乗/cm3としたとき、真性状態のシリコンのフェルミ
準位の位置、電子と正孔の密度を求めよ。シリコンの禁止帯幅(バンドキャッ
プ)を1.12eVとする。

お願いします。解けません。
130132人目の素数さん:01/10/29 11:49
>>115
(1)発散する
n<mとして
a(n)/S(n)+・・・+a(m)/S(m)
<{a(n)+・・・+a(m)}/S(m)→1(n→∞)
(∵lim[m→∞]S(m) = ∞)
コーシーの判定条件と、与えられた級数が正級数であることより
この級数は発散。
131132人目の素数さん:01/10/29 11:53
スマソ、訂正:4行目
誤:(n→∞)
正:(m→∞)
132中学生の問題:01/10/29 11:58
解けない問題があるので誰か教えてください。

AB=ACとなる二等辺三角形△ABCがあり、∠Aは20度です
B、CそれぞれからAC、ABに向かって底辺BCと60度、50度
になるように直線をひいて交わった点をそれぞれD、Eとします

そのときの∠BDEは何度?って言う問題です。

分かるかたいましたら、答えと証明をおしえてください。
お願いします
133数学?:01/10/29 12:15
下のURLの問題がわかりません。
誰か教えて下さい。
ttp://dare.mine.nu/ai/
135中学生の問題:01/10/29 13:13
ありがとうございました
136132人目の素数さん:01/10/29 16:16
円Oの円周上に3点A,B,Pをとり、弦ABに対する円周角∠APBをとる。
また円周の内側に任意の点Qをとり、円周の外側に任意の点Rをとる。
このとき∠AQB>∠APB>∠ARBとなることを証明せよ
137名無し:01/10/29 16:21
直線ABに関してP,Q,Rが同じ側にないと成り立たないので, 以下,そのように仮定します。
位置の一般性とかがちょっと自信ないけど。
BQの延長と円との交点をSとすると,∠AQB>∠ASB
AR,BRが円と交わらないときは,僊BRと僊BPの内角を比較。
少なくとも一方が交わるときは,その交点をTとすれば,∠ATB>∠APB
138ちむ教の信者:01/10/29 18:59
ありがとうございます。
数列で質問があるのですが、
p、qが定数のとき、一般項anがan=pn+qの
形で表される数列であることを示せ。

で、an+1とすつとき、pnの項が、pn+1
していい理由がわかりません。
答えが変わるのでは?
わかりやすく教えてください。
>>138
本人以外で修正できる猛者きぼん
問題の全文載せてないだけだと思うが・・・
140ちむ教の信者の母:01/10/29 19:29
an+1とすつとき以外についても教えてやってください。
もうちむ信は放置で行こうよ・・・
お化けに押しつけよう
143ちむ教の信者:01/10/29 20:22
95ですけど、お願いします。
144ちむ教の信者の母:01/10/29 20:27
私からも、お願いします。
145ちむ教の信者:01/10/29 20:49
ごめんなさい、ありました。
中心の座標は(b/2a,c-b^2/4a)<中心ってなんのこと?
頂点?
a<0,b<0のときc-b^2/4a<0であるためにはc<0。ってこと。<CがB^2より、
おおきいかわからないのでは?
cの判定はx=0を代入してグラフを見れば、一発。
y切片でしょ。cって<では、最初にCを求めろということですね。
ありがとうございました。

ありがとうございます。
数列で質問があるのですが、
p、qが定数のとき、一般項anがan=pn+qの
形で表される数列であることを示せ。

で、an+1とするとき、pnの項が、pn+1
していい理由がわかりません。
nはn項めをということ。
答えが変わるのでは?
わかりやすく教えてください。
>>145
ごめん。マジで聞きたいんだけど、チミは中学生?それとも高校生?
147難問:01/10/29 21:14
>>75 >>78
ありがとうございます助かりました

この問題の数値を変え虚数にした場合はどうなるのか教えてください。
お願いします。

食糧供給をめぐって競い合う二つの種のためのモデルは
dx/dt=ax-by
dy/dt=cy-dx
で与えられる。ここでx,yは2種の個体群であり、a,b,c,dは正の数である。
xは
d^2x/dt^2-(a+c)dx/dt+(ac-bd)x=0
をみたすことを示せ。またxは

x=Ae^α1t+B^α2t
の形の解を持つことを導け。ここでαiのうち少なくとも一つは正である。
yに対する解の形も求めよ。
パラメターの値
a=c=2,b=d=1/2
をもちいて、時刻t=0においてはx=100,y=200であるとし、一方の
種が排除される時間を決定せよ。
148ちむ教の信者:01/10/29 22:22
>>146
問題のレヴェルを。みておくり。
149ベクトル:01/10/29 22:23
ベクトルの問題がわからないので教えてください

空間において一辺の長さが2の正四面体OABCがある。
OAベクトルをa、OBベクトルをb、OCベクトルをcとおく。
また、△ABCの重心をGとおく。

1.△OBCの重心をgとおく。線分OGとAgが交わることを示せ。

2.また、その交点をFとするとき、OFベクトルをa,b,cで表せ。

3.また、比OF:FGを求めよ。
>>148
小学生だネ!(・∀・)
>>148
問題の内容は高校の範囲だが、
君の思考力は中学以下のように見えるので・・・
152132人目の素数さん:01/10/29 22:32
すみません、わからない証明問題があるのですが・・・。

非負の無限数列{a(n)},a(n)≧0,n=0,1,2,・・・がある。
そのとき関数f(z)=Σ_[n=0,∞]a(n)Z^nが、|z|<1で収束すると仮定する。
このとき
lim_[z→1-0(左側極限)](1/n)Σ_[n=0,n-1]a(n)=A<∞
となるための必要十分条件は、
lim_[z→1-0](1-z)f(z)=A
が成り立つことであることを証明せよ。

これがわかりません・・・。Abelの定理でもないし、どうするのでしょう?
153ちむ教の信者の母:01/10/29 22:35
ウチの子は学校の枠に囚われず、自由にのびのび育てています
154132人目の素数さん:01/10/29 22:58
pとqを2つの異なる奇素数とする。(a,pq)=1のとき、

    a^[{φ(pq)}/2]≡1 (mod pq)

であることを証明してください。誰か天才の方。解答求む。
>>152
Abelの定理だYO!
g(z)=(1-z)f(z)とおく。a(-1)=0とおいてg(z)=納k=0,∞](a(k)-a(k-1))z^k。
lim[z→1-0]g(z)=A⇔lim[n→∞]納k=0,n](a(k)-a(k-1))=A(Abelの定理)
よって
lim[n→∞]納k=0,n](a(k)-a(k-1))⇔lim[n→∞](1/n)納k=0,n-1]a(k)
をしめせばいい。これはさほどむずかしゅうない。
156132人目の素数さん:01/10/29 23:41
>>108
ad-bc≡kp (mod p^e)
の場合は数えなくていいのですか?
157132人目の素数さん:01/10/29 23:45
>>156
GL_2(Z/nZ)の元[[a,b],[c,d]]はad-bcがZ/p^eZで可逆になる元でしょ?
ad-bc≡kp (mod p^e)ならこれは可逆元でないっしょ。
xの類がZ/p^eZで可逆⇔xとpが互いに素。
158文系:01/10/29 23:46
質問なんですけど…
文系なのにレポート課題出されてこまってます。
助けてぇ〜

☆対数の法則
Pr{lim_[n→∞] X1+X2+……+Xn/n =μ}=1
を確かめるために、X1,X2……をサイコロ投げとしたときの
Pr{k≦X1+X2/2<k+1}、k=1,2,3,4,5

Pr{k≦X1+X2+X3+X4/4<k+1}、k=1,2,3,4,5
の確率から求めたヒストグラムを比較せよ。

という課題なんですけど。
いったいどういう計算して、どういうヒストグラムが書ければいいんでしょうか??
159文系:01/10/29 23:48

すいません。
「対数」じゃなくて「大数の法則」デシタ(・∀・)
>>158そのHNはやめろ
161132人目の素数さん:01/10/29 23:49
>>157
あ、そうだったのか〜〜
確かにそうでないと逆行列が存在しないな〜
クソーこんなことに時間結構無駄にしちゃった。
逝ってきます
162152:01/10/30 00:07
>>155さん
えっ!?n=0,1,2,・・・・なのにa(-1)っておけるんですか?
163132人目の素数さん:01/10/30 00:13
>>162
正確にやると
(1-z)f(z)
=(1-z)納n≧0]a(n)z^n
=納n≧0](a(n)z^n-a(n)z^(n+1))
=納n≧0]a(n)z^n-納n≧1]a(n-1)z^n
=a(0)+納n≧1](a(n)-a(n-1))z^n
となるけどa(-1)はとくに指定されてないからかってにa(-1)=0と
することにするとさらに
=納n≧0](a(n)-a(n-1))z^n
となってカコイイからそうしただけ。きにくわんかったらその前の行のまま
計算すべし。
164132人目の素数さん:01/10/30 00:20
質問です。
ωが微分形式である、ということを
証明するにはどうしたらいいんですか?
165152:01/10/30 00:31
>>163
ありがとうございました。わかりました。
166132人目の素数さん:01/10/30 00:35
>>165
あ〜うそうそ。>>155はうそ。いまみなおしてみたら完全うそ。
もっかいやてみる。
167152:01/10/30 00:42
>>166
うっ!そうなんですか・・・。
誤りに気づかなかった自分に鬱・・・
すみません、またお願いします。
168質問です:01/10/30 01:08
∬_[D]cosxy dxdy D:0<= x <=1 , 0<= y <=π/2
という二重積分を解く問題なのですが
自分で計算したところsiny/yが出てきて積分が分からなくなってしまいました。
解法を教えてください。
169名無し:01/10/30 01:17
>>168
それは,yからさきに積分すればできるはず。
170168:01/10/30 01:33
>>169
レスありがとうございます。
しかし、yから先に積分してもsin(xπ/2)/xになってしまって
xから先に積分した時と同じような形になると思うのですが?
171名無し:01/10/30 01:38
>>170
ごめん,間違えた。
172168:01/10/30 01:56
>>171
いえいえ、僕も最初そういう風に考えました。
自分で もう少し考えてみます。
173132人目の素数さん:01/10/30 03:32
kを3以上の定数とする。
実数a,b,cが
 a+b+c=k , a≧1,b≧1,c≧1
を満たすとき、
(a/b) + (b/c) + (c/a) のとり得る値の範囲を求めよ。
174有効数字わからんさん:01/10/30 03:51

誰か有効数字についてご教授お願いします。
0.75は有効数字2桁だけど、1.75とか12.02とかは?
175132人目の素数さん:01/10/30 03:54
1.8と12
176132人目の素数さん:01/10/30 03:55
>>174
1.75は3ケタ、12.02は4ケタ
177有効数字わからんさん:01/10/30 04:07

サンクス!
1.76を有効数字2桁で表すには?
178132人目の素数さん:01/10/30 04:07
1.8
179132人目の素数さん:01/10/30 05:03
正級数蚤_n = ∞ であるとき、S_n=a_1+a_2+・・・+a_n とおいて
蚤_n/(S_n)^2  は収束するか、発散するか?

↑誰かやりかた教えてください
180132人目の素数さん:01/10/30 11:44
>>174
0.75の0は意味を持たないので2桁
1.75の1は桁として意味を持つので3桁

但し、0.750だとしたら、最後の0は意味を持つので3桁。
181132人目の素数さん:01/10/30 15:00
.
182132人目の素数さん:01/10/30 15:12
質問です。

(1)Rでもって実数対、すなわちすべての実数の集まりを表す。
Rで微分可能な実数値関数の全体はR上のベクトル空間
を成すことを証明せよ。

(2)f_1(x)=x 、f_2(x)=x^2 、f_3(x)=x^3 という三つの関数
はVの元であるが、これらはR上一次独立であることを証明せよ。

(3)ベクトル空間VはR上有限次元であるか、無限次元であるか。
理由を付して答えよ。

どなたか教えてください。よろしくお願いします。
>>182
ベクトル空間の定義は知ってる?
一次独立の定義は?
次元の定義は?

これらがわからなければ話にならないし、
逆にこれらがわかっていれば(1)(2)(3)は
簡単なはず。
まず、上に述べたtermの定義を教科書で確認せよ。
184132人目の素数さん:01/10/30 15:30
ベクトル空間の定義のところをもう一度読んで、
関数の和とスカラ倍をどう決めればいいか
考えてみる(そのまんまだけど)。

(2)は一次独立の定義に実際に三つを当てはめて考える。
これがわかれば(3)もわかるね。

教科書に他の例が示してあったら、その証明を参考にすれば
すぐに分かるんじゃないかな。
185184:01/10/30 15:31
えーと、えーと、かぶすま!
186 ◆pvySbQO2 :01/10/30 16:00
>>179
a(n)/S(n)^2≦∫_[S(n−1),S(n)](dx/x^2)
を使う。
>>186
ありがとうございます
1881:01/10/30 16:19
1わる3の答おしえてくれ 1なの 0・999999・・・・・なの
189132人目の素数さん:01/10/30 16:20
1/3です。
1901:01/10/30 16:20
↑1わる3かける3だった・・・ スマソ
191132人目の素数さん:01/10/30 16:22
ε-δで1に押さえます。以上。
192190:01/10/30 16:24
(゚Д゚)ハァ? スマソ・・・190なんだけど意味がわからん↑
193132人目の素数さん:01/10/30 17:58
ある自然数を2乗するとその答えの100の位と10の位と1の位が
全て同じ数になる。ある自然数の最小のものは何?
>>193
38^2=1444。
>>194
39です
196132人目の素数さん:01/10/30 19:31
152が未解決ですぞ。
197132人目の素数さん:01/10/30 19:38
>>196
おお、おれ>>152じゃないんだけどおれのほかにも>>152がきに
なってる人がいたか。これ昨日の晩から気になってしょうがない。
これどうやんの?
198132人目の素数さん:01/10/30 20:10
つーか極限操作の意味とシグマの前にnが出てる理由が分からん
199132人目の素数さん:01/10/30 20:14
>>198
こうだとおもう
非負の無限数列{a(n)},a(n)≧0,n=0,1,2,・・・がある。
そのとき関数f(z)=Σ_[n=0,∞]a(n)Z^nが、|z|<1で収束すると仮定する。
このとき
lim_[n→∞](1/n)Σ_[n=0,n-1]a(n)<∞
となるための必要十分条件は、
lim_[z→1-0](1-z)f(z)<∞
が成り立つことであることを証明せよ。またこのとき両極限値が等しいことをしめせ。
200132人目の素数さん:01/10/30 20:15
>>199
4行目ちがった。

非負の無限数列{a(n)},a(n)≧0,n=0,1,2,・・・がある。
そのとき関数f(z)=Σ_[n=0,∞]a(n)Z^nが、|z|<1で収束すると仮定する。
このとき
lim_[n→∞](1/n)Σ_[k=0,n-1]a(k)<∞
となるための必要十分条件は、
lim_[z→1-0](1-z)f(z)<∞
が成り立つことであることを証明せよ。またこのとき両極限値が等しいことをしめせ。
201132人目の素数さん:01/10/30 20:17
今日伊藤家の食卓でやってたトランプ見た人、
数学的理論で説明できるといってたが具体的に
どうなってるのか説明してくれ。
202132人目の素数さん:01/10/30 20:20
時間無いので>152は真面目にやらんけど、ぱっと見で答えます。
#とりあえず左極限であることは無視(w

>lim_[z→1-0(左側極限)](1/n)Σ_[n=0,n-1]a(n)=A<∞

これは平均ですな。n項の相加平均

>lim_[z→1-0](1-z)f(z)=A

これはf(z)がz=1で高々1位の極で、留数が-Aですか?
が同値であるという問題じゃないの?

じゃ、頑張ってくれ(w
>201
そういう問題は自分でビデオに取って
少なくとも問題書けるようになってから来い
204132人目の素数さん:01/10/30 20:23
>>202
ちがうよ。f(z)の収束半径は|z|<1でz=1が極でないけど
lim[z→1-0]f(z)が有限確定値になる例はやまほどある。
だからそんな方針ではだめなのよ。
205132人目の素数さん:01/10/30 20:27
>>204
ちょっといいかた不正確。でもわかるよね。たとえば|Rez|<1&|Imz|<1
でさだまる4角形と単位円をむすぶ正則写像(リーマンの写像定理から
存在する。)で4角形の角が単位円のz=1に対応するものをとれば
等角写像の原理からz=1をふくむ領域に正則に拡張することはできない
ので留数もへったくれもないのよ。
206132人目の素数さん:01/10/30 20:38
>>205
lim[z→1-0]f(z)ではなく
lim[z→1-0](1-z)f(z)では?
207132人目の素数さん:01/10/30 20:44
>>206
いやいや。ともかくf(z)のz=αにおける留数が定義できるためには
すくなくともz=αをかこう領域に正則に拡張できんとだめなんじゃないの?
それが仮定からはそんなに自明にはいえないといいたい
つまり問題の仮定をみたすf(z)でz=1を中心とするローラン展開を
もたないのがあるといいたい。
208132人目の素数さん:01/10/30 20:49
>>202,>>206
もしかしたら
単位円|z|<1で定義された関数g(z)(=(1-z)f(z))がlim[z→1-0]g(z)
をもつときg(z)はz=1で正則(またはローラン展開をもつ。)
なんて定理あんの?聞いたことない。てかなりたたないんじゃないの?
209132人目の素数さん:01/10/30 20:52
>>202,>>206
というかその方針でできそう?できそうならぜひ解答うぷしてYO!
これ昨日からずっとわかんなくて一日気になってしかたないよ。
210132人目の素数さん:01/10/30 20:53
いや>>202の2行目にとりあえず無視と
211132人目の素数さん:01/10/30 20:55
>>204-205=>>207-209
の早とちりということですか?
何か興奮して書きまくってる人がいると思ったら
そういうことか
213教えてください:01/10/30 20:58
3次関数f(x)=x~3-x~2+axがあり、f(x)は常に増加している。更に、f(x)の逆関数をg(x)とするとき、2曲線y=f(x),y=g(x)は原点で共通の接線を持つものとする。
(1)定数aの値を求めよ。
(2)2曲線y=f(x),y=g(x)で囲まれた部分をx軸の周りに回転してできる回転体の体積を求めよ。

どなたか教えてください。お願いします。
214132人目の素数さん:01/10/30 20:58
>>210
ああ、つまりz=1を囲う領域に拡張できるならいえるってこと?
確かにその場合は問題ないね。でもこの問題の難点はそんなこと仮定
できんってことだと思うんよ。おれにはむづい。しかも実は前の方にある
まちがった解答つくったのオレ。くやしくって今日いちにちはがゆかった。
誰か解答うぷしてちょ。
215ちむ教の信者:01/10/30 21:02
中心の座標は(b/2a,c-b^2/4a)<中心ってなんのこと?
頂点?
a<0,b<0のときc-b^2/4a<0であるためにはc<0。ってこと。<CがB^2より、
おおきいかわからないのでは?
cの判定はx=0を代入してグラフを見れば、一発。
y切片でしょ。cって<では、最初にCを求めろということですね。
ありがとうございました。

ありがとうございます。
数列で質問があるのですが、
p、qが定数のとき、一般項anがan=pn+qの
形で表される数列であることを示せ。

で、an+1とするとき、pnの項が、pn+1
していい理由がわかりません。
nはn項めをということ。
答えが変わるのでは?
わかりやすく教えてください。
ごめんなさい。どうしてもわからないのです。教えてください。
216132人目の素数さん:01/10/30 21:04
>>152は結構難問だが、久々にじっくり長時間考えられる問題になりそう。
この問題だけ専用のスレが欲しいね。
217質問です。:01/10/30 21:17
x+y+z=4
xy+yz+zx=5
xyz=1       の時

@ x2(二乗です)+y2+z2
A x3+y3+z3
B
   1 1 1
   _+_+_
   x y z
   
ってどうやって解くんですか?
見づらい式ですいません。
218132人目の素数さん:01/10/30 21:21
>>217
x^2+y^2+z^2=(x+y+z)^2-2(xy+yz+zx)
x^3+y^3+z^3=(x+y+z)^3-3(x+y+z)(xy+yz+zx)+3xyz
1/x+1/y+1/z=(xy+yz+zx)/xyz
219工学系:01/10/30 22:05
>>180

ちょっとまって!有効数字の桁数の数え方。
頭の(一番左の)「1」については数えないって慣習があるのは、
工学の世界だけかい?2以上なら有効桁に入れるが「1」は入れないものだと
思っていたのは俺とその周囲だけかい?あれれれ?
数学の時間にそう習った気がするのは気のせいか。

たとえば、12,345,678を有効数字3桁であらわせば、1.234x10^7だろ?
23,456,789を3桁であらわせば、2.34x10^7だよな?
220132人目の素数さん:01/10/30 22:08
厨房質問多いな。
ちゃんと>>2-5読め
最低限ある程度は考えろや。
221質問です。:01/10/30 22:12
どうやって次の循環小数を分数で表すんですか?
  ・ ・
0.123

    ・・
1.1414

答えはわかってるんでやり方を主に希望。
222132人目の素数さん:01/10/30 22:15
123/999
1.14+14/99×1/100
223219:01/10/30 22:15
うわー、だめだ。「有効数字」でweb検索かけても、
どこにも俺が覚えてた考え方(一番左の1は無視)ってのはなかった....

いったい、どこでそんな考え方を教わったんだろう。鬱.....
すんません、逝ってきます....
224132人目の素数さん:01/10/30 22:16
↑ヒントは無限等比級数の和の公式
>>221
循環部がn桁のとき、(10^n)倍して元の数を引く。
循環部をカッコで囲うことにすると、、、

>0.(123)

.    1000X=123.123123123・・・・・
 −)     X=  0.123123123123・・・・・
  ̄ ̄ ̄ ̄ ̄ ̄ ̄ ̄ ̄ ̄ ̄ ̄ ̄ ̄ ̄ ̄ ̄ ̄ ̄ ̄
     999X=123
        X=123/999
         =約分する

>1.23(45)

.    100X=123.454545・・・・・
 −)    X=  1.23454545・・・・・
  ̄ ̄ ̄ ̄ ̄ ̄ ̄ ̄ ̄ ̄ ̄ ̄ ̄ ̄ ̄ ̄ ̄ ̄ ̄ ̄
     99X=122.22
       X=122.22/99
        =12222/9900
        =略

「0.000・・・=0」なのはなぜ?
などと考えてはいけません。
>221
100・・・0倍して、下の方の桁が重なるようにして引き算すれば下の方の無限に伸びるところは消える
227132人目の素数さん:01/10/30 22:42
今日も元気に轄ユり
228132人目の素数さん:01/10/30 22:42
>223
そんなだから工学系しか逝けなかったんだろーね
229質問です。:01/10/30 23:02

  (1994−28b) が、17の倍数で
 bは、0<b<33の整数のとき、bは何か?
230132人目の素数さん:01/10/30 23:10
>>229
b=0とおいて1994/17を計算してみなさい。
あまりはいくつ?
そのあまりが0になればいいわけだろ?
んじゃbにナニ入れたらいいでしょうか?
231質問です。:01/10/30 23:33

x+y=2のとき
xyの最大値。

2x+y=4の時x^2+y^2の最小値

x^2+y=3の時x^2−2yの最小値

x≧0、y≧0、x+2y=6の時xyの最大値と最小値。

よろしくおねがいします。
232質問です。:01/10/30 23:34
はやく解答しろ!!
233132人目の素数さん:01/10/30 23:38
くだらない質問なのですが、ここでお答えされている方々は
大学生なのでしょうか?院生なのですか?
234132人目の素数さん:01/10/30 23:42
>>233
両方
235132人目の素数さん:01/10/31 00:01
旧帝大ですか?
236231:01/10/31 00:16
232はわたくしじゃありません。
お気をわるくなさらるよう
237質問です:01/10/31 00:23
もうわかったからいいや
ここの奴ら俺よりブァカばっか
238132人目の素数さん:01/10/31 00:24
0≦2sinθ+cosθ≦1のとき
sinθ,cosθ,sinθ+2cosθの値の範囲を求めて下さい
239132人目の素数さん:01/10/31 00:28
>>231
xyの最大値は、x+y=2なのでy=-x+2をxyに代入。
すると、x(-x+2)となり、これは平方完成すると、

   -(x-1)^2+2 となり、-(x-1)^2は常にマイナスなので、
これが0になるときが最大となる。
よってx=1,このときy=1。最大値xy=1となる。
240132人目の素数さん:01/10/31 00:35
>>235
もいればそうでないのもいる
241質問です。:01/10/31 00:36
もしかしてこの板レベル低い(ププp
242132人目の素数さん:01/10/31 00:36
残りも同じように代入→平方完成してけばよい。
2x+y=4→y=-2x+4→x^2+(-2x+4)^2→5(x-8/5)^2+16/5 よってx=8/5

以下 略
243質問です。:01/10/31 00:41
こんな問題も解けないのか。
ホント俺よりヴァカな奴らしかいないみたい(藁
>>231
質問すれば答えが返ってくるのが当たり前と思ってんじゃねえ、厨房が。
解答方針が書いてあるだろ。これで理解できないか?

で、x, yは実数と書かないとダメだぞ。
例えば x = 1 + ai, y = 1 - ai (a:実数)とかすると x+y=2で…

だが最後のはx, yが0以上というのがあるからこんな風にすると楽:
x≧0, y≧0であるから、相加平均と相乗平均の関係が使えて
  x + 2y ≧ 2√(x・2y) = 2√2・√xy
一方、x + 2y = 6 より
  6 ≧ 2√2・√xy
  ∴xy ≦ 9/2 (等号は、x = 2y, すなわち x = 6/2, y = 3/2 のとき)
また、x≧0, y≧0 より xy ≧ 0(等号はx=y=0のとき) である。
なぜならばxy < 0となることはないから。
よって、0≦xy≦9/2
245質問。:01/10/31 00:53
(a+b+1)(a−b+1)(a+b−1)(a−b−1)

(2a−b)^2(2a+b)^2(4a^2+b^2)^2

(5p+q)^3(5p−q)^3

(m+n)(m−n)(m^2+mn+n^2)(m^2−mn+n^2)

解いてちょ。
246質問です。:01/10/31 00:56
数学版ってボンクラの集まりだな。
>>243
どんな問題?
248239と242だけど:01/10/31 00:58
>>245
くだらない問題はここへ書けのスレッドに書き込んだほうが良いのでは?
249132人目の素数さん:01/10/31 01:08
0≦2sinθ+cosθ≦1のとき
sinθ,cosθ,sinθ+2cosθの値の範囲を求めて下さい
250132人目の素数さん:01/10/31 01:09
>>240
ずばり、東大?
251132人目の素数さん:01/10/31 01:40
>>245
解いてちょ…って、展開しろ、ということか?
(a+b+1)(a-b+1)(a+b-1)(a-b-1)
={(a+1)+b}{(a+1)-b}{(a-1)+b}{(a-1)-b}
={(a+1)^2-b^2}{(a-1)^2-b^2}
={(a^2-b^2+1)+2a}{(a^2-b^2+1)-2a} = ……

その2={(2a-b)(2a+b)(4a^2+b^2)}^2=……
その3={(5p+q)(5p-q)}^3 = ……
2,3は(A+B)(A-B) = A^2 - B^2の応用問題

その4=(m^2-n^2){(m^2 + n^2)^2 - (mn)^2}=(M-N)(M^2+MN+N^2)=……
(Mはm^2, Nはn^2ね。展開公式使ってM^3-N^3)

>>238
泥臭い方法しか思いつかなくてすまん。誰かエレガントな解法を求む。
三角関数の合成により、与えられた不等式から
0≦√5・sin(θ+α)≦1 (ただし、αはsinα=1/√5, cosα=2/√5を満たす数)
よって、0≦sin(θ+α)≦1
単位円を書いて考えればわかりやすいと思うけど、θの範囲は
0 + nπ≦θ+α≦α+nπ, π-α + nπ≦θ+α≦π + nπ(nは整数)
で表される。
これから、sinθ、cosθの範囲は求まると思う。
まあ、一般角で書いたけどわかりにくければ、これは
0≦θ+α≦α, π-α≦θ+α≦π
という範囲にθがあると考えて良いから。
つまり
-α≦θ≦0, π-2α≦θ≦π-θ
で、最後のはa≦sinθ+2cosθ≦bとする。
sinθとcosθの範囲を単純に足しただけじゃだめそうだからだ(あくまで勘だけど多分ダメ)
与えられた不等式と足して、a≦3(sinθ+cosθ)≦b+1
a/(3√2)≦sin(θ+π/4)≦(b+1)/(3√2)
sin(θ+π/4)の範囲は求まるから比較して終わりっと。

多分、-1/√5 ≦ sinθ ≦ 0, 1/√5 ≦ sinθ ≦ 4/5
他は自信ないので自分で計算してくれ。
>>251
>0≦√5・sin(θ+α)≦1 (ただし、αはsinα=1/√5, cosα=2/√5を満たす数)
>よって、0≦sin(θ+α)≦1

よって、0≦sin(θ+α)≦1/√5・・・かと。
253251:01/10/31 07:28
>>252
その通り。訂正ありがとう。
他にも数字がおかしいな
×>0 + nπ≦θ+α≦α+nπ, π-α + nπ≦θ+α≦π + nπ
→○0 + 2nπ≦θ+α≦α+2nπ, π-α + 2nπ≦θ+α≦π + 2nπ
とか。
ロジックはあってると思うので>>238よ頑張ってくれ。頼りなくてすまん。
>>238
c{x|−2/√5≦x≦−3/5}∪{x|2/√5≦x≦1}。
s{x|−1/√5≦x≦0}∪{x|1/√5≦x≦4/5}。
2c+s{x|−3/√5≦x≦−2/5}∪{x|3/√5≦x≦2}。
255132人目の素数さん:01/10/31 10:39
>230
良く分かりません。
256132人目の素数さん:01/10/31 12:14
質問:「極限を持つ」とはどういうことなのですか?「持たない場合」というのは、どういうケースを言うのでしょうか?
257132人目の素数さん:01/10/31 12:32
>>256
いろいろ
258名無し:01/10/31 12:42
>>229
1994-28b=(117*17+5)-(2*17-6)b=(117-2b)*17+5+6b
これが17の倍数になるのは,5+6bが17の倍数になるときである。
5+6b=7nとすると,7n-5=6b
0<b<33より,7n-5が6の倍数になるのは,
7*7-5=6*5,7*11-5=6*12,7*17-5=6*19,7*23-5=6*26
である。つまり,b=5,12,19,26 (← 7ずつ増えることに気づけば,途中からはもっと楽かも)
259258:01/10/31 12:46
すまん,途中から17が7に摩り替わってしまった。もう一度。
1994-28b=(117*17+5)-(2*17-6)b=(117-2b)*17+5+6b
これが17の倍数になるのは,5+6bが17の倍数になるときである。
5+6b=17nとすると,17n-5=6b
0<b<33より,17n-5が6の倍数になるのは
17*1-5=6*2,17*7-5=6*19
である。つまり,b=2,19
260無題:01/10/31 12:50
素朴な質問なんですが、n(n+1)<650をみたす最大のnの出し方というのは、
1つ1つ当てはめていくしかないのでしょうか?
何か、他にいい方法はないのでしょうか?
261>260:01/10/31 12:57
(n+1/2)^2<650.25
(2n+1)^2<2601
262132人目の素数さん:01/10/31 13:01
>>260
n(n+1)<650
n^2 + n < 650
(n + (1/2))^2 - (1/4) < 650
(n + (1/2))^2 < 650 + (1/4)
263132人目の素数さん:01/10/31 13:01
叶{磨
264132人目の素数さん:01/10/31 13:13
>>257
「いろいろ」とは?
例を幾つか教えて下さい。
265132人目の素数さん:01/10/31 13:14
つーか、25^2=625ぐらい覚えとけよ(w
265は>>260
>>264
x→∞で発散する関数の例

f(x)=x
f(x)=x^2
f(x)=x^3
f(x)=x^4
f(x)=x^5
f(x)=x^6
f(x)=x^7
f(x)=x^8
f(x)=x^9
f(x)=x^10

10個も挙げてやったんだから十分だろ?
268132人目の素数さん:01/10/31 13:36
>>267
「発散する」=「極限を持たない」
ということでよろしいか?
269132人目の素数さん:01/10/31 13:37
>>267
「発散しない」=「極限を持つ」
ということでよろしいか?
270132人目の素数さん:01/10/31 13:37
>>267
「有界」ってことと同じなの?
↑ばーか
x→∞で極限を持たない有界な関数の例

f(x)=sin(x)
f(x)=sin(2x)
f(x)=sin(3x)
f(x)=sin(4x)
f(x)=sin(5x)
f(x)=sin(6x)
f(x)=sin(7x)
f(x)=sin(8x)
f(x)=sin(9x)
f(x)=sin(10x)

10個も挙げてやったんだから十分だろ?
273132人目の素数さん:01/10/31 13:49
>>272
なるほど。「有界」は値域で与えられた範囲なのに対して、
操作しているxのほうは定義域ってわけ・・・・ですか?
で、この例の場合、f(x)は有界だがその値はグルグル動き
回って収束しない、つまり極限を持たない、ということ
・・・・でいーですか?
よし
教科書で定義の内容確かめれ
276教えて君:01/10/31 13:57
よーするに、「極限を持つ」とは、ある変数xを無限大にまで
操作したときに、その関数の値がある収束先を持つ、というこ
とですか?
で、「有界」とは、ある変数xを操作したときに、その関数の
値に上限(あるいは下限)がある、ということですか?
277132人目の素数さん:01/10/31 13:58
>>275
おいおい。内容に自信ないのかよ?
教科書もろくに読めないような文盲は去れ
>>276
だいたいよし
280教えて君:01/10/31 14:09
>>279
「だいたい」ということは、もう少し正確に言えばどういうところがダメですか?
281132人目の素数さん:01/10/31 14:14
>ある変数xを無限大にまで操作したときに
ダメ
282教えて君:01/10/31 14:20
>>281
「ある変数xを無限に大きくしていったとき」
「ある変数xを無限大に近づけていったとき」
では?
283古潭らーめん:01/10/31 17:42
y_n+1 = a*y_n*(1-y_n) (0≦y_n≦1) を考える。

3.57< a ≦4 のとき、y_1 が0,1以外の値であれば、充分大きい n に対する y_n の値は
(1/4)*a~2*(1-(a/4))≦ y_n ≦ a/4 を満たすことを示せ。
284名無しに戻って:01/10/31 18:55
等差中項って、教科書にはのってないんだけど、
どういうこと?
また、調和数列ってどういうこと?
285問題ですよ:01/10/31 18:57
この問題、結構悩んでやっと解けたのですが、ボクの解答よりスマートに
解けた方いらっしゃったら、レスください。大学受験にもでたみたいです。
ボクの解答はあとで載せます(タブン)。

Q.n個の箱とn個の玉があります。それぞれの箱と玉には1、2、…、n
と番号がかかれています。n個の箱に1個づつ玉を入れるとき、箱と玉の番号が
一致せず全て異なっているような入れ方の総数をA(n)と記することにしましょう。
そのときA(n+1)とA(n)のあいだに成り立つ関係式はどんなんでしょう?

ボクの解法はかなり泥臭いやり方で解きました(たぶん、正解だとおもうけど)。
チャレンジしてみてください。
286132人目の素数さん:01/10/31 22:20
>259
最後の方の、6の倍数である事から2と19をだすのに、
19は自力で19まで計算したんですか?大変かと思いまして。
287132人目の素数さん:01/10/31 22:30
18%の食塩水400グラムに6%の食塩水500グラムを何グラム入れると14%になる?
誰か教えてください(TT)
288問題ですよ:01/10/31 22:30
kの倍数の集合をK()、
lの倍数の集合をL()、
(k+1)(l-1)の倍数P()とすると、
K()+L()<P()<=K()*L()
であることを示せ
>>288
k=2、l=3とおいて、2∈K()、3∈K()、4∈P()において
2+3>4
従って問は成立しない。
290素朴な疑問:01/10/31 23:10
円Oに接線ABを引いてその接点Pに中心から線を引くとなぜAB⊥OPとなるのですか?
この証明が参考書に載ってないんです、宜しくお願いします。
291132人目の素数さん:01/10/31 23:25
>>290
円の定義から明らか。
接点Pは円Oから直線OPへの最短の足のはず。
292初心者:01/10/31 23:31
整数全体の集合を一般に
「Z」であらわすと思うのですが、
何か由来と言うか、語源が有るのですか?
お願いします。
293132人目の素数さん:01/10/31 23:32
ある
294132人目の素数さん:01/10/31 23:34
>291訂正・・・直線OPへの最短の足→直線ABへの最短の足
>291に追加
逆に考えてみよう。
点Oから直線ABへ垂線を下し、その足がPになったとしよう。
それで半径OPの円を作図する。
OPはABへの最短ルートのはずだから、P以外に共有点はないはず。
よってABはPを接点とする接線となるはず。
295132人目の素数さん:01/10/31 23:35
結論としては円、接線の定義から明らか
>>292
ゾフィーのZ
>>291>>294
円の定義が違うと思う
というか、ちゃんと定義しないとトートロジーになっちゃうよ
298132人目の素数さん :01/10/31 23:43
ギリシャ乗法のアルゴリズムを教えてください。
すいません、もうひとつ。20桁×20桁の計算を
8桁表示の計算機でするにはどうすればいいでしょうか?
お願いします。
299教えてくだちい:01/10/31 23:48
サンドイッチの作り方。

パンは6種類の中から(必ず一種類)
ハムは4種類の中から2種類まで、(挟まなくても可)
チーズも4種類の中から2種類まで、(挟まなくても可)
ガーニッシュは12種類、全て選んでいい。(挟まなくても可)

全部でパターンはいくつっすか?
300132人目の素数さん:01/10/31 23:49
>>298
対数使いなよ。
301132人目の素数さん:01/10/31 23:50
ガーニッシュってなに?
302初心者:01/10/31 23:50
>>296
ふ━━( ´_ゝ`)━━ん
303132人目の素数さん:01/10/31 23:51
6×(4C2+4+1)^2×2^13
304教えてくだちい:01/11/01 00:05
あのー、結局299の答えはいくつなんですか?
305132人目の素数さん:01/11/01 00:05
>>301
(⌒, -- 、⌒)
 Y      Y
 | ・  . ・| < 『質問です』って名前で質問を始めてくれると見つけやすくて助かるわー

http://www.pluto.dti.ne.jp/~bbb/eqhp/Baking1.htm
306132人目の素数さん:01/11/01 00:10
結局>>152はどうなった?
俺じゃあ解けない〜〜
307アリストテレス:01/11/01 02:05
みなさんにつかぬ事をお伺いするが、ハミルトンの四元数体を大学の授業で習われた方はおられるか。
それから習われたのは何年生であったか教えて欲しい。標準メニューにあるのだろうか。
それとも専門すぎるか。
308132人目の素数さん:01/11/01 02:12
うちでは2回生配当科目で習うと思われる。
309KARL ◆gjHKPQSQ :01/11/01 03:33
>>292
ドイツ語のZahl(ツァール:数)に由来すると思います。
>307
2年生までに習っていると思う。
311132人目の素数さん:01/11/01 05:46
R×R−>Rの関数gが任意の実数a,b,cに対して
g(a,b)+g(a,c)=g(a,b+c)
g(g(a,b),c)=g(a,bc)
g(a,c)+g(b,c)=g(a+b,c)
g(a,1)=a
となるときgを全て求めよ。
これ分かる人いますか。
313 :01/11/01 08:27
>>290
背理法(実際には対偶を示しているだけだが)を使う。

直線XYが、点Xと点Yの間の点Pにおいて、
点Oを中心とする円に接している。

0°< ∠OPX < 90°と仮定する。
このとき、円周上に点Aを
∠AOP = 180°- 2∠OPX
となるように、点Xと同じ側に取る。
すると、三角形AOPは OA=OP の二等辺三角形なので、
∠OPA = (1/2) (180°- ∠AOP) = ∠OPX
となる。つまり、点Aは半直線PX上にある。
特に、直線XYは相異なる二点A,Pで円Oと交わっている。
これは、直線XYが円Oに接するという仮定に反する。

同様に、0°< ∠OPY < 90°と仮定しても矛盾。

以上より、
直線XYが、点Xと点Yの間の点Pにおいて、
点Oを中心とする円に接しているとき、
∠OPX = ∠OPY = 90°
であることが示された。
314初心者:01/11/01 09:37
>>309
ありがとうございます。
315cc:01/11/01 09:43
ラグランジェ補間について、詳しい方おしえてください。
いま、曲線について勉強しているのですが、今後
役に立ちそうなので。
>312
コピペやめれ
荒らすな
317132人目の素数さん:01/11/01 10:00
132番目の素数って774か?
318132人目の素数さん:01/11/01 10:02
それ偶数。。。
319132人目の素数さん:01/11/01 10:03
>>317
ボケ!それって偶数だろ!
320132人目の素数さん:01/11/01 10:04
ダブった
321132人目の素数さん:01/11/01 10:07
しかしここのHNの由来は132番目の素数=774さんだよ。
>321
132番目の素数=743
323[質問]確率が分かりません:01/11/01 13:46
質問です。

ここに、偏りのないコインがあります。そして、コイントスマシンがあったとし
ます。とりあえず、100億回くらい試行したところ、誤差1パーセントくらい
で表裏の出現が半々の分布になったとします。

さて、ここで、A君がやってきて、そのトス現場を見ました。A君が超能力者
だった訳でもないのでしょうが、そのあと、1000回分、トス結果が「表」とな
りました。

確率論から言うと、その次のトスでは表裏の出現確率は半々である、となるので
しょうが、同時に、大数を取ったとき、偏らないということから、裏の出現確率
が高くなるようにも思えます。そして、十分にたくさん試行を重ねると、大数の
法則どおりになるでしょうから、やっぱり、分布に(優位な)偏りが出るというこ
とにならんのですか?
100億回で誤差1%もあったら偏りがないとは言えないだろ。
325132人目の素数さん:01/11/01 15:02
7743が素数だったらな・・
326[質問]確率が分かりません:01/11/01 15:12
>>324
ごめんして。

>>323
十分な精度で半々の分布になる。

でゆるしてください。このとき、どうなるのでしょうか。
「ごめんして」

            ( )
           //
    ∧_∧    ミ)
   (・∀・)_//   / ̄ ̄ ̄ ̄ ̄ ̄ ̄ ̄ ̄ ̄ ̄ ̄ ̄ ̄ ̄
  /     //   < >>326死語は慎んで下さい!
 (   _ //つ    \_______________
  \ / //(
   (彡/   )
    \___∧
    //  \\
    ││   //
   (_) (_)
328132人目の素数さん:01/11/01 16:14
>>323

偏りのないコインを仮に100億トスすれば、同じ面が連続して出る
カタマリが平均50億、うち31連以上のカタマリが平均約5個、
と考えると1000連は99.9%でははるかにすまない異常、
→100億トスと1000トスが同じ条件とはとてもとても…となる。
>大数を取ったとき、偏らない

これが違ってる。大数の法則は偏りが減ることしか保証しない。
そもそもなぜこんな保証が出来るのかと言うと
1000回連続で表が出ようが次の目に影響を与えないから。
330132人目の素数さん:01/11/01 17:06
331[質問]確率が分かりません:01/11/01 18:40
>>323
では、少し質問を変えてみます。

ある星に人類が到着しました。すると、コイントスマシンが設置されていて、表→0
裏→1をずっと表示していました。Aがそのマシンを見つけ、100億回のトスを
検証したところ、1パーセント分、0になる確率が多そうでした。ストップボタンが
見つかったので、それを押して、Bを呼びに行きました。Bに、100億回のトスを
検証させると、今度は、1パーセント分、1になる確率が多そうでした。そして、
ストップボタンを押しました。

そして、AはBに対して賭けを申し出ました。次のトスはどちらが出る?

さて、ここで質問。Aはどちらにベットしたでしょうか。Bはどちらにベットした
でしょうか。
332132人目の素数さん:01/11/01 18:42
男は男、女は女に決まってます
333132人目の素数さん:01/11/01 19:59
>285
解けた方いませんかぁぁぁ。答えは
A(n+1)=(n+1)A(n)+(-1)^(n-1)
だと思うんですが。簡単よ。しょせん入試問題だしねぇ。
334132人目の素数さん:01/11/01 20:01
>>331
コインがない・・・。
335風呂上がりの変態:01/11/01 20:06
ルートってどうすれば記号で出せるんですか?
336質問です:01/11/01 20:08
円筒座標系の解はベッセル関数になるとおもうんですけど(1次元の円筒)
この円筒は中が詰まっている(原子が詰まっていると考えるのかどうかはわかりませんが)
として解いていると思うんですけど、中が中空の場合(ある厚みをもつ円筒)
の場合はエネルギー準位はどうなるのですか?
すいません、物理わかってないですが分かる方宜しくお願いします(上記の内容がすでに
分からないかもしれませんが・・・)
337132人目の素数さん:01/11/01 20:08
「るーと」で変換
338132人目の素数さん:01/11/01 20:09
「るーと」って打って変換すればでるよ。
339名無しに戻って:01/11/01 20:10
てか、このすれこたえろ!!
340132人目の素数さん:01/11/01 20:10
かぶすま
341風呂上がりの変態:01/11/01 20:10
ルート
Route
343132人目の素数さん:01/11/02 00:31
>>336

いいたい事がわからん。
そもそもそんなことやって物理的な意味があんの?
>>336
っていうか何故数学板に?
345132人目の素数さん:01/11/02 04:40
これって数学でっすか?ここであってますか?

正直者とうそつきが住む村がある。
旅人が村人に「あなたは正直者ですか?」
とたずねたところ
「私か妻のどちらかはうそつきだ・と答えた。
この住人と妻のそれぞれは正直かうそつきか?

って問題です。p→qみたいな、論理学の問題です
はっきりいってサパーリなのでよろしくお願いします。
村人が嘘つきだとすると、「私か妻のどちらかはうそつき」が本当の事となるので矛盾。
という問題なのかなぁ・・・
ナンカイマイチ
347132人目の素数さん:01/11/02 04:49
いろいろ仮定して考えてね。
住人がうそつきなら両方うそつきかしかありえないが、それだと
正直がいなくなるからダメ。
結局、住人は正直で妻がうそつき。
348132人目の素数さん:01/11/02 04:50
全部否定と部分否定を区別することがポイントだよ。
349345:01/11/02 04:54
え?全然分からん・・・
矛盾するのか?
え〜っとえ〜っと。。。
その村人は独身。
351132人目の素数さん:01/11/02 04:57
>>350
チョットワラタ!
352132人目の素数さん:01/11/02 04:57
まず、条件として、正直とうそつきが1人ずつ必要。
住人がうそつきで妻が正直なら、住人は本当のことを言っているので却下。
住人が正直で妻がうそつきなら、住人は本当のことを言っているのでOK。
353132人目の素数さん:01/11/02 05:00
ポイントは「私か妻のどちらかはうそつきだ」が偽だとすると
「私と妻の両方がうそつき」もしくは「私と妻の両方が正直」が真となること。
354132人目の素数さん:01/11/02 05:02
>>345
h=を夫、wを妻、1を正直、0を嘘つきとすると
h=(1-h)+(1-w)-2(1-h)(1-w) ⇔ w(1-2h)=0 ⇒ w=0
よって妻は常に嘘つき夫はどちらでも良い、
排他的でないとすると
h=(1-h)+(1-w)-(1-h)(1-w) ⇔ h(1+w)=1 ⇒ h=1かつw=0
やはり妻は嘘つき、夫は正直
355132人目の素数さん:01/11/02 05:05
妻、住人両方うそつきの場合は
「どちらかがうそつきである」を満たすんですよね?
どちらかがってXですよね?
356132人目の素数さん:01/11/02 05:10
>>355
日本語の問題じゃ。
出題者に聞いてくれ。
そういう場合普通は「少なくとも1人はうそつき」で「いずれか」と「両方」
を含めると思う。
「いずれか」のときは「両方」を含まないと思う。
357132人目の素数さん:01/11/02 05:11
こういうことでしょうか?

正直=1
うそつき=0

住人がうそつきなら
住|妻
ーーー
0|0←どちらかがうそつきを満たすので矛盾
0|1←これも満たすので矛盾
1|0←仮定違うので矛盾
1|1←同上
ってことで全部矛盾

住人が正直なら
住|妻
ーーー
0|0←仮定と違うので矛盾
0|1←同上
1|0←どちらかがうそつきを満たしつつ、住人が正直を満たすのでOK
1|1←どちらかがうそつきを満たさないで矛盾
358132人目の素数さん:01/11/02 05:14
てゆーか、正直者とうそつきは1人ずついるという設定なんじゃねーの?
359132人目の素数さん:01/11/02 05:20
共立的(包含的?)選言としないと、解が一つにならない。

>>357
そうそう
360357:01/11/02 05:24
みなさんどうもありがとうです。

普通はこういう問題は

p p->q
------
q

みたいに推論規則つかってやるんでしょうか?
>>357みたいな感じのでもいいんでしょうか?
361132人目の素数さん:01/11/02 07:46
う〜ん、困った。
(1)SO(2)={A:二次実正方行列。しかも直交行列で、その行列式は1}
SO(2)がS^1(単位円周)と同型であることを示せ。
(2)SU(2)={U:二次複素行列。しかもユニタリ行列で、その行列式は1}
SU(2)がS^3と同型であることを示せ。
まだ多様体に不慣れなものでして・・・。よろしくお願いします。
362132人目の素数さん:01/11/02 09:15
a[n+1]=4a[n](1−a[n])

0<a[1]<1 として
a[n]の分布関数を求めよ


・・・ちょっとさっぱりわかりません
よろしくお願いします
363132人目の素数さん:01/11/02 11:02
>>361
(1) 回転行列って知ってる?
(2) 有名な事実だがこちらの方はすこし骨かな。
  ハミルトンの4元数体で「絶対値1」の元全体は積について
  閉じているな。ところで4元数体は2次の複素正方行列でも
  書けるんだな、これが。
  
364363:01/11/02 11:06
ハミルトンの4元数体はこの問題を解くだけならいらないな。
蔚山車濃。(注 山車=だし)
365132人目の素数さん:01/11/02 11:13
>>360
指示がなければどっちでもいいはず。
当然ながら、真理値表を作るような方法>>357は、
複雑な問題では爆発的に時間がかかるのでおすすめできないが、
簡単な問題なら有効。
366132人目の素数さん:01/11/02 17:22
ちょっと板違い気味ですが、数学板のほうが良い答えが返ってきそうなので
ここに書きます。

プログラムしていてちょっと困ってることがあるんですが、
紙の上でかけなくなったら、どうしたら良いか分からなくなってしまいました。

x^2 とかを取り合えず x2 と書くことにします
 | x1  x2  x3  x4 ...
-------------------------
y1|y1x1 y1x2 y1x3 y1x4
y2|y2x1 y2x2 y2x3 y2x4
y3|y3x1 y3x2 y3x3 y3x4
.
.
.

みたいな形で直積を作ってこれをより次数の小さいほうから
たどっていくアルゴリズムを作っています。
y1x1 -> y1x2 -> y2x1 -> y1x3 -> y2x2 -> y3x1 ...
とななめに進んでゆけば、いくらでも大きい x^n を含む直積を
列挙できますが、x y z ... と、こちらもいくつでも追加したいです。

よい列挙方法はないでしょうか?

#強力な自由度の必要な
#コンテナとイテレータを作っているのですが難しい・・・
367361:01/11/02 17:41
>>363
(1)な〜んかクルクルしているイメージはわくけど、初耳なお名前です。
(2)ん?骨なんですか・・・?困った。
368363:01/11/02 17:50
>>361
別に骨じゃない。SO(2)やSU(2)に2次の正方行列が入る条件を成分の
言葉で書いてみ。
369360:01/11/02 19:04
>>365
どうもです。支持は特になかったので
真理値表でいきたいと。。。
でもいつか困るとも宇野で
p->qみたいなやりかたも考えてます
370132人目の素数さん:01/11/02 21:11
二次関数で、解と係数の関係があるのですが、
それって、どのようにして証明されたのでしょうか?
教えてください。
371πパン:01/11/02 21:46
積分についての質問。

∫[0,1]x^2√(1+y^2)dy
って、どうやって解けばいいでしょうか?
√{1+y^(-2)}=tっておけばいいの?
教えてください。
372132人目の素数さん:01/11/02 21:48
>>370
どんな本にも載ってるし、超簡単
373132人目の素数さん:01/11/02 21:48
>>366
それでいいんじゃないのか?

>>369
古典論理では問題ない、はず

>>370
なんのこっちゃい?
(x-α)(x-β)=x^2-(α+β)x+αβ
のことか?
374πパン:01/11/02 22:14
>>371
間違った・・・
文字ぐちゃぐちゃ

訂正
積分についての質問。

∫[0,1]y^2√(1+y^2)dy
って、どうやって解けばいいでしょうか?
√{1+y^(-2)}=tっておけばいいの?
教えてください
375132人目の素数さん:01/11/02 23:09

o
376132人目の素数さん:01/11/03 00:31
y^2=tとでも置いて二回置換積分してもいいし。
y=tantとおくのも面白い。
377132人目の素数さん:01/11/03 00:37
>y=tantとおくのも面白い。
面白くない
378132人目の素数さん:01/11/03 01:40
ラゲールの多項式の重みとやらはどのやうに計算したらでるのですか。
がいしゅつかもしれないのですが、
「どんな4以上の偶数も,2つの素数の和で表わされる」
は正しいか間違っているか。
正しければそれを示し、間違っているなら反例を記せ。

反例思いつかないのですが、正しいのでしょうか?
お願いします。
380132人目の素数さん:01/11/03 02:49
>379
NETA?
世界中の数学者が誰も解けない問題
このスレに書かれても...
381132人目の素数さん:01/11/03 04:10
うぉぉ、それ先に言ってくれ。
思わず640996まで検算しちゃったじゃないか〜。
382132人目の素数さん:01/11/03 04:13
プログラム作ってやったのなら
それなりの財産だが・・・
プログラム書いたよ。めちゃめちゃ簡単な奴を…。
道で1円玉拾ったくらいには財産になりそうだよ。
384質問です:01/11/03 10:32
楕円x^2+(y-1)^2/2=1の内部でy≧0にある部分をx軸の周りに回転して得られる立体の体積を求めよ。

この問題なんですが、解き方教えてください。どうかお願いします
385132人目の素数さん:01/11/03 12:52
3桁×3桁の虫食い算です。(ずれませんように)
□に数が入ります。
20個の□には、0〜9がそれぞれ2つずつ入ります。

よろしくお願いします。

●●●●●●●□□□
●●●●●●×□□□
●●●●●●―――――
●●●●●●●□□□
●●●●●●□□□
●●●●●□□□
●●●●―――――――
●●●●●□□□□□
386132人目の素数さん:01/11/03 13:07
集積点が孤立点であることの証明がどうしてできません。
どなたかご教授ください。
>>385
  179
 ×224
―――――
  716
 358
358
―――――
40096。

>>386
正しくないので証明できない。
388385:01/11/03 15:42
>>387

すごいっ!
こういう解はどうやって求めればいいんでしょうか?
389132人目の素数さん:01/11/03 15:44
「complete set of conjugates」って「共役の完備な集合」という日本語訳で
すか?けど、それって・・・どんな集合なのでしょーか?
文脈みないとわからんがcompleteは完備ではないな。
というかそれ代数の話だろうたぶん
391ティベリウス:01/11/03 16:10
教えて下さい。

納n=1,∞]{(-1)^n}/n
は収束しますか?
出来れば証明付きでお願いします。
392132人目の素数さん:01/11/03 16:14
そうです、二次関数の有るα+β=・・・・・・
って、二つのやつの証明というかどのようにして、導かれたのかです。
393132人目の素数さん:01/11/03 17:00
>>390
禿胴。“共役類の完全代表系”のことだとおもわれ。
394132人目の素数さん:01/11/03 17:03
素数関係の公式にはたいていLogが出てくるのはどうしてですか。
395ふなこ:01/11/03 17:21
>>374

∫[0,1]y^2√(1+y^2)dy
=∫[0,1](1+y^2)^(3/2)dy-∫[0,1]√(1+y^2)dy
=[y(1+y^2)^(3/2)][0,1]-3∫[0,1]y^2√(1+y^2)dy-∫[0,1]√(1+y^2)dy

∴∫[0,1]y^2√(1+y^2)dy=(1/4)*{[y(1+y^2)^(3/2)][0,1]-∫[0,1]√(1+y^2)dy}
=(1/4)*{[y(1+y^2)^(3/2)][0,1]-[(1/2)*(y√(1+y^2)+log|y+√(1+y^2)|)][0,1]}
=(1/4)*{3√2/2-(1/2)*log(1+√2)}
=(1/8)*{3√2-log(1+√2)}
こんなかんじでいいんじゃない?
396361:01/11/03 17:41
>>363
マトンなことを言っていたらゴメンなさい。
とりあえずは「SO(2)の元ってのはS^1をそれ自身にうつす」
ってことを言えばよいのですか?そこから先はどの辺を突っつけばいいのでしょう?
397他スレにゴバクしちゃったぞ、コノヤロー:01/11/03 18:18
>>391
(記号が見えんがΣってことだろコノヤロー)

絶対収束(Σ|a_n|=α)ならば収束(Σa_n=β)ってことが言えるらしいぞ。
「絶対収束」を調べてみれ。

ちなみに
|(-1)^n| = |-1|^n = 1^n = 1
だ。おれもヘタレゆえ自信なしだがなコノヤロー
>>397
(1/n)は収束しないので意味無しヒント
399132人目の素数さん:01/11/03 18:33
つーかそれは条件収束で足す順番によって値が変わるのではないか?
400132人目の素数さん:01/11/03 18:34
>>396
ヨコレススマソ。
ぜんぜんちがうよ。証明すべきなのは
SO(2)={A=[[a,b],[c,d]];ad-bc=1,A~A=E}(ただしA~はAの転置行列)

S^1={(p,q);p^2+q^2=1}
のあいだに同相写像があることをしめせって問題。
>>363さんのヒントはSO(2)の元は
{[[cosθ,-sinθ],[sinθ,cosθ]]}
とかけることを利用すればってことだよ。(ないしはそれを証明せよって事。)
401397:01/11/03 18:36
>>398
その記号ってなんだ?おれのブラウザには「・」としか映らんのだが。
もし級数だったら0に収束するんじゃないの?
記号が読めんせいで、マジだかネタだかさっぱりだ。
402132人目の素数さん:01/11/03 18:47
>>401
1+1/2+1/3+・・・+1/n+・・・ = ∞
403132人目の素数さん:01/11/03 18:47
Σ=
404132人目の素数さん:01/11/03 18:49
機種依存文字だからな。
スマソ。機種依存するシグマがあるのか
406397:01/11/03 18:51
>>402
イテテイテイテキマース
407132人目の素数さん:01/11/03 18:53
すみません。わからない問題があるのですが...。
(X^12+X^11+X^6+X^3+X^2+1)/(X^6+X^4+X^2+1)
の割り算がわかりません。さっぱり。
途中の計算過程も教えていただけませんか?
また、これって何年生くらいの問題なんでしょう?
筆算する
くだらないもんだいですが、頭の良い人たち教えて下さい。

x^3 + \frac{1}{x} ---(*)の 1 \le x \le 2での最小値を求めるために、
相加相乗平均を使って(*) が2x以上とし、よって、x=1 のときに最小値が2
であるとしたのですが、(*) のグラフを増減表を用いて計算すると、x が
3 の4乗根であるときに、どうやら最小値になるようなのです。

いったい、始めの答えの何がおかしかったのですか???
410132人目の素数さん:01/11/03 19:18
>>407
俺は高校1年のときに習った。
              x^6 +x^5- x^4 -x^3 +1
             ノ ̄ ̄ ̄ ̄ ̄ ̄ ̄ ̄ ̄ ̄ ̄ ̄ ̄ ̄ ̄ ̄ ̄ ̄ ̄ ̄ ̄ ̄ ̄ ̄ ̄ ̄
x^6 +x^4 +x^2 +1ノ x^12+x^11 +x^6 +x^3+x^2 +1
x^12 +x^10 +x^8 +x^6
 ̄ ̄ ̄ ̄ ̄ ̄ ̄ ̄ ̄ ̄ ̄ ̄ ̄ ̄ ̄ ̄ ̄ ̄ ̄ ̄ ̄ ̄ ̄ ̄ ̄ ̄
x^11-x^10 -x^8
x^11 +x^9 +x^7 +x^5
 ̄ ̄ ̄ ̄ ̄ ̄ ̄ ̄ ̄ ̄ ̄ ̄ ̄ ̄ ̄ ̄ ̄ ̄ ̄ ̄ ̄ ̄ ̄ ̄
-x^10-x^9-x^8-x^7 -x^5
-x^10 -x^8 -x^6 -x^4
 ̄ ̄ ̄ ̄ ̄ ̄ ̄ ̄ ̄ ̄ ̄ ̄ ̄ ̄ ̄ ̄ ̄ ̄ ̄ ̄ ̄ ̄
-x^9 -x^7+x^6-x^5+x^4+x^3
-x^9 -x^7 -x^5 -x^3
 ̄ ̄ ̄ ̄ ̄ ̄ ̄ ̄ ̄ ̄ ̄ ̄ ̄ ̄ ̄ ̄ ̄ ̄ ̄
x^6 +x^4+2x^3+x^2 +1
x^6 +x^4 +x^2 +1
 ̄ ̄ ̄ ̄ ̄ ̄ ̄ ̄ ̄ ̄ ̄ ̄ ̄
2x^3
411132人目の素数さん:01/11/03 19:22
>>409
そりゃだめ。f(x)≧g(x)でg(x)がx=2で最小だからってf(x)がx=2で最小
なんていえるわけない。1≦x≦2,f(x)=x+10,g(x)=-x+1とかでかんがえればわかるんでは?
相加相乗平均って間違った証明書かせるために習う気がしないでもない
>>406
よく見かけられる有名な誤論だ。
「不等式
  x^3 + 1/x ≧2√(x^3 ・1/x) =2x・・・★
 が成り立つこと、等号成立がx=1のときであること」
は正しい。でもそこから
「よってx^3 + 1/x はx=1のとき最小」
を導くのは筋違い。★の右辺が「定数でない」ことに注意!!!
そんな論法(?)をつかえば、たとえば
「不等式x^2 ≧2x-1 が成り立つ。等号成立はx=1のとき。
 よってx^2 はx=1 のとき最小。」
なんてトンデモ解が言えちゃうよ。

ようするに、「等号成立すること」と「最小値をとること」には
一般には何の関係もないのだ。

なお、本問で相加・相乗平均の関係を使うなら
次のようにするとよい。
(ただ、「4変数の相加・相乗」は証明無しに使うと減点かも):
「x>0において、
  x^3 + 1/x
 =x^3 + 1/3x + 1/3x + 1/3x
 ≧4{(x^3)(1/3x)(1/3x)(1/3x)}^(1/4) =4/(27)^(1/4)
 (等号成立はx^3=1/3x つまり x=(1/3)^(1/4) のとき)」
414406:01/11/03 20:31
>>413
誤爆デス?
415384:01/11/03 20:45
なんか無視されてるみたいなんでもう一度。

楕円x^2+(y-1)^2/2=1の内部でy≧0にある部分をx軸の周りに回転して得られる立体の体積を求めよ。

この問題なんですが、解き方教えてください。どうかお願いします
せめてヒントだけでも
>>415
   x^2+(y-1)^2/2=1
⇔ y=1±√(2-2x^2)

V/(2π)=∫[0,1]{1+√(2-2x^2)}^2dx - ∫[1/√2,1]{1-√(2-2x^2)}^2dx
∫√(2-2x^2)dx=√2∫(1-x^2)dxは円(半円)の面積を考えて出せる
417132人目の素数さん:01/11/03 21:04
>>415
いろいろあるけどバームクーヘン積分していいならそれが楽では?
その楕円とy=kとの交わりのながさは√(1-(k-1)^2/2)なので
その軌跡のつくるわっか状の帯の面積は2πy√(1-(k-1)^2/2)
これをk=0〜3まで積分したものが体積
V=∫2πy√(1-(k-1)^2/2)dk
ここから計算は(k-1)/√2=sintで置換積分すればできる。
418訂正:01/11/03 21:05
∫√(2-2x^2)dx=√2∫√(1-x^2)dx
これはx^2+y^2=1,y>=0の面積を考えて出せる
419132人目の素数さん:01/11/03 21:06
>>417
>これをk=0〜3まで積分したものが体積
0〜1+√2の間違いじゃ。スマソ。
>>416
かぶたあるよ。すまあるよ。
420πパン:01/11/03 21:07
>>395
レス遅れてすみません。
どうもありがとう!
421132人目の素数さん:01/11/03 21:11
>>417
まだまちがっとる。2πk√(1-(k-1)^2/2)じゃ。すまんでおじゃ。
422413:01/11/03 21:25
>>414
>誤爆デス?
ほんとだ。
正しくは「>>409 」だった。スマソ
423384:01/11/03 21:42
ありがとうございます!
物理化学の教科書に、こんな問題がありました。

問. i^iを計算せよ。

かなりわかりません。
どうやらオイラーの式e^iθ=cosθ+isinθを使うようなのですが・・・
よろしくお願いします。
>>424
a^b=exp(bloga)だからi^i=exp(ilogi)...(*)。まずlogiをもとめる。
それはexpz=iの解。z=x+iyとするとexpz=expx・expiy=expx・(cosy+isiny)。
これがi=cosπ/2+isinπ/2となるのは絶対値、argumentを比較してx=0,y=π/2+2nπ。
これを(*)にいれてi^i=exp(ix-y)=exp(-π/2+2nπ)。
426132人目の素数さん:01/11/03 22:07
age
427424:01/11/03 22:30
>>425
(゚Д゚)スゲー!よくわかりました。
べき乗で巨大化してしまう虚数はつくづく不思議デス。

では自習に戻ります。
ありがとうございました。
428392:01/11/03 22:37
教えてください。参考書には、導かれた仮定が
かかれてないのです。
教えてください。
429132人目の素数さん:01/11/03 22:38
数学で、質問があるのですが、二次関数y=3x^2−6x+2
(0<x<3)で、
解の公式で解くと、最大値はなし
となるのはなぜですか?上記の範囲だと、2,999・・・・が
該当するのではないのでしょうか?
これを分数で表そうと試みたのですが、矛盾が生じてしまいます。
x=2,999・・・・    ・・・@
@×10より、
10x=29,999・・・・    ・・・A
Aー@より、
9x=27
x=3
しかし、実際はxは3じゃありません。
この矛盾についても説明をお願いします。
>>429
ネタですね?
ぜんぜんおもしろくないです。
431132人目の素数さん:01/11/03 22:44
>>428
ax^2+bx+c=0の2解をα、βとする。f(x)=ax^2+bx+cとおく。
因数定理からf(x)はx-α、x-βでわりきれる。
f(x)=k(x-α)(x-β)となってkは0次、つまり定数。展開して
ax^2+bx+c=kx^2-k(α+β)x+kαβ。係数比較してa=k、b=-k(α+β)、c=kαβ。以下略。
 ○  2.99・・・=3
 ×  2.99・・・<3

なぜかわからなければ新スレ立てれ(w
433132人目の素数さん:01/11/03 22:50
lim(x→0)xlogx=?
434132人目の素数さん:01/11/03 23:00
>>433
ロピタル(l'Hospital)の定理。基本っぽいぞ
435132人目の素数さん:01/11/03 23:03
>>433
logx=tとおいたら明らかに見えない?
436396:01/11/03 23:13
>>400
>>363
ははぁ、やはりマトンな私でした。お手数おかけしました。どうもです。
437132人目の素数さん:01/11/04 00:15
有界閉区間[a,b]で単調増加な実数値関数f(x)について

F(x)=∫[a,x]f(t)dt がxの関数として[a,b]上で微分可能であるための
fの条件を求めよ。さらに、F'(x)=f(x)が全てx∈[a,b]で成り立つためのf
の条件を求めよ
>>437
連続。
連続。
439132人目の素数さん:01/11/04 00:21
>>437
なんじゃこりゃ?fが[a,b]で単調増加な実数値関数だったら無条件に
F(x)は微分可能じゃないの?F'(x)=f(x)もまんま微積分学の基本定理
(強いていうならニュートンの定理)じゃないの?
なんかのひっかけ問題か?
440132人目の素数さん:01/11/04 00:23
>>438
ああ、連続か。なる。
>>437
訂正
a<x<bで連続。
a≦x≦bで連続。
442132人目の素数さん:01/11/04 00:39
cosAcoshB=x/2
sinAsinhB=-(y/d)



(x2/cos2A)-(y2/sin2B)=d2
(x2/cosh2A)+(y2/sinh2B)=d2

に導きたいです。どうしたら?
443442:01/11/04 00:42
あ、2ってのは2乗のこと・・・
444442:01/11/04 00:44
さらにx/2→x/d
(;´Д`)(;´Д`)(;´Д`)(;´Д`)
445132人目の素数さん:01/11/04 00:46
∞って中学数学で習う?
446132人目の素数さん:01/11/04 01:01
>>442
cos2Aもcos(2A)じゃなくてcos^2(A)のほう?だったら
dcosAcoshB=x,dsinAsinhB=yから
x^2/cos^2(A)-y^2/sin^2B
=d^2cos^2(A)cosh^2(B)/cos^2(A)-d^2sin^2(A)sinh^2(B)/sin^2(B)
=d^2(cosh^2(B)-sinh^2(B))
=d^2 (∵cosh^2(t)-sinh^2(t)=1)
もいっこも同様。
447442:01/11/04 01:37
>>446
サンクス〜
これで寝れる…(w
448132人目の素数さん:01/11/04 03:28
378 :132人目の素数さん :01/11/03 01:40
ラゲールの多項式の重みとやらはどのやうに計算したらでるのですか。
バネばかりで計る。
450132人目の素数さん:01/11/04 06:39
ねむれない
ラゲールの多項式の重みとやらはどのやうに計算したらでるのですか。まじで。
零点が0.2356のときに0.521755になるやつです。
451132人目の素数さん:01/11/04 07:39
(coshA+sinhA)(cosB+isinB)=(x+h+iy)/(x-h+iy)
から、
(x-hcothA)^2+y^2=(h/sinhA)^2
x^2+(y+hcotB)^2=(h/sinB)^2
を導出する。


似たような問題だけど442じゃないよ
誰か宜しくお願いします
452132人目の素数さん:01/11/04 08:57
>>450
ラゲール多項式の重みってどんなの?
使い方くらいはわかるよね。
453パチンコ板:01/11/04 09:11
パチンコ板の連中が、ああでもないこうでもないと
悩んでいます↓。どなたかヘルプを。。。

http://piza2.2ch.net/test/read.cgi/pachi/1004782423/1
454教えてください。:01/11/04 10:12
「任意の可換環Aに対しSpecAはT3空間でない。」
の証明か、またはT3になる条件を教えてください。
>>453のスレで
http://piza2.2ch.net/test/read.cgi/pachi/1004782423/50
なんとなくワラタ

(実際にやったら理系以外ではニュー速が1位で生活系が全滅っぽいかも)
456132人目の素数さん:01/11/04 13:02
0≦X、0≦Y、2X+Y=4の時、XYの最大値・最小値を求めよ。
これを、二次方程式で、解け、って問題解けますか?
二次関数じゃないですよ。

9a^2+8a+4/4って、約分してもいいのですか?
わかりやすく説明してください。

なんで、なんですか?
2,99999・・・・・・・・・・=3
高1のDqnにもわかるように、説明して。
すれたてんのめんどう。
457132人目の素数さん:01/11/04 13:13
1つ目は,どの参考書にも,殆ど同じ問題が載っているはず.

2つ目は4/4を1にしてもいいと思います.が,分母と分子でフォントが違うことに,何かあるの?

3つ目のは,1=0.9999999999999999・・のスレと内容が同じでは?
458132人目の素数さん:01/11/04 13:14
y=-2x+4
ならば
xy=?
459132人目の素数さん:01/11/04 13:45
(x-2)^2+(y+1)^2+4の最小値と、そのときのx,yを
求めろってもんだいがわかりません。
解説みたら、x-2=0、y+1=0よか
書いてあるんですよ。その理屈がわかりません。
それに、x=999999・・・・の時のほうが
値が大きいと思うのですが、どうでしょ。
お願いします
460132人目の素数さん:01/11/04 13:48
>>459
平方が最小になるのは中身が0のときだから。
461132人目の素数さん:01/11/04 13:50
(x-2)^2≧0
だから
462132人目の素数さん:01/11/04 14:07
ガウスラゲールの重みがマジでわかりません。
零点はわかっています。n=1や5のときの求め方をかいてもらえませんか。
463132人目の素数さん:01/11/04 19:33
>>462
だからそれは何って聞いてるのに・・
ガウスの求積法で使うクリストフェル数のこと?
464教えてください:01/11/04 20:38
電卓で3乗根を求めるにはどうしたらよいでしょうか
>>464
関数電卓でやれ
466教えてください:01/11/04 20:49
>>465さん
試験問題で内部収益率を計算する問題がありまして、内部収益率=期間√(将来価値÷投資額)-1というものです。
期間が偶数なら簡単なのですが奇数だと?試験会場は関数電卓は使用禁止なので困っていました。
ただ、期間が奇数で問題を作るとは思えないのではありますが、疑問になってしまってここにスレしたわけです。
>>466
ニューd法で近似にゅ
468教えてください:01/11/04 21:01
関数機能の無い電卓では無理ということですね。どうも
469132人目の素数さん:01/11/04 21:39
ガウスラゲール求積法の重みが…
470132人目の素数さん:01/11/04 22:29
最大値が3で、x=−1のとき、y=−5、x=3
のときy=−5の二次関数をエレガントに解けますか?
471初めまして。:01/11/04 22:39
悩んでますが解けません。ぜひ,皆様のお力を...
@1/cos(x)の不定積分を求めよ。
Ap>0,q<0の時,(p-q)^2/(p^2+q^2)の最大値を求めよ。

例の検定で出た問題です。
@はt=sin(x)と置くことで,置換積分で解けましたが,なぜか,手持ちの参考書の解答と一致しません。
計算がちがっているのか,その本が間違っているのか???
正しい答えをお教え下さい。

Ap+q=a,pq=b とでも置くのかな?と思ったんですが,置いても手が着かないです。。。
最小値なら出るようなんですが。。。
予想は『max=2』!当たってますか?
>>470
エレガントかどうかはともかくx=3,x=-1における値が等しいから
軸がx=1だと気付けばy=a(x-1)^2+3とおいて以下簡単と思われ。
473 :01/11/04 22:53
>>471
多重投稿下げ
474132人目の素数さん:01/11/04 23:54
新たな質問です。どなたかお知恵を拝借。
長さl(エル)の棒が、両端をそれぞれx軸とy軸につけたまま第一象限を動くと、
棒の掃く図形は何でしょうか。て優香境界線の方程式をlで表したいのですが。
475132人目の素数さん:01/11/05 00:07
こまってます。教えてください。

「数列{a_n}は有界単調増加数列または有界単調減少数列とする。このと
き次を証明せよ。

・lim(n→∞)a_n = a のとき lim(n→∞)(a_1+a_2+…+a_n)/n = a となる。」

とりあえず、b_n = (a_1+a_2+…+a_n)/n とおいて、
{a_n}が単調増加なら{b_n}も単調増加でかつ上に有界なので、{b_n}は収束
する、ということまではわかりました。
476にんぺん:01/11/05 00:10
収束の定義を使いましょう。
477132人目の素数さん:01/11/05 00:19
ラゲール


∫e^(-x)F(x)dx=ΣwF(x)
0      

のw(重み)の求め方を…
478132人目の素数さん:01/11/05 00:28
>>474
これ有名問題だったとおもう。ほうらく線の知識があれば楽にとけるけど
初等的にとくならいろいろやりかたはあると思うけど
めんどいので(1/l倍してl=1の場合にしてしまって)
線束xcosθ+ysinθ=cosθsinθ (0≦θ≦π/2)の軌跡をもとめる。
つまりf(θ)=x/sinθ+y/cosθの最小値が1以下になるx>0,y>0の範囲
をもとめる。微分して増減表かいてtanθ=(x/y)^(1/3)のとき
つまりcosθ=x^(1/3)/√(x^(2/3)+y^(2/3)),sinθ=x^(1/3)/√(x^(2/3)+y^(2/3))
のとき最小となるのでその最小値(x^(2/3)+y(2/3))^(3/2)≦1が
もとめる軌跡。つまりx^(2/3)+y^(2/3)≦1がもとめる軌跡。
...だったとおもう。この2/3という数字は記憶にあるので
あってるとおもうんだけど。もしかしてもっとうまいやり方あるかも。
479132人目の素数さん:01/11/05 00:34
>>477
やっと書いたね(笑)
やっぱりクリストフェル数だ。
ラゲール多項式をP_n(x)、その零点をx_iとおくと、
w_iは

∫P_n(t)e^(-t)/(P’_n(x_i)(t-x_i))dt
0
で求められる。
480475:01/11/05 00:42
>>476
すみません、やっぱりわかりません・・・
イメージはできるのですが、証明するとなると
何を言ったらいいのか・・・
>>478
線束xsinθ+ycosθ=cosθsinθ (0≦θ≦π/2)の軌跡をもとめる。
のまちがいっす。スマソ。したがってつぎの行も
f(θ)=x/cosθ+y/sinθ
に訂正。それ以降はノートの走り書きまるうつししたので
あってるとおもう。自信なさげ。
482通りすがりの者:01/11/05 00:52
>>481
答えは合ってる。やり方は俺と少し違うけど。
俺はxを固定した時のyの最大値からやった。
(こうすると、cos^3θ=xの時にy=sin^3θで最大)
いずれにしても微分は出てくるが。
ところでこの曲線の名前を忘れてしもうた。
アルカノイド?あれ?
アステロイド
485132人目の素数さん:01/11/05 01:19
Q~(代数的数)Q(有理数)と書きます。[Q~:Q]=∞ってど〜やって示せばよいでしょ?
簡単なことみたいなんですケド・・・。
x^n−2はQで既約でnはいくらでも大きく取れる。
487通りすがりの者:01/11/05 02:11
>>483
サンクスw
488ブルセラアイドル:01/11/05 02:23
GTOで雅役をやったあのコ
リップスティックで星川壱成(石田)とラリッた援交娘役をやってたあのコ

昔ブルセラでお小遣い稼いでました
http://www2.ttcn.ne.jp/~yuyu/nakamuraaimi001.jpg

んで、事務所クビになったんだって(w



http://www.kt.rim.or.jp/~nor/
http://www.kt.rim.or.jp/~nor/cgi-bin/resbbs.cgi
>>488
し根。
490132人目の素数さん:01/11/05 02:44
あげ
数学屋と違ってテレビに出てる奴らは不健全だな。
492132人目の素数さん:01/11/05 03:02
原点を通り、方向比がa:b:cおよびA:B:Cである二直線のいずれにも垂直である直線の方程式を求めよ。
求める直線の方向余弦を(λ,μ,ν)とすると
Aλ+Bμ+Cν=0
aλ+bμ+cν=0
λ^2+μ^2+ν^2=1
の三つが導かれると思うんですが、答えが出ません。ワカリマセン。教えて下さい、おねがいします。
493KARL ◆gjHKPQSQ :01/11/05 03:06
素数番目の素数の列
3,11,17,31,41,59,67,83,...
からその逆数の列をつくり和をとります。
1/3+1/11+1/17+1/31+1/41+1/59+1/67+1/83+...
この級数は収束するでしょうか。
494132人目の素数さん:01/11/05 03:14
みんな休憩しようよ。このこなんてどう?
http://www22.big.or.jp/~storm7/cgi-bin3/img-box/img20011016235402.jpg
495KARL ◆gjHKPQSQ :01/11/05 03:27
>>492
前2式からλ,μをνであらわして第3式に代入すればνが得られます。
>>493
あてずっぽうですが発散するのではないかと思ってます。
496132人目の素数さん:01/11/05 04:03
ラゲ〜ルで

n=2のとき
P_n(t)=t^2-4t+2
P’_n(t)=2t-4 であるから


∫P_n(t)e^(-t)/(P’_n(x_i)(t-x_i))dt
0

に当てはめると

∫(t^2-4t+2)e^(-t)/((2x_i-4)(t-x_i))dt
0

なのでこれでmathematicaで計算すると does not converge on {0,∞}
とエラーになり計算できませぬ。
いままでもこれでなやんでおります。
どこが違っているのでしょうか?
497132人目の素数さん:01/11/05 04:05
>>492
外積が楽でしょ.

すいません質問です.
A=k[x1,…,xn],B=k[y1,…,yn],C=k[x1,…,xn,y1,…,yn]
P1をAの素イデアル、P2をBの素イデアル.
この時、P1,P2で生成されるCにおけるイデアルは素でしょうか?
お願いします
498132人目の素数さん:01/11/05 04:21
>>486
さんくすでありまする。
499132人目の素数さん:01/11/05 04:47
>>493
KARLさん、おひさ。
確か過去ログで、海外のサイトに、
log(log(log(x)))+・・・
で近似できるとあったから、発散するよん。

<近似式>
自然数なら
log(x)+・・・
素数なら
log(log(x))+・・・
素数番目の素数なら
log(log(log(x)))+・・・
・・・らしい。
500 ◆pvySbQO2 :01/11/05 05:00
>>391
 煤Q{1≦k≦n}((−1)^k/k)
=煤Q{1≦k≦n}((−1)^k・∫_[0,1](x^(k−1))dx)
=−∫_[0,1](煤Q{1≦k≦n}((−x)^(k−1)))dx
=−∫_[0,1]((1−(−x)^n)/(1+x))dx
=−∫_[0,1](dx/(1+x))
 +∫_[0,1]((−x)^n/(1+x))dx
=−log(2)+(−1)^n・∫_[0,1](x^n/(1+x))dx

 0
≦∫_[0,1](x^n/(1+x))dx
≦∫_[0,1](x^n)dx
=1/(n+1)
なので
 lim_{n−>∞}煤Q{1≦k≦n}((−1)^k/k)
=−log(2)。
501 ◆pvySbQO2 :01/11/05 06:00
>>475
c=lim(a(n)),d=lim(b(n))とすると
b(n)≦a(n)からd≦c。
e<cとなるeをとるとN≦nならばe≦a(n)となるNがある。
N≦nとすると
(N・b(N)+(n−N)e)/n≦b(n)。
N・b(N)/n+(n−N)e/n≦b(n)。
lim(N・b(N)/n+(n−N)e/n)≦lim(b(n))。
e≦d。

lim_{e<c,e−>c}e≦lim_{e<c,e−>c}d。
c≦d。

よってc=d。
502132人目の素数さん:01/11/05 06:24
(a[n]は数列)

a[n+1]=4a[n](1−a[n])

0<a[1]<1 として
a[n]の分布関数を求めよ

よろしくお願いします
503 ◆pvySbQO2 :01/11/05 07:00
>>493
n以下の素数の個数をπ(n),n番目の素数をp(n)とする。
π(n)<A・n/log(n)となるAが存在する。
n<A・p(n)/log(p(n))。
n・log(n)<A・p(n)。
p(n)・log(p(n))<A・p(p(n))。
n・(log(n))^2<A^2・p(p(n))。
1/p(p(n))<A^2/n・(log(n))^2。

∫_[2,∞](dx/x(log(x))^2)=1/log(2)
なので煤i1/p(p(n)))は収束する。
504132人目の素数さん:01/11/05 07:53
>>496
式が分かってるならなぜ先にそれを書かない。無駄手間だったじゃないか。
Mathematica持ってないからなぜエラー吐くのかは知らないけど、
少なくともこの積分は発散しない。
t^2-4t+2=(t-x_1)(t-x_2)を使って

(t^2-4t+2)/(t-x_1)=(t-x_2)

と約分(ってのも久しぶりに言う言葉だが)しておかないと
特異積分になるから計算出来ないのでは?
505お願いします:01/11/05 14:05
y=cosx
y=ax+1
が一点だけで接するとき、aの値と接点の座標を求めよ。
506132人目の素数さん:01/11/05 14:28
>>502
分布関数ってなんだ?
507質問です:01/11/05 14:32
http://www.geocities.co.jp/SiliconValley-Bay/4869/c.gif

 図の四角形ABCDと四角形ECGFは合同な平行四辺形です。また、BCGは一直線上に
あります。三角形PCQ(青部分)の面積が3cu、三角形QGF(黄部分)の面積が24
cuの時、三角形PCB(赤部分)の面積を求めてください。
508通りすがりの者:01/11/05 16:08
9
509132人目の素数さん:01/11/05 16:13
>>ラゲールの質問に答えてくれた人

ラゲールの重みがどうしてもわからなかったのでガウス型積分公式とやらをつかい解きました。

質問に答えてくれた人ありがとうございました。&あまり役に立ちませんでした。

ホントにバカなアフォ学生でスマソ。
510132人目の素数さん:01/11/05 16:29
>>508 どうやって求めました?
511492:01/11/05 16:32
どうもありがとうございました。
512答えた人:01/11/05 17:29
>>509
>&あまり役に立ちませんでした。

そういうことは思っても黙っておくもんだよ(笑)
ま、解けてよかったね。
513509に贈るコピペ:01/11/05 18:49
教えてクン養成マニュアル
明日の「教えてクン」を目指す、若き戦士達に以下の文章を捧げる。
日々精進し、パソコンヲタクどもの親切を蹂躙してやれ。

1.努力を放棄すること
  いやしくも「教えてクン」たるもの、努力をしてはならない。
 過去ログを読んだり、検索してはいけない。
 「英語は苦手なので、分かりません。」は、高く評価できる。
 辞書片手にマニュアルやReadMeを読むなど、決してしてはならない。
 他力本願と言われようと、自分で調べたり試行錯誤したりせず、
 他人の努力の結果を搾取するのが、正しい「教えてクン」である。
 また、「もう何が悪いのかサッパリ分かりません。」と言って
 ふてくされるのも有効である。「サッパリ」という単語が
 「やる気の無さ」を効果的に表現している。
 「原因を特定するには、何をすべきでしょうか?」と訊いてしまうと
 自己の積極性が現れてしまうので、「教えてクン」失格である。

2.情報を開示しないこと
  使用OSや、機器構成などの必須の情報を知らせてはならない。
 マザーボード名やBIOSのバージョンも同様だ。
 具体的なアプリ名やバージョンも隠蔽すべきだ。
 「DVD再生ソフト」のように曖昧に表記しておけばよい。
 反対に「前から欲しいと思っていた○○」とか「安売りされていた
 ○○」 等の「どうでもいい情報」は、どんどん書いてやれ。
  トラブルの場合は、状況を正確に記述してはならない。
 「なんだかうまく動きません。」とか「エラーが出ます。」等と
 具体的なことは何も書かないことが重要である。
 また、自分の試してみた事も具体的に書いてはいけない。
 考えられる組合せのマトリックスを作成し、状況を整理するなど
 もってのほかである。最悪の場合、それだけで問題が解決してしまう
 こともあるのだ。
 「いろいろやってみたけど、動きません。」が理想的だ。
514509に贈るコピペ:01/11/05 18:49
3.答える人間のことを考えないこと
「教えてクン」は、孤高の戦士である。相手のことを考えるようでは
 教えてクン失格というものだ。
 以下のような行動が、望ましい。
  初心者であることを高らかに宣言し、初心者向けの丁寧で
 分かりやすい説明を強要する。専門用語の使用を禁じておくと
 さらに効果的である。簡潔な説明を禁じられたヲタクどもは、
 同じ内容を説明するのに、何倍もの労力を強いられる。
 自分は努力せず、相手には多大な努力をさせることこそが
 「教えてクン」の真骨頂である。
  マルチポストも有効である。そのBBSを信用していないことを
 明確に示せる。「どうせ、お前らじゃ分からんだろう。」という
 意志表示として高く評価できる。もちろんマルチポストの非礼を
 あらかじめ詫びてはならない。それでは、単なる「急いでいる人」
 になってしまう。それは、教えてクンではない。
  質問のタイトルは、「教えてください。」で良い。
 タイトルを読んだだけでは「何に関する質問」か全く分からない。
 そういう努力は、答える人間にさせれば良いのだ。
 とにかく、答える人間が答えやすいように気を使って質問しては
 ならない。傲慢で不遜な態度が必須である。
 「聞きたいことがあります。」など、プロの仕事であろう。

最後に、言うまでも無いことだとは思うが、答えてくれた人達に
お礼の言葉を返すなど言語道断である。
せっかく「教えてクン」を貫いてきたのに、最後にお礼を言っている
ようでは、臥竜点睛を欠いていると言わざるを得ない。
質問だけしておいて、後はシカトが基本である。
上級テクニックとして、「そんなことはもう試しました。」とか、
「そこまで初心者じゃありません。」などと言って、回答者の
神経を逆なでしておけば完璧である。

以上のことを踏まえて質問すれば、君も立派な「教えてクン」である。
ビバ!教えてクン! 教えてクンに栄光あれ!!
515132人目の素数さん:01/11/05 20:59
曲線C:y=sin^2Xと、X軸に平行な直線L:y=sin^2Tについて、(0<T<π/2)
(1)自然数n≧2について、(n-1/2)π<X<nπにおけるCとLの交点のX座標を
Xnで表す。T≦X≦Xnなる範囲において、CとLにより囲まれた2n-1個の部分の
面積の総和Sn(t)を求めよ。
(2)tが0<T<π/2なる範囲を動くとき、Sn(T)が最小となるtの値Tnを求めよ。
(3)極限値lim n→∞ n^2(1-sin2Tn)を求めよ。
516132人目の素数さん:01/11/05 21:00
すいません、間違えました。
曲線C:y=sin^2Xと、X軸に平行な直線L:y=sin^2Tについて、(0<T<π/2)
(1)自然数n≧2について、(n-1/2)π<X<nπにおけるCとLの交点のX座標を
Xnで表す。T≦X≦Xnなる範囲において、CとLにより囲まれた2n-1個の部分の
面積の総和Sn(t)を求めよ。
(2)Tが0<T<π/2なる範囲を動くとき、Sn(T)が最小となるTの値Tnを求めよ。
(3)極限値lim n→∞ n^2(1-sin2Tn)を求めよ。
>>497
たぶん答えはNOだとおもう。RのイデアルIが素イデアル⇔R/Iが整域
でRのイデアルIとSのイデアルJに対し(R,SをR○S(○はテンソル積、以下同様)
の部分代数とみて)I,Jで生成されるイデアルKとするときR/I○S/J≡(R○S)/K
だから結局問題は“R、Sが整域⇒R○Sも整域”だけどこれはNO。
たとえばKを標数pの有限体、aをその元でp乗根がKにないものとするとき
R=K[X]/(X^p-a)、S=K[Y]/(Y^p-a)は体、よって整域だけど
R○SはX○1-1○Y(の類)は0でないのにp乗すると0になるので整域でない。
たしかK代数Aが“任意のK上の整域BにたいしA○Bも整域”をみたすものを
分離的代数とよぶんだったとおもう。今てもとにある教科書にはみあたらないけど
たしかAlgebra I,Carl Faithにそんな事がのってたような。のってなかったらゴメソ。
>>513-514
「教えてクン」を「ちむ信」に読みかえると・・・
519Lee:01/11/05 22:30
第一問
a=50%×{1−E×(−k×A)乗}
a=30% A=10万
の場合Kは?
520132人目の素数さん:01/11/05 23:24
スイマセン、ここにいる方には簡単で教える気が失せるかもしれないけど
忘れちゃったんで教えて下さい。
327分の1の確率で起こる事象を71回の試行で3回だけ引き当てる
計算式を教えて下さい。
>>520
C[71,3](1/327)^3(326/327)^68
522132人目の素数さん:01/11/05 23:42
>>521
ありがとうです!
523132人目の素数さん:01/11/06 01:24
>517
ありがとうございます.
Kが代数閉体でもダメですか?
でも、その言い換えができるなら、考えれそうです.
調べてみます.
524132人目の俺:01/11/06 06:19
>>520
覚えるようなものなのか?
525132人目の素数さん:01/11/06 08:32
1.投資金額を13分割する。
2.まずは投資レースにその1/13を賭ける。
3.当たったら当たった分を投資金額合わせて1.に戻る。
4.外れたらこんどは3/13を次の投資レースに賭ける。
5.当たったら当たった分を投資金額合わせて1.に戻る。
6.外れたらこんどは9/13を次の投資レースに賭ける。
7.当たったら当たった分を投資金額合わせて1.に戻る。

的中率を60%、倍率を1.5として
13000円からこのやり方を行うと、どのくらいの確率で
10万、50万、100万超えるのでしょうか?
また4万から40分割してやって、上記の金額に達する確率はどのくらいでしょうか?
>525
複コロか?
527132人目の素数さん:01/11/06 12:51
>>525
限り無く0%に近い。ワラ
そうゆうのおもしろいね。正確な答え知りたい。
528132人目の俺:01/11/06 15:02
10万に達する確率は
(936/1000)^[log_(27/26)(100/13)+1]
50万に達する確率は
(936/1000)^[log_(27/26)(500/13)+1]
100万に達する確率は
(936/1000)^[log_(27/26)(1000/13)+1]

[]はガウス記号。
誰か計算してくれ。
529いなかもん:01/11/06 15:39
 最近,圏論をかじったんですけど,任意のAbel圏が,ある環RのR加群のなす
圏に埋め込む事ができるっていう定理があるそうなんですけど,手に入りやすい文献
(日本語の購入可能な本だといいなあ。)で,その証明がのってる物ってなにがある
でしょうか。ちなみにここは田舎なので,数学の本がそろってる図書館とか近くにな
いんで,できればネットで購入できる本だといいんですけど。
530132人目の素数さん:01/11/06 16:19
>>525
モンテカルロでやったら

10万 2.6%くらい
50万 0.2%くらい
100万 0.0%くらい

40分割は何がやりたいのか理解できない
531384:01/11/06 18:01
∫[0,1]{1+√(2-2x^2)}^2dxを一番手っ取り早く積分する方法教えてください
ちなみに答えは 8/3+√2/2+√2π/2 でいいんですよね?
532132人目の素数さん:01/11/06 18:20
>>530
40分割はいみないですか?
533525:01/11/06 18:42
では、いくらまでコロガシて、また13000円にリセットするのが最適ですか?
>>531
>418 名前:訂正 投稿日:01/11/03 21:05
>∫√(2-2x^2)dx=√2∫√(1-x^2)dx
>これはx^2+y^2=1,y>=0の面積を考えて出せる

正確(?)にはx^2+y^2=<1,y>=0の面積
グラフを書けば区間[0,1]や[1/√2,1]の面積は自明でしょう
535474:01/11/06 21:00
>>478>>481>>482>>483>>484
いろいろと教えていただいてありがとうございました。
少しほうらく線に興味を持ちネットや本を調べているのですが
なかなかいいのに出会えません。
こんなのはどんなほうらく線が出来るのでしょう?
「点Aはx軸上を(0,0)から(1,0)へ、点BはY時苦情を(0,1)から(0,0)へ
等速で移動するときの線分AB」
536132人目の素数さん:01/11/06 21:00
4x^2+7xy-2y^2-5x+8y+がx,yの一次式の積に分解されるように
定数kの値を定めてください。
537132人目の素数さん:01/11/06 21:29
Γ分布の乱数の生成方法のアルゴリズムご存知な方おられませんか?
簡単な生成方法を教えていただけるとあり難いです。
>>529
たしかAlgebraI,Carl Faithでみた。日本語の本ではみたことないな。
証明はその圏の上で定義されるアーベル関手全体の圏をあるトーション部分圏
でわるという操作をやるんだったとおもう。
たぶんあきらめてどっかからか英語の本を入手するしかないと思われ。
>>536
kが消えてます・・・

(ax+by+c)(dx+ey+f)=(4x^2+7xy-2y^2-5x+8y+cf)が
x,yに関して恒等式となるようにa,b,c,d,e,fを定めればk=cf
・・・だるすぎ。だめな見本。
540132人目の素数さん:01/11/06 22:10
>>532
今更40分割の意味が分かったよ。
まぁ価値のある情報が得られそうにもないので、
特にやろうとは思わないけど。

結果なんぞ似たようなもんだ。

>>533
「コロガス」という概念の説明がないので分からない。
基本的に賭け事はやればやれほど負ける。

最適なのは「賭け事に参加しない」ことだ。
わかったらさっさとギャンブルなんて止めろ
>>539
最初にa=1とでも決めてしまえばそんなにだるくない
542132人目の素数さん:01/11/06 22:33
>>536
4x^2+7xy-2y^2-5x+8y+k = 4x^2+(7y-5)x-2y^2+8y+k
の判別式が(ay+b)^2となるようにkを決める
543質問です:01/11/06 22:35
無理数全体の集合をKとする。#Kと#Rについて
#K=#R、#K>#R、#K<#Rか決定せよ。
544お願いします:01/11/06 22:42
0≦z/2+1/z≦2をみたす複素数z(z≠0)を表す複素数平面上の点の集合をもとめよ
545132人目の素数さん:01/11/06 22:44
>540
止められません、これ(馬)ばっかりは。
546質問ですお願いします:01/11/06 22:49
zは│z-1-i│≦1/√2をみたす複素数、tは0≦t≦8をみたす実数とする。

さらにw=-itz(zはzバーにしてください)とする。zとtが上記の二つの条件をみたしながら変化する時
(1)複素数平面上においてwの存在範囲を求めよ
(2)wの偏角のとりうる値の範囲を求めよ。
547132人目の素数さん:01/11/06 22:54
#K=#R です。
有理数の全体=Q={q0,q1,q2,・・・・}とし,
Kから可算集合A={a0,a1,a2,・・・・}をぬきとる。
f:K→Rを
x∈K-A のとき f(x)=x
また、a_n∈A にたいして
f(a_2m)=a_n, f(a_2m+1)=q_m としてやれば
f は 1-1 onto
548質問ですお願いします:01/11/06 22:55
複素数zとwの間にはw=1/(zーα)の関係がある。
ただし、αは複素数である。複素数平面上において、実軸上(ただし、点αが実軸上にあるときには、
点αを除く)を点zが動く時、点wはある一つの直線上を動く
(1)αのみたす条件を求めよ。
(2)虚軸上(ただし、点αが虚軸上にある時には、点αを除く)を点zが動く時、点wの描く図形を求めよ
549質問ですお願いします:01/11/06 22:59
xy平面上に2点A(2,-4)B(4,0)と楕円x^2/9+y^2/4=1がある。この楕円上に点Pをとって三角形PABを作る時、その面積の最小値を求めよ
550132人目の素数さん:01/11/06 23:01
可分距離群には同値な左(右)不変な距離が存在する。
これってどうやって証明するのか知ってり人おりませんか?
551質問ですお願いします:01/11/06 23:01
3辺の長さがいずれも70より小さい整数でかつ等差数列になっている三角形は何個あるか。ただし、合同な三角形は区別しないものとする。
552132人目の素数さん:01/11/06 23:22
♯{Gの部分群}<∞ならば、Gは有限群である とういうことを示せます?
553132人目の素数さん:01/11/06 23:56
>>552
♯{<g>|g∈G}<∞ かつ sup{♯<g>|g∈G}<∞
よりすぐです。
554だれかおしえて:01/11/07 00:02
 最近,圏論をかじったんですけど,任意のAbel圏が,ある環RのR加群のなす
圏に埋め込む事ができるっていう定理があるそうなんですけど,手に入りやすい文献
(日本語の購入可能な本だといいなあ。)で,その証明がのってる物ってなにがある
でしょうか。ちなみにここは田舎なので,数学の本がそろってる図書館とか近くにな
いんで,できればネットで購入できる本だといいんですけど。
>>553
全然すぐじゃない気が。
っていうか、本当に正しいか?
556543:01/11/07 00:04
>>547
ありがとうございました!
557中3です:01/11/07 00:06
証明のやりかたを教えて下さいっ!!!!
558小3です:01/11/07 00:07
あきらめろっ!!!!
559中3です:01/11/07 00:10
いや、ほんとに教えて下さい。困ってるんです・・・
560132人目の素数さん:01/11/07 00:13
>>553でいう部分群は真部分群のこと?
そうでないなら問題以前だYO!
5613歳:01/11/07 00:13
>>559
仮定と,万人が納得する論法を展開すれば,
必ず証明できるが・・・
何に困っているんですか?
562中3です:01/11/07 00:15
何の定理を使ってやっていったらいいのかわからないんです。
563132人目の素数さん:01/11/07 00:17
教えてくんマニュアルの見本みたい
>>560
どうして?
GはGの部分群だから
566560:01/11/07 00:37
勘違いしていた、部分群の個数が有限ね
ウツダシノウ
>>553,555
Gは<g> (g∈G)のunionなのですぐです。
>>557
問題を教えて下さいっ!!!!
569132人目の素数さん:01/11/07 01:01
ルベーグ積分に関してです。
(1)∫[a,b]fdμ と lim[ε→∞]∫[a,b-ε]fdμ
(2)∫[0,∞)fdμ と lim[c→∞]∫[0,c]fdμ
「これらを対比しなさい」なんてお題を出されたわけでして。
数冊かの本を見ると、「f(x)≧0と限らない時は、広義リーマン積分可能
であってもルベーグ積分不可能なことがある」「ルベーグ積分の枠に広義
積分が含まれることはない」なんてことが書いてありました。
(2)の反例として”f(x)=sinx/x [0,∞)”なんてのがありましたが、
(1)はどうなのでしょうか?
私の見た本にはあまり詳しく語られていませんので、他に何かご存知
でしたら、是非お教え願います。
570132人目の素数さん:01/11/07 01:06
↑あっ!ゴメンなさい、(1)はε→+0です。↑
>>569
x−>(x−a)/(b−x)という対応でも考えれば
[a,b)と[0,∞)は同じようなもの。
572!:01/11/07 01:52
X^2/(X^2+4) を積分するにあたっての秘密を教えてください。
573質問です:01/11/07 01:53
方程式|3x-20|=|x-10|+|x+1|を解け。の答えの出し方を教えて下さい。
絶対値の方程式です。「●≦x のとき」 というのを使う方法です。
574中3です:01/11/07 01:57
>568サマ
えっと、円の中心から弦にひいた垂線は弦を2等分する

って問題なんです
>>573
絶対値の中が0以上の時と0以下の時に場合分け。この場合は4通り。
>>574
2つの三角形が合同だからですっ!!!
577132人目の素数さん:01/11/07 02:01
BASICで
1! とか 6! とか書いてあったら
何を意味するのですか?
Webで検索したけれど、階乗ではない、って事以外分かりません。
宜しくお願いします。
578質問レス!:01/11/07 02:05
マイナスにマイナスをかけると
なんでプラスになるの〜?
579573:01/11/07 02:05
>>575
すみません。もう少し具体的にお願いします。
580!:01/11/07 02:07
単精度浮動小数点数型 (Single) の変数は、IEEE 32 ビット (4 バイト) の浮動小数点数の変数です。
負の値は -3.402823E38 〜 -1.401298E-45、正の値は 1.401298E-45 〜 3.402823E38 の範囲の値をとります。
単精度浮動小数点数型の型宣言文字はエクスクラメーションマーク (!) です。
581132人目の素数さん:01/11/07 02:08
>>577
単精度浮動小数点数かな?
582昔、塾の先生:01/11/07 02:08
絶対値が切り替わるxが3つありますね。
x=20/3,10,-1
小さい順に書き直すと
x=-1,20/3,10
よって
x≦-1
-1≦x≦20/3
20/3≦x≦10
10≦x
の4通り
583581:01/11/07 02:09
かぶった
584572:01/11/07 02:10
すいませんが、
X^2/(X^2+4) の積分のやりかた分かりませんか?
585132人目の素数さん:01/11/07 02:13
>580-581
ありがとうございました。
586578:01/11/07 02:18
文体がふざけてたけど
質問は結構本気です。
納得できる説明がほしいのです。
−×−=+
の理由を教えてください
588132人目の素数さん:01/11/07 02:42
>>571
ははぁ、なるほど・・・。要するに「ルベーグ積分というやつは、広義積分に
関しては苦手である」って感じでしょうか?変な表現ですが・・・。
とにかく、ありがとうございました。
589昔、塾の先生:01/11/07 02:45
>584
X=2tanθと置く。
分母=4tan^2θ+4=1/(4cos^2θ)
分子=4tan^2θ
分数=sin^2θ
dX=2/(cos^2θ)dθ
よって被積分1次形式は
X^2/(X^2+4)dX
=2tan^2θdθ
=2(1/cos^2θ-1)dθ
積分して
2tanθ-2θ+C
Xに直して
X-2Arctan(X/2)+C
590KARL ◆gjHKPQSQ :01/11/07 02:49
>>503
>n以下の素数の個数をπ(n),n番目の素数をp(n)とする。
>π(n)<A・n/log(n)となるAが存在する。 (1)
>n<A・p(n)/log(p(n))。 (2)
>n・log(n)<A・p(n)。 (3)
>p(n)・log(p(n))<A・p(p(n))。 (4)
>n・(log(n))^2<A^2・p(p(n))。 (5)
>1/p(p(n))<A^2/n・(log(n))^2。 (6)

>∫_[2,∞](dx/x(log(x))^2)=1/log(2) (7)
なので煤i1/p(p(n)))は収束する。

(1)はなぜですか。いわゆる素数定理から言えることなのでしょうか。
(2)は(1)のnのかわりにp(n)を入れたものという意味でしょうか。
(3)は(2)から(3)が導かれるという意味でしょうか。だとすると、左辺は
n・log(p(n))になるのではないですか?
とすると(4)はp(n)・log(p(p(n)))<A・p(p(n)
となります。。(nをp(n)にさしかえたわけですね)
とすると、だいぶ事情が違ってきます
(4)〜(6)とくに(5)は何をやっているのか分らない。解説願います。
(7)の積分も分りません。よろしく。要するにわからない。間違いではないのか。
591132人目の素数さん:01/11/07 05:15
放物線 y = ax^2 + bx + c 上の点( t , at^2 + bt + c) における接線の傾きを求めよ。

この問題を教えてください。
592132人目の素数さん:01/11/07 05:17
>>591
2at
593132人目の素数さん:01/11/07 05:18
どうやったらそんなに早く求められるのですか?

それと式を教えていただけるでしょうか?
594132人目の素数さん:01/11/07 05:20
> どうやったらそんなに早く求められるのですか?
warata
595592:01/11/07 05:21
いやだ。
もっとマジメに勉強しろ。
596132人目の素数さん:01/11/07 05:23
もったいぶらないで教えてくださいよ
597プ:01/11/07 05:29
>>592 しかも間違っているし。
598132人目の素数さん:01/11/07 05:45
だみだこりゃ!
次いってみよう!
599132人目の素数さん:01/11/07 05:52
2at+b だね
600お願いします:01/11/07 07:18
544,546,548,549,551の解答をお願いします
601だ、だれか...:01/11/07 07:50
>>554.....だ、だれか...
>>554
そのくらい自分で考えてください......
603132人目の素数さん:01/11/07 10:12
550だれか教えてくださいー
604いなかもん:01/11/07 10:22
>>602そんなに簡単?お願い!だったら教えて
そもそも圏論の専門書自体すごく少ないよね。
洋書で買うしかないのかな。結局マクレーンとか言う人の
本買うしかないのかな。
605132人目の素数さん:01/11/07 19:07
>n個の箱とn個の玉があります。それぞれの箱と玉には1、2、…、n
>と番号がかかれています。n個の箱に1個づつ玉を入れるとき、箱と玉の番号が
>一致せず全て異なっているような入れ方の総数をA(n)と記することにしましょう。
>そのときA(n+1)とA(n)のあいだに成り立つ関係式はどうなるでしょう。

(かいとう)

A(1)=0
A(2)=1
A(3)=2

A(4)=9
n=4で場合わけをします(A(2)とA(3)の関係を見てみる)
まず考える玉の番号の組みを(a,b)、[入れる玉の番号、入る箱の番号]
とnotationをとる。
(1,2)のみを考えるとき
<1> [1,2]
<2> [1,2]以外
の2つの場合で言い尽くしていることになる。

<1>[1,2]
このとき(3,4)は[3,4]。この場合はA(2)と同じ。
同様に、[1,3],[1,4]の場合もA(2)と同じ。
よって、A(2)*3

<2>[1,2]以外
このとき(2,3,4) の場合を考えると 
ex)玉1が箱2へ入って、玉2が箱1へは入らない状態
この場合はA(3)と同じ。同様に、[1,3]以外,[1,4]以外の場合もA(3)と同じ。
よってA(3)*3=6

<1>+<2>=A(2)*3+A(3)*3=9となる。

以下の議論で一般化してみる。
n+1個まで一般化してみる。
<1>[1,2]
残りは(3,…,n+1) でA(n-1)の場合と同じ。
同様に、[1,3],…,[1,n+1]も同じ。
よってA(n-1)*n.

<2>[1,2]以外
このとき(2,3,…,n+1)のときを考えて、これはA(n)と同じ。
同様に、[1,3]以外,…,[1,n+1]以外の場合もA(n)と同じ。
よってA(n)*n

<1>+<2>= A(n-1)*n+ A(n)*n=A(n+1)となる。

ここからは、単なる式変形で等比級数型の漸化式へもっていける。
A(n+1)=n*A(n)+n*A(n-1)
よって
A(n+2)-(n+2)*A(n+1)=-{A(n+1)-(n+1)*A(n)}
A(2)-2A(1)=1より
A(n+1)-(n+1)*A(n)=(-1)^(n-1)
よって求めたい関係式は
A(n+1)=(n+1)*A(n)+(-1)^(n-1)
となる。
606132人目の素数さん:01/11/07 19:13
//////////////////
 ̄ ̄ ̄ ̄ ̄ ̄ ̄ ̄| ̄ ̄ ̄ ̄ ̄ ̄ ̄ ̄ ̄↑ ̄
             <             
             >            
             <   k:バネ係数   
             >                L
             <             
             >          
                |           
                |             ↓
          / ̄ヽ   g         ̄
           | m  | 
          \_ ノ  ↓

問1.上の物体の運動方程式をもとめよ
607132人目の素数さん:01/11/07 19:14

            / \   /     
          / m  \/
          \    /      g
           \ /        ↓ 
            /
_______/)θ _____
///////////////

問2.斜面の物体が動かない場合の摩擦力を求めよ
608蚊系:01/11/07 20:15
606の戸籍上のアントニオ猪木です.
物板にいけ,この野郎!
609132人目の素数さん:01/11/07 20:19
>>606
もとのばねの長さは?
610132人目の素数さん:01/11/07 20:23
>>606
自然長でつり合うばねなんてありません。
それとも加速度gで落下してるんですか?
611132人目の素数さん:01/11/07 20:29
数列 0、1、0、−1、0、1、0、−1、.....
の一般項を求めよ。
>>611
a_n=sin(πn/2) n=0,1,2,3....
613132人目の素数さん:01/11/07 20:41
すいません、確率論争に決着つけてください。
文系の住人ばっかなので、なにがなんだか分からなくなってます。
http://news.2ch.net/test/read.cgi/newsplus/1005118614/
614132人目の素数さん:01/11/07 20:41
二次不等式で、a>0のときの、
解しかかいてないんだけど、
a<0のときは解全般の答えはなにになるの?
615132人目の素数さん:01/11/07 20:46
>>612
三角関数は使わずにという条件ではどうでしょう?
616制約が少ないので:01/11/07 20:51
これで全部の解を示したことになる。
a<=8のとき
a1=0,a2=1,a3=0,a4=-1
a5=0,a6=1,a7=0,a8=-1
a>=9のとき
an=任意
617132人目の素数さん:01/11/07 20:56
>>616
4項ずつ循環する数列をひとつの式で表してください。
>>616
ん?
619613:01/11/07 20:58
あっすいません、確率論争は
http://news.2ch.net/test/read.cgi/newsplus/1005118614/
の25から始まってます。暇があったら結論びしっとおながいします。
620132人目の素数さん:01/11/07 21:52
何でマイナスとマイナスをかけるとプラスになるんですか?
621質問です:01/11/07 22:01
log10ってなに?
622132人目の素数さん:01/11/07 23:00
真数をインフォシークで調べたら、
http://jiten.www.infoseek.co.jp/Kokugo?pg=result_k.html&col=KO&sm=1&enc=SJIS&svx=980100&brand=0000&vers=606&qt=%90%5E%90%94
「真数は常に正である。」

・・・・ほんとかなぁ。
>>611
(i^n+(-i)^n)/2
624613:01/11/07 23:42
数学板から来たひと、どうもありがとうございました。
>>623
-(i^n+(-i)^n)/2カモナ
>>613関連

どうでもいいけど、「数学板から来ました」とか言って
「確立」とか書くのはやめて欲しい…
>>312
これ分かる人いますか。
628132人目の素数さん ◆EvsOYBFM :01/11/08 09:14
以下の問題を詳しく教えてください。

n次対象群S(n)の交代群をA(n)として、それぞれの自己同型群Aut(S(n)),Aut(A(n))を求めよ。また初代
征夷大将軍は誰か。
629abc:01/11/08 17:03
バーローの求積法について教えてください。  
630.:01/11/08 17:11
バーローそんなん知るかっ!
631132人目の素数さん:01/11/08 17:14
>>630
思いついても恥ずかしくて誰も言えないことをよく言った!
632132人目の素数さん:01/11/08 17:43
バーロー岬は寒いぞ。
633132人目の素数さん:01/11/08 17:45
上田バーロー
634132人目の素数さん:01/11/08 18:22
逆行列について
|sinθcosφ sinθsinφ cosθ |
|cosθcosφ cosθsinφ -sinθ |
|-sinφ cosφ 0 |
の逆行列を教えてくださいお願いします。
答えだけでもOKです。
635132人目の素数さん:01/11/08 18:38
|sinθcosφ  sinθsinφ   cosθ |
|cosθcosφ  cosθsinφ  -sinθ |
| -sinφ     cosφ       0  |
636質問death:01/11/08 19:35
クイズミリオネアで1000マソ獲得できる確率を教えて下さい。
637132人目の素数さん:01/11/08 19:40
15問だっけ?
だったら1/536870912だな。
まあ当てずっぽうで行った場合の話だ。
638132人目の素数さん:01/11/08 19:43
longstaff-schwartzの社債プライシングの計算式をエクセルかmathematica
で作ってくれませんか??
639教師の2チャンネラー:01/11/08 20:02
数学板の頭脳集団に問う。

昆虫は何であんなに足が速いんだ?
人間が昆虫ぐらい早く足を動かせたら、電車よりも速く走れるのに。
なぜ、人間には無理なんだ?

あくまでも、数学の問題です。
640三角関数:01/11/08 20:06
y=2Sin(2X+α)+7の最大値・最小値    0°≦x≦90°
ただし0°<α<90° Sinα=5/13 Cosα=12/13 
解いてください

最小値の答えが2なんですけど、どうして?
641132人目の素数さん:01/11/08 20:05
>>639
1+1=2を証明したら教えてあげよう
642教師の2チャンネラー:01/11/08 20:10
>>641
それは残念。自分の中では1+1=10です。
643教師の2チャンネラー:01/11/08 20:13
数学板の頭脳集団に問う。パート2

サッカーボールの正五角形と正六角形の数はいくつずつだ?
中学生にもわかる方法で説明して下さい。
644132人目の素数さん:01/11/08 20:13
アインシュタインクラスの天才なら高1のときに
どんな問題、解いてますか??
645132人目の素数さん:01/11/08 20:17
640の問題誰か解いて下さい
>>640
問題か解答がおかしい
y=13sin(2x+α)+7なら最小値2
647132人目の素数さん:01/11/08 20:19
は?
648132人目の素数さん:01/11/08 20:22
どうして2?
649132人目の素数さん:01/11/08 20:25
誰か解けませんかぁ?
650教師の2チャンネラー:01/11/08 20:25
>>640
646が正解

それよりもだれか639と643を答えろ!
651132人目の素数さん:01/11/08 20:25
おおおおおおーい
652132人目の素数さん:01/11/08 20:25
>>649
おまえは640か?
653工房:01/11/08 20:26
>>650
うぜ
654132人目の素数さん:01/11/08 20:27
だから何で2なの?
655教師の2チャンネラー:01/11/08 20:27
>>653
お前程じゃない。
>>643
それぞれ正二十面体の頂点と面の個数
657教師の2チャンネラー:01/11/08 20:29
>>656
別解答望む。教育板で答えられる人がいなかった。
>>628
坂上田村麻呂
659三角関数:01/11/08 20:30
640
解いて
660三角関数:01/11/08 20:31
だれかぁぁぁぁ
661教師の2チャンネラー:01/11/08 20:31
>>659
問題が間違っている。
662三角関数:01/11/08 20:31
どうして最小値2が出るのでしょう?
663132人目の素数さん:01/11/08 20:31
>>660
>>646をみたか?
664132人目の素数さん:01/11/08 20:32
>>662
ていうか問題が間違っていたのかどうかをはっきりしろ
>>640
sinθ=-2.5を解けば納得してくれる?
666132人目の素数さん:01/11/08 20:42
U⊂F^n が部分空間で、x⊂F^n とする。
Fに係数をもつ連立1次方程式で、方程式の数はn個,
変数の数もn個で,解集合がx+Uであるものが
必ず存在することを示せ。
667132人目の素数さん:01/11/08 20:57
質問です。

D={(x,y)|x^2+y^2≦x,0≦y}として2重積分
∬_[D]√(1-x^2-y^2)dxdy
の値を求めよ。

どなたか教えてください。よろしくお願いします。
668132人目の素数さん:01/11/08 21:03
>666
Uの方程式書いてみたら?
n個に増やしたらいいだけ.
>>554
まえに答えたのになにげに無視されてるよ。
そもそもステートメントが不正確。正確には

 定理 任意の小さいアーベル圏Aに対し環RとFully Faithfull、exact
 なうめこみF:C→ModRがとれる。

でしょ?証明の概略は

 補題 Bがアーベル圏、Sがちいさいfull subcategoryでEがBの
 injective objectで任意のSのobjectにたいしAの完全列0→X→E'→E''
 をE',E''がEの有限直和と同型であるようにとれるときR=EndEとおき
 表現関手F=B(-,E):S→ModRはfully faithfull exact。

を証明しておいてさらに

 補題 任意のちいさい圏Aに対しそれを上のようなB,S,Eがとれて
 AとSが同型になるようにとれる。

をしめす。前の方でレスしたけど見逃したのか?無視したの?
といっても日本語で証明ののってる文献はみたことないけど。どっちみち。
670( ´∀`):01/11/08 23:41
今日大学の授業で初めて「∀」の記号が出てすごくうれしくなりました。
でも、記号の意味がわかりません。先生も何にも言わなかったので。
確か文字の左上につけてました。
意味わかりますか?
>>670
ワラタ
672_:01/11/08 23:51

>>670
チミの大学は11月から前期が始まるのか?
673( ´∀`):01/11/08 23:51
・・・そんな基本的なことだったのか。
授業は全部でてるはずなのに・・・。おかしいなあ。
\forall
675( ´∀`):01/11/09 00:01
あと「∃」とかも説明無しに使ってきます。
うーん・・・・。
676_:01/11/09 00:10
>>675
おめーの大学には図書館がないのかよ?
677( ´∀`):01/11/09 00:11
>>676
蔵書数が多すぎてどれ見ればいいかわかんないです。
ていうかそんな言い方ないでしょうに。
>ていうかそんな言い方ないでしょうに
確かに。
>>676は優しすぎ。
\exists
680132人目の素数さん:01/11/09 00:17
>( ´∀`)さん
大学名を教えて頂けないでしょうか。
681( ´∀`):01/11/09 00:17
>>680
駅弁大学です。さすがに大学名は・・・。
682_:01/11/09 00:18
683674=679:01/11/09 00:21
せっかく教えてやってんのに無視かよ!>( ´∀`)
684( ´∀`):01/11/09 00:21
>>682
ありがとう。
って高校で習うものなの???

任意の・・・・?
685( ´∀`):01/11/09 00:23
>>674
あ、そういう意味だったんですか。
はい。すみませんー。
まあ、駅弁じゃしょうがないな
>\forall
>\exists
これ何。
TeX
>>683
ほんとに知らない人が見ると、全く無関係なもにに見えても仕方ないと思われ
せめて

>>670
\forall

とか書いてあげなよ。
690_:01/11/09 00:55

>>677
教科書を探すのでしたら、
集合論とかの最初のほうに載ってたよな気がします。
691いなかもん:01/11/09 09:07
>>669
>をしめす。前の方でレスしたけど見逃したのか?無視したの?

おお!!!なんということでしょう。見逃してしまったのです。
ぜっっっっっっっっっったいっっっっっ無視なんかではありません。
感謝!!感謝!!感謝!!
とにかく私の周りにはこんな話できる人だーーーーーーーーーーーれも
いないもんですから,絶っっっっ対無視なんかではありません。

>そもそもステートメントが不正確。正確には

そうですそうです。ただ埋め込まれただけではあんまり役に立たないですよね。
おそらく「Abel圏での完全性とか,射の存在とかを,R加群のだと思って証明
してもよい」というような応用の仕方するんでしょうからね。
 特に,抽象的なAbel圏だと対象の元(要素)見たいなのが取れなくて不便
しますよね。ただ,岩波の基礎数学の「ホモロジー代数」ではmemberとかいう
概念が出てたのが感心しました。私は任意のAbel圏でのsnake Lemmaはmember,
comemberの概念で証明しました。いやあノートにまとめるのが大変でした。
これが,R加群の圏だと思うと,下手するとノート1ページに要領のいい図式
を書けば証明が納得できてしまうんですよねえ。

>といっても日本語で証明ののってる文献はみたことないけど。どっちみち。
実はMac LaneのCategories for the Working Mathematicianを丸善で注文して
しまいました。もちろん所望の定理の証明がのってるかどうかはわかりません。
賭けです。しかも丸善だと
値段は一万円を越す。さらに送料も取られる。しかも1ヶ月かかるかもしれない。
それが,注文した後にamazonで調べてみたら
値段は7千円ちょっと,しかも送料無料,しかも2週間で届く。
大大大ショッッッッッッッッッッッッッッッッック!!!
これも>>538を見逃した天罰でしょうか。(;_;)
692いなかもん:01/11/09 09:10
インターネット時代のありがたみ。しみじみ。(−_−)
693132人目の素数さん:01/11/09 11:15
>>644 アインシュタインは、16才の時に光に乗って移動したら光はどう見えるか?
という疑問を抱いてから、思考実験を10年間暖めていたらしいですけど、
数学の方は相当できが悪かったみたいですよ。
694132人目の素数さん:01/11/09 13:20
>>691
 アーベル圏の便利なところは、矢印の向きを全部入れ替えた図式に
関する定理が「双対性により明らか」といえるところですね。member
を使うと、あたかもR加群で証明を実行してるみたいな感覚で証明で
きるので、「元」という概念が無いもどかしさを救ってくれます。
 それでも一般のアーベル圏で2重複体の定理、例えば被覆のコホモ
ロジーを用いたド・ラームの定理の証明みたいなもんをアーベル圏で
も証明するにはどうするのでしょうか?R加群で証明しておいてから
埋め込み定理を使ったほうがよっぽど楽なんでしょうか?この定理っ
て、台に制限の無い微分形式に対するドラーム・コホモロジーに関す
る証明と、台がコンパクトな微分形式の場合のドラーム・コホモロジ
ーが丁度矢印を反対向きにした形になってるので、R加群だと両方証
明しなくちゃならないわけですけど、アーベル圏での証明がもし楽に
できるなら、一方だけ証明しておけば、他方は「双対性により明らか」
とできて楽なんですが。
695132人目の素数さん:01/11/09 13:36
すいません。
「二人の兄弟がいます。兄と弟の誕生日が同じ確率は?」
なんでしょうか?

私の考えでは、
二人の誕生日がどれかの日で同じ確率は(1/365)^2で、
それが365通りの日にちがあるから、
答えは(1/365)だと思うのですが。
>695
兄弟の年齢差にもよるからそんな筈はない
血のつながりのある兄弟に限定しても
双子なら同日と日をまたいでの出産の確率が必要だし
697132人目の素数さん:01/11/09 13:43
同じ年齢に達していて誕生日が5ヶ月差だったらどうしよう・・・
698695:01/11/09 13:44
>>696
閏年は無し。満月等の外性的変数は捨象してください。
日をまたいだとしても、誕生日はどっちかになるはずですから、
それで計算をお願いします。
699いなかもん:01/11/09 13:50
>>694
どわどわどわっっっっっ!!
それ番号の振り先を間違ってる。私は”これから”こほもろじーとかをやろうと
してるんですから。でもたぶん>>538>>669さん(あるいは他の方々)もここ見
てるから,そっちからコメントあるといいですね。
700test:01/11/09 13:51
test
701計算不能:01/11/09 14:21
>>695 一人産んでから、十月十日は子供が生まれない(だけど未熟児で生まれちゃうことも)
とか、授乳期間中は妊娠しにくい。。。とかそういう外的要因は無視しきれないと思う。。
小造りの意思がどうかという問題もあるし。。。
毎日ヤリまくれば精子が薄まって妊娠しにくくなるし。。。
妊娠可能なのは、おおむね28日周期のうちの3日間だけだし。。。。
702695:01/11/09 14:24
>>701
外的要因は全て捨象してください。
703計算不能:01/11/09 14:27
>>702 いやだからせめて、一人産んでから、何日後から出産可能かを指定してもらわないと。。。
704695:01/11/09 14:32
>>703
出会った兄弟の誕生日の日付が同じ確率です。
兄が生まれてから、ちょうど一年後から、
弟が出生可能としてください。
705132人目の素数さん:01/11/09 14:37
>>704
なんで兄弟でないといけないの?
>>704 ちょうど一年後というのは、一年後の同じ日に産まれる可能性も含むのか?
707数学科指導法I:01/11/09 15:54
バーローの求積法についてのいい本、誰か教えていただけませんでしょうか?
 理学部数学科2年
 
708数学科指導法I:01/11/09 15:56
フェルマーの求接法についての本も紹介お願いします。
709695:01/11/09 16:52
>>706
含みます。
710へたれ:01/11/09 18:16
4□4□4□4=10
□に+、−、×、÷を入れて、=10にできるのでしょうか?
( )カッコはどこに入れてもOKです。
例えば、
4÷4+4−4=1
4+(4−4)×4=4
{(4×4)+4}÷4=5
こんなカンジです。
711614:01/11/09 19:51
誰か、教えて。
>>711
お化けのほうがよさげ
713132人目の素数さん:01/11/09 20:29
>>710
ルート使えば出きるけどな。
714132人目の素数さん:01/11/09 20:56
すみませんが、アドバイスおねがいします。
e^(j*a*sinθ)=cos(a*sinθ) って成り立ちますか?
Eulerの公式にしたがったら
e^(j*a*sinθ)=cos(a*sinθ)+jsin(a*sinθ)となるんですが・・・
(j:虚数単位 a:定数)
厨房な質問ですみませんがお願いします。
715蚊系:01/11/09 21:29
2031年から成り立つので,それまで待つのが吉.
(44−4)/4
>>714
sinθ=0なら成り立つ
718132人目の素数さん ◆EvsOYBFM :01/11/09 21:39
>>628はちょっと難問だね。
719蚊系:01/11/09 21:45
たった,1問です.多分
nが1,2,3,4,6以外の正の整数のとき
n次対称群の自己同型群はn次対称群になることはほぼ証明できた。
i^i(iのi乗)=e^(2n-0.5)π

誰か証明して(;´д`)
722721:01/11/09 23:02
書き忘れ
n=整数で何でもOKだそうです
>>722
>>425みれ
724132人目の素数さん:01/11/09 23:18
総当りで試したが全滅。>>710
725真剣におしえて:01/11/09 23:33
自由に折り曲がる紙面を数理モデル化することをやった数学者はいますでしょうか?

具体的には、紙面上にデカルト座標系があるとして、その紙面をぐにゃって曲げたとき、3次元空間中でその座標系がどのように表されるか。
屈曲点や特異点はできないように曲がる場合だけでいいです。

簡単なのは、まげの自由度が1の場合だけど、円すいになるだけで途端に
難しくなる。う〜〜〜〜

わかる人教えてください。

本題は、その逆変換なんだけど、上記がわかれば後は、計算機が頑張ってくれると思うので。(^_^;)
726真剣におしえて:01/11/09 23:39
>>725
ちなみに、3次元空間上で陽で示される場合だけでいいです。
端と端がくっつく場合なんて考えたら、超難問になってしまいますよね。
727132人目の素数さん:01/11/09 23:41
>>720
おせーて。

フィボ( ● ´ ー ` ● )なっち数列

( ● ´ ー ` ● ) ( ● ´ ー ` ● ) ( ● ´ ー ` ● ) ( ● ´ ー ` ● ) ( ● ´ ー ` ● ) ( ● ´ ー ` ● )
( ● ´ ー ` ● ) ( ● ´ ー ` ● ) ( ● ´ ー ` ● ) ( ● ´ ー ` ● ) ( ● ´ ー ` ● ) (;● ´ Д ` ● )
729教えて☆逸す(おしえてほしいっす):01/11/10 00:34
質問です。
『A、Bは直線l(エル)の同じ側にある2点とする。l上に点Pをとった時、
AP+PBが最小となるのはPがどの位置にあるときか?』
という問題なのです。
一応答えとして『点Aの直線lに対して線対称な点A'をとり、そことBを結
んだ時にlとぶつかったところをPにすればよい』という解答をしたら「もっ
ともだ。だが、もしそこではない適当なところで点Pをとったとして、どう
してそこが最小にならないということができるのか」と指摘され返答に困っ
てしまいました。この問題について教えて欲しいです。よろしくお願いします。
>>729
直線A'Bとlの交点をP、それ以外の点をQとして三角形A'BQを
かんがえる。(おそらく三角不等式は証明なしで使っていいだろうから)
AP+PB=A'B<A'Q+QB=AQ+QB。
731132人目の素数さん:01/11/10 00:43
>>729
A’P=APであるから, ←三角形の合同より
AP+PB=A’P+PB
となる。
つまり,A’P+PBが最小となるPの位置を
考えればよい。それは,もちろんPが線分A’B上に
あるときである。
732132人目の素数さん:01/11/10 01:14
あほばっかやな、この板は!!
ラウ房なめんなよ!!!!!!!
733教えて☆逸す(おしえてほしいっす):01/11/10 02:34
132人目の素数さん、どうもありがとうございました。
そうか、三角不等式を使うのか。うかつだったな。
また、わからない問題がありましたらカキコします。
ほんとにありがとうございました。
734132人目の素数さん:01/11/10 02:35
実数の定数θに対して
f(n)=sinθ・sin2θ・sin3θ・・・・・sinnθ (n=1,2,3,・・・)
とおく。このとき
lim_[n→∞]f(n)=0 であるといえるか。
735132人目の素数さん:01/11/10 02:42
いえる
736132人目の素数さん:01/11/10 02:50
>>734
当然いえるんじゃないの?
737 :01/11/10 03:13
test
>>614
a<0のときのx^2+x+a<0の解は
(−1−√(1−4a))/2<x<(−1+√(1−4a))/2。
739132人目の素数さん:01/11/10 11:55
どなたか助けてください!!
数学的帰納法はとっても苦手です。
よろしくお願いします。

k[j+1]=2(N-1)/N・l[j] + (N-2)/N・k[j]
l[j+1]=(N-2)/N・l[j] - 2/N・k[j]

k[0]=l[0]=1/root(N)

このkとlに関する漸化式をとくと以下がでてくる。

k[j] = sin((2j+1)θ)
l[j] = 1/root(N-1)・cos((2j+1)θ)
ただし、sin^2θ = 1/N (0 < θ <= π/2)
これを数学的帰納法で証明せよ。
740132人目の素数さん:01/11/10 13:32
>>733
ドキュソ発見!
741質問です:01/11/10 18:03
適当にn個の整数をとった時
その要素の和にnで割り切れるものが存在することを示せ。
ってことなんですけど誰か教えて下さい!よろしくお願いします。
742132人目の素数さん:01/11/10 18:32
nで割ったあまりで考えたら?
743132人目の素数さん:01/11/10 18:37
>>741
どういう形で証明すればいいのかというのが
確かに思いつきにくいかも知れないな。
とりあえず742氏の方向でもう少しがんばれ。
面白いから。
744:01/11/10 19:33
    1
∫―――――――dx
    3
   x −1

を積分せよ。
745741:01/11/10 20:45
ありがとうございました。
もう少し頑張ってみます!
746132人目の素数さん:01/11/10 21:55
2点A(1,1,3)、B(4,−1,2)がある。
A,Bから等距離にあるx軸上の点Pの座標を求めよ。
またA,Bから等距離にあるy軸上の点qの座標を求めよ。

どなたかよろしくお願します。
>>746
>A,Bから等距離にあるx軸上の点Pの座標を求めよ。
√{(x-1)^2+(0-1)^2+(0-3)^2}=√{(x-4)^2+(0+1)^2+(0-2)^2}を解け
>またA,Bから等距離にあるy軸上の点qの座標を求めよ。
√{(0-1)^2+(y-1)^2+(0-3)^2}=√{(0-4)^2+(y+1)^2+(0-2)^2}を解け
748132人目の素数さん:01/11/10 22:09
>>747
なるほどー。どうもありがとうございます。
749無駄骨さん:01/11/10 22:18
>>746
直線ABに垂直で線分ABの中点を通る平面をπとすると
A,Bから等距離にある任意の点は平面π上の点。(証明略)

πの式を出して
y=z=0を代入→点Pが求まる
x=z=0を代入→点Qが求まる
∫[0,1]√(1+x^2) がわかりません。教えてください。
751132人目の素数さん:01/11/10 22:51
あ、sageちゃった。
752132人目の素数さん:01/11/10 23:13
>>751
x=sinθとおいて後は2倍角の公式をつかって次数をおとす。
753752:01/11/10 23:13
すまん。
「+」だったか。
x=tanθとおいてみれば?
754750:01/11/10 23:23
そうすると
∫[0,π/4](1/cos^3θ)dθ になりますよね?

その後はどうしたらいいんですか?
755132人目の素数さん:01/11/10 23:28
いま、積分を勉強しています。非常に一般的な問題なのですが、正規分布の-infから+inf区間での定積分の解法を教えてください。
>>754
>そうすると
どうしたの?

>>753のいうx=tanθならおかしくない?
757つっこみ:01/11/10 23:36
>>750
∫[0,1]√(1+x^2)dt
なら
√(1+x^2)
だね。
じゃ。
758752:01/11/10 23:37
>>756
何度もすまんね。
一目見て、分数だと思ってた。
部分積分でできるよ。
こんどこそホント
759752:01/11/10 23:47
∫√(1+x^2)dx=x√(1+x^2)−∫{x^2/√(1+x^2)}dx
=x√(1+x^2)−∫{(x^2+1−1)/√(1+x^2)}dx
=x√(1+x^2)−∫√(1+x^2)dx+∫{1/√(1+x^2)}dx
と変形して、右辺第2項を左辺に移項。
あとは分かるよね?
760750:01/11/10 23:51
>>759
ありがとうございました。助かりました。

>>757
dxを書き忘れてたってことですよね?
>>756
x=tanθとおいたつもりです。
761つっこみ:01/11/10 23:56
>>760
ごめんね。
揚げ足取りだけど、数学ってこういうの致命傷になるから気をつけてね。
762752:01/11/11 00:00
>>755
求める定積分をIとすると
I^2を2重積分に直して、あとでルート
763132人目の素数さん:01/11/11 00:48
問題じゃないんですが・・・・・
A,Eを集合とする。写像f:A→P(E)を、Eの部分集合の族ともいう。
(東京大学出版会 解析入門1の397ページです)
これの意味がまったくわかりません。
764132人目の素数さん:01/11/11 01:19
>>762
それは、ちとヒントが少ないんじゃない?
I^2にするのはいいんだが、その後、∫--dx ∫--dyとしてやって、独立性を使って、重積分にするんだよね。さらには、なにするん?
>>762
結構難題だと思うよ。覚えてしまえば、楽勝だけど、一度解かないと思いつかんよね。
765>:01/11/11 01:28
>755
762を具体的に書く

∫[-∞, ∞] exp(-x^2) dx

expの中の係数は1 の場合だけでいいよね(変数変換すればいい)
偶関数だから [0,∞] についての積分が求まればいいな。

A =∫[0, ∞] exp(-x^2) dx とおく

A * A = ∫[0, ∞] exp(-x^2) dx * ∫[0, ∞] exp(-y^2) dy
= ∫∫[0, ∞][0,∞] exp(-x^2-y^2) dx dy

この重積分は
  x = r cos t
y = r sin t
と変数変換すればわりと容易
766132人目の素数さん:01/11/11 02:39
何やら出題されている模様。

http://mentai.2ch.net/test/read.cgi/mystery/1004692476/130
767132人目の素数さん:01/11/11 09:48
>>766
3が何の事を指してるか知らないが1,2は仮定が抜けてるので何かのコピペと
思われ。相手にする必要ないと思われ。
768132人目の素数さん:01/11/11 10:58
以下を教えてください.よろしくお願いします.
制限つき尤度の議論のところで出てくる行列代数です.
-----------------------------------------------------------
行列M:n×(n-p),V:n×n,X:n×pについて
M'X=O
が成り立ってるとする.このとき,以下が成り立つ.
M(M'VM)^{-1}M'=V^{-1}-V^{-1}X(X'V^{-1}X)^{-1}X'V^{-1}
|M'VM|=|M'M||V||X'V^{-1}X|/|X'X|


((表記の注意))
X'はXの転置を意味する.
76918:01/11/11 11:23
難問です。a,bの二つの文字を2のn乗個円形にうまくならべると、連続するどのn個も互いに違う列にできることを証明したいのですが、どう考えてもうまくできません。誰か教えてください。
770>:01/11/11 11:31
任意個でなくn個のパタンについてでいいなら
(左右反転は区別 abb と bba は別とする)

a を n個かためて bをn個固めて ならばべる配置にすれば
どこからn個とりだいしても、左に固まってる a の数が違う
771質問です。:01/11/11 11:35
集合Xi⊂R^n      (i=1,2,・・・,m)(各Xは閉集合かつ下方有界)
集合 Y⊂R^n      (Yは凸かつ閉集合)
0∈Y,Y∩R^n+={0}(R^n+は、n次元ユークリッド空間の正象限)

F=ΠXi×Y (滅茶苦茶読み難くて申し訳ないのですが、Fは全てのXiとY
         との直積です。)

「Fがコンパクト集合であることを証明せよ」という問題なのですが、手がかりが
掴めません。文系厨房質問で恐縮なんですが、教えていただけないでしょうか。
772ちむ教の信者:01/11/11 11:50
二次関数で、質問があります。

二次関数 f(x)=x^2ー2x+3について次の問いに答えなさい。

(1) ー1≦x≦aにおける最小値を求めよ。

(2) ー1≦x≦aにおける最大値を求めよ。

これは、aは任意の数かつ∞なんですよね。
つまり、答えはひとつのはずですよね。最も〜なんですから。
当方の答えだと、(1)は頂点、(2)はa(9999・・・・、この世で最も大きい数)
なんですが、解答をみると、なんかほかに答えが載ってます。
お願いです。教えてください、なぜなのですか?
773ちむ教の信者の母:01/11/11 11:59
うちの子も、これからは心を入れ替えてがんばると言っておりますので、よろしくお願いします
774おかma:01/11/11 12:21
>>772
場合分けをするのよ。
あを−1から右へ少しずつずらして逝くと、最大値と最小値が変わるでしょう。
あを?
776おかma:01/11/11 12:37
aってこと分からないかしら?
777ちむ教の信者:01/11/11 12:49
訂正
二次関数で、質問があります。

二次関数 f(x)=x^2ー2x+3について次の問いに答えなさい。

(1) ー1≦x≦aにおける最小値を求めよ。

(2) ー1≦x≦aにおける最大値を求めよ。

これは、aは任意の数かつ∞なんですよね。
つまり、答えはひとつのはずですよね。最も〜なんですから。
当方の答えだと、(1)は頂点、(2)a^2ー2a+3は(9999・・・・、この世で最も大きい数)
なんですが、解答をみると、なんかほかに答えが載ってます。
お願いです。教えてください、なぜなのですか?

場合わけはなんで必用なのですか?
             ∩
                  | |
                  | |
         ∧_∧   | |   / ̄ ̄ ̄ ̄ ̄ ̄ ̄ ̄ ̄
  / ̄\( ´Д`)/./  < おカマはすっこんでろ!
.r ┤ @ ト、      /    \_________
|.  \_/  ヽ   /
|   __( ̄  |   |
|    __)_ノ ̄ ̄ ̄ ̄\
ヽ___)_ノ          \
  ||\|| ̄ ̄ ̄ ̄ ̄ ̄ ̄|| ̄
  ||  || ̄ ̄ ̄ ̄ ̄ ̄ ̄||
     .||              ||
779ちむ教の信者の母:01/11/11 12:52
おかまじゃない正常な方、どうかお願いします。
780おかma:01/11/11 12:59
>>777 いい番号ね。

あは任意なんでしょ。ならあ=0でもOKでしょ?
じゃあこのとき(あ=0のとき)の最大値と最小値を求めてみなさい。

こうしてあを変化させていくと最値が変わっていくでしょ。
あなたの答えだとあ≧3のときのこたえでしかないわ。
781おかma:01/11/11 13:03
>>777
それに任意かつ無限大なんて表現はないわ。
782ちむ教の信者:01/11/11 13:08
教えてください。解説を。
783ちむ教の信者の母:01/11/11 13:10
おかま以外の方、お願いします。
784答え:01/11/11 13:15
(1)
-1≦a<1の時、a^2ー2a+3
1≦aの時、2

(2)
-1≦a<3の時、6
3≦aの時、a^2ー2a+3
785ヒント:01/11/11 13:21
(1)
-1≦a<1の時、(a,a^2ー2a+3)
1≦aの時、(1,2)

(2)
-1≦a<3の時、(-1,6)
3≦aの時、(a,a^2ー2a+3)
786名無しさん:01/11/11 14:22
すみません。
http://isweb21.infoseek.co.jp/play/akinga/speace/mm.gif
このXってどうやって求めるんですか?
普通の公立中学校の知識でお願いします。
ちなみに、僕は中三です。
787名無しさん:01/11/11 14:31
厨房問題スレにもかいてみました。
>厨房問題スレにもかいてみました。

死ね
>厨房問題スレにもかいてみました。
じゃ,そっちでやるってことで.
790名無しさん:01/11/11 14:47
>>789
はい。そうしてください。
かなりきついですね。
結構傷ついたというか、思ったよりも心にダメージが・・・。
791ちむ教の信者:01/11/11 14:56
あ、なるほど。グラフを書いて考えてみました。
また、同じ問題で質問があります。
(2)の解答が、−1≦a<3のときと、
書いてあるのですが、これはなぜですか。
当方の見解だと、−1≦a<≦3までが、6になるのですが。
場合わけはなぜ〜以上t<〜になるのですか?
〜≦t≦〜でもいいのでは?
792132人目の素数さん:01/11/11 15:00
>結構傷ついたというか、思ったよりも心にダメージが・・・。

おもしろい。まあ気にするな。それより
あの問題が解けない事に危機感を持ったほうがよい。
793おかma:01/11/11 15:07
>>791
それはどちらでもいいのよ。−1≦あ<3でも−1≦あ≦3でも。
>>784のを借りると

-1≦a<3の時、6
3≦aの時、a^2ー2a+3   ←この式にあ=3を代入してもa^2−2a+3=6となるでしょ。
 ↓
-1≦a≦3の時、6
3<aの時、a^2ー2a+3

上でも下でも同じことなのよ。
794ちむ教の信者の母:01/11/11 15:14
おかま以外の方、お願いします。
795ちむ教の信者:01/11/11 15:21
−1≦a≦3までが←訂正
796ちむ教の信者:01/11/11 16:40
なるほど、わかりました。
しかし、逆の問題の、

f(x)=x^2−2axのー1≦x≦1
での最大値、最小値を求めよ、で、もう理屈は
わかったのですが、具体的にaは〜以上、〜以上a〜以下っていう、
〜つまり、数字がわかりません。
さきほどの問題はx軸を中心にやればよいから、
わかるのですが、これは軸がaなのでわかりません。
そのようにすれば、簡単にわかりますか?

>>794
せっかく、教えてくれてんだから。言わないで。
797>:01/11/11 16:55
f(x) = (x-a)^2 - a^2
だから この放物線の縦の真ん中って x = a の ところ

このまんなかが、 着目区間(-1,1) の
    1 左外にある場合
2 内側にある場合
3 右外にある場合
どうなるか 絵をかいてみる。。。
798おかma:01/11/11 16:57
>>796
今度は2つの直線x=−1、x=1(2つともy軸に平行ね)の間に挟まれている部分を
斜線でも引いておきなさい。

そうして
 ・f(x)=0の解は0、2あ  ←必ずy=f(x)のグラフは原点を通るわね。
 ・y=f(x)のグラフは下に凸
ということを考えると

・(軸x=)あ<−1のときの最大値=f(1)=1−2あ
              最小値=f(−1)=1+2あ

・(軸x=)あ>1  のときの最大値=f(−1)
              最小値=f(1)

・−1<(軸x=)あ≦0のとき最大値=f(1)
                  最小値=頂点のy座標=あ^2

・0<(軸x=)あ<1のとき最大値=f(−1)
                最小値=頂点のy座標

となると思うわ。
799ちむ教の信者の母:01/11/11 17:08
おかまと口聞いちゃだめよ!
800質問・・・:01/11/11 20:49
これをお願いします。

三角形OABにおいて、
 OA=5 OB=4 ∠OAB=60度
とし、点Oから辺ABにおろした垂線の足をHとする。
Vector(OA)=Vector(a) Vector(OB)=Vector(b)とおくとき、
Vector(OH)をVector(a)、Vector(b)を用いてあらわせ。

頻出問題だと思うんですが、この手の問題はなぜか苦手・・・。
801132人目の素数さん:01/11/11 20:56
Vector(OH)=Vector(b)
802ベクトル:01/11/11 21:00
>>801
その根拠は?
803132人目の素数さん:01/11/11 21:07
↑チガウヨ
804132人目の素数さん:01/11/11 21:07
↑1つズレちまった
805ちむ教の信者:01/11/11 21:30
ちょっと、数学の二次関数の壁を乗り越えた気がしています。
ありがとうございました。

質問です。

和がaであるような2つの正の数の平方
の和が最小となるのは、
どんなときか。
また、そのときの和の最小値を求めよ。

>この問題の日本語的な意味および解き方がわかりません。
解説お願いします。
806質問:01/11/11 21:31
  この問題をお願いします!
 
  実数x、yの間に”x二乗-2xy+2y二乗”の関係があるとき、
  xとyとx+yの値の範囲を求めよ。

  
807132人目の素数さん:01/11/11 21:42
>>806
どんな関係?
808132人目の素数さん:01/11/11 21:51
だ、誰か>800を助けてくだされ・・・。
809おかma:01/11/11 22:04
>>808
60゚なのは∠OABであってるの?
∠AOBじゃないのかしら?
810おかma:01/11/11 22:08
>>805
その2つの性の数をx、あーxと表せれるのはお分かり?
811おかma:01/11/11 22:08
>>810
0<x<あ の条件付でよ。
812おかma:01/11/11 22:12
>>806
”x二乗-2xy+2y二乗=♥”
               ̄ ←この♥のとこに何か数字が与えられていないかしら?
813132人目の素数さん:01/11/11 22:13
>>800
AB=(b-a)
OH=(b-a)t+a
として
AB・OH=0となるtを求める。(tだけスカラ)
AOB=60なら結果も簡単
814ベクトル:01/11/11 22:16
>>813
>>809

すみません。ミスです。
∠AOBが60度です。
失礼しました。
815おかma:01/11/11 22:17
>>814
いいってことヨ♥
816おかma:01/11/11 22:19
>>814
あ、もしかして∠AOBが60゚と分かってもまだ問題が解けないの?
817ベクトル:01/11/11 22:20
>>816
ただいまやっている途中でござんす。
818ベクトル:01/11/11 22:26
>>816
でけました。

AH=tABとおくのを忘れていたか、
それ以外の妙なとき方をしようとしていたようです。

皆様どうもありがとうございました。
819質問:01/11/11 22:32
あっ!!ホントだ!x二乗-2xy+2y二乗=1でした。。。
すみません。
820おかma:01/11/11 22:42
>>819
自信は少しないけど
答えは

−√2≦x≦√2、−1≦y≦1、−√5≦x+y≦√5

だと思うけどどう?解等はもってないのかしら?
821おかma:01/11/11 22:50
ところで>>805はどこに逝ったのよ。
気になるじゃないのよ。
>>821
やっぱり次スレから「ちむ信は放置」って明記しといたほうがいいな。
823おかma:01/11/11 23:03
まぁいいわ。もしかしたら>805の家が火事になったのかも知れないし。
世の中、何があるか分からないと思うの。
>>806
>x^2-2xy+2y^2-1=0

yの二次式と見て整理すると
2y^2-(2x)y+(x^2-1)=0
yが実数解を持つ条件は
判別式D/4=x^2-2(x^2-1)>=0
これを解けばxの範囲が出る

xの二次式と見て整理すると
x^2-(2y)x+(2y^2-1)=0
xが実数解を持つ条件は
D/4=y^2-(2y^2-1)>=0
これを解けばyの範囲が出る

x+y=kとおいてx^2-2xy+2y^2-1=0の式からyを消去すると
5x^2-6kx+(2k^2-1)=0
xが実数解を持つ条件は
D/4=(3k)^2-5(2k^2-1)>=0
これを解けばk=x+yの範囲が出る

答えは>>820に同じ
825おかma:01/11/11 23:16
>>824
なるほどネ。
私のやり方はすっきりキリキリ切り子ちゃんよ。

変形して (x−y)^2+y^2=1 ・・・@ → x−y=cosθ、y=sinθ  θは任意の値ネ
もうこれで −1≦y≦1 はでてるわ。
そしてxは x=cosθ+sinθ=√2sin(θ+π/4) となって θは任意だから -√2≦x≦√2 となるわね。

x+yも同様よ。 x+y=cosθ+2sinθ=√5sin(θ+α)  (ただしcosα=2/√5、sinα=1/√5)

これで -√5≦x+y≦√5 となるわ。

@の式から sin、cos に気付くと早いわよ。θには何の制約も無いから楽になるわ。
826おかma:01/11/11 23:22
自分で言うのもなんだけど、上はちょっとうまいわね・・。
わたしもまだ捨てたもんじゃないわね♥
おかまも無視ということでよろしく
>>825
汎用性に欠けるかも
与式が(x-2y)^2-y^2=1の場合はsinh,coshで解くのかね
829おかma:01/11/11 23:38
一つ言っておくと、別に
x^2−2xy+2y^2=1 の右辺が1じゃなくても上のやり方でいけるわよ。

x^2−2xy+2y^2=k(kは実数) となっていても、
{(x−y)^2}/k+y^2/k=1 となって ±(x−y)/√k=cosθ、±y/√k=sinθ とおけば
範囲は (1/√k)が掛けられたものになると思うわ。 k>0なら絶対OKね。

>>828
それは因数分解できるわね。 (x−y)(x−3y)=1となるわ。
そして X=x−y、Y=x−3y として XY=1・・・@
2y=X−Y、2x=3X−Yで @のグラフと左の2直線の切片の範囲でx、yの範囲は出ると思うわ。
x+yも同様でね。
830質問:01/11/11 23:39
長々とありがとうがざいました。
理解できたような気がします。
でも、おかまさんの解き方すごい複雑…。
831おかma:01/11/11 23:40
ちょっと>>829の上はおかしかったわ。k>0じゃないとだめね。
ただ、k<0なら √(−k)とすればOKね。場合分けすれば一般化できるとおもうわ。
832おかma:01/11/11 23:44
>>830
そんなことないわよ。超簡単よ!
あなたが出した問題自体ならすぐsin、cosに気付くはず。そのあと2つを合成したら終わりよ。

(ある数の2乗)+(別の数の2乗)=1 なら まずは sin、cosの置き換えを考える事ね。
833オサレもどき: ◆Spitzrvs :01/11/11 23:49
とき方度忘れしました。
誰かお願いします。


n^3-3n^2+2n=504
834おかma:01/11/11 23:54
>>833
ん=9となるわね。
与式の左辺は因数分解して (んー2)(んー1)n=504=7・8・9  これで終りね。
835おかma:01/11/11 23:57
解き方というより素因数を適当に分解していったらいづれ気付けると思うわ。
836ちむ教の信者:01/11/12 00:07
質問があります。

y=2x^2−4x+3の 0≦x≦aに
おける最大値・最小値を求めよ。
で、当方は場合わけを試みたのですが、なんか解答と
違います。0≦a<1,1≦a<2、2≦a
なんですが、解答のと、不等号がちがいます。
どうやれば、不等号をちゃんとできますか?

また、a>2のとき、解答は最小値を1と書いているのですが、
1は頂点で、a=1のときなんですよ。
なぜ、このときの最小値が1なのでしょうか?
837132人目の素数さん:01/11/12 00:07
Pが素数のならば(P-1)!+1はPの倍数。
[証]
838オサレもどき: ◆Spitzrvs :01/11/12 00:20
サンキュデス 簡単でした
839おかma:01/11/12 00:21
>>836
上のほうの質問については、不等号を下のようにしていても正解よ。
 0≦あ≦1、1<あ≦2、2<あ
 0≦あ≦1、1≦あ≦2、2≦あ
あまりこの問題では気にしなくていいわ。
    ~~~~~~~~~~~~~
下の質問は、
あ>2のとき、直線x=0と直線x=あ に挟まれた部分の最小値が1なのよ。
上のほうで前にも言ったけど、↑この2直線ではさまれた領域を斜線をしてみなさい。
での最小値が1ってことよ。

とにかくグラフを書きなさい。もう寝るわ。
>>828
汎用性なら >>824 も二次式のときにしか使えない。
>>840
you win
842お願いします:01/11/12 01:28
教えてください。。。

n人のなかで誕生日が少なくとも2名が一致する確率は、余事象を使って

1−(365Pn)/365^n ですが、

誰も一致しないという部分の365Pnがわかりません。365Cnでは駄目なんですか?
843工房一年生:01/11/12 01:40
分からない問題があるので、教えていただけませんか?

〇点Pから放物線y=x^2に引いた2つの接線が直交するとき、点Pの軌跡を求めよ。

お願いします。
844132人目の素数さん:01/11/12 02:10
y=-1/4の直線。

二つの接線の方程式から点Pを記述し、
直交の条件(傾きの積が-1)を用いる。
845132人目の素数さん:01/11/12 02:13
>>842
じゃあ何で分母を365^nにしたの?
846工房一年生:01/11/12 02:26
>>844
あ…そっか、なるほど。ありがとうございます。
847132人目の素数さん:01/11/12 02:33
質問、いいすか?

{a(n)},{b(n)}が等差数列ならば、次の数列も等差数列であることを示せ。

(1) {a(5n)}

(2) {2a(n)-3b(n)}
一般項を作って
{a(5(n+1))} - {a(5n)} = nによらない定数
になればよい。(2)も同様。
849132人目の素数さん:01/11/12 03:37
>>847
a(5n)-a(5(n-1))
=a(5n)-a(5n-5)
={a(5n)-a(5n-1)}
+{a(5n-1)-a(5n-2)}
+{a(5n-2)-a(5n-3)}
+{a(5n-3)-a(5n-4)}
+{a(5n-4)-a(5n-5)}
これはnによらない定数.

{2a(n)-3b(n)}-{2a(n-1)-3b(n-1)}
=2{a(n)-a(n-1)}-3{b(n)-b(n-1)}
これはnによらない定数.
850132人目の素数さん:01/11/12 11:10
代数体Kの有理数体上の「整基底」って何者ですか?
「基底をKの整数に限定してとれ」ってことなのでしょうか?
ひとつよろしくお願いします。
851132人目の素数さん:01/11/12 12:45
ちょっと場違いな気がしますが教えて下さい。

http://www.sougetu.com/funpic/damasie/true_true.html

↑何でこうなるの?
場違いなら,教えません.さげ
>>851
何か不思議なことでも?
854目の錯覚ですね。:01/11/12 12:54
y = 5/13 x で、yが整数になるようなxは、、x=0の次はx=13まで存在しない。
>目の錯覚ですね。
目が悪いんだよ
856851:01/11/12 17:08
よく見たらわかりました。すみませんでした・・・。
1;U,WをVの部分空間でU+W=V(直和)としBu={u1,・・・・,un}をUの正規直交基底、
Bw={w1,・・・・・,wm}をWの正規直交基底とする。
2;UをVの部分空間としBu={u1,・・・・,un}をUの正規直交基底、
Bu⊥={u'1,・・・・,u'm}をU⊥の正規直交基底とする。

このとき1、2のそれぞれBu∪Bw、Bu∪Bu⊥ はVの正規直行基底と言えるか。

なんですけどどっちも同じことじゃないかと思うんです。
1はW=U⊥になると思うし・・・・。
どなたか教えてください。お願いします。
>>857
1はいえない。2はいえる。
>1はW=U⊥になると思うし・・・・。
これがいけない。Vを2次元ユークリッド空間、Uを(1,0)で張られる空間、
Wを(1/√2,1/√2)ではられる空間にしたらUとWは直交してないっしょ。
>>850
そだよ。
>>858
そっかー。親切にありがとうございます。助かりました。
861132人目の素数さん:01/11/12 21:23
中学生です。
なんで素数に一が入らないんですか?
素因数分解の一意性が崩れるからですか?
なんか納得いかなかったので・・。
ヨロシク。」
>>861
>素因数分解の一意性が崩れるからですか?
そう思っててまちがいないんじゃないと思われ。
>なんか納得いかなかったので・・。
世の中納得いかないことのほうが多いことを覚えときたまへ。
ドキュソダウ学生です。
ヤコビアンの意味がわからないのですが、
何故変数変換したらヤコビアンを
かけるのでしょうか?誰か証明おねがいします。
微分形式の引き戻しを計算するとヤコビアンが出てくるんです
862さまありがとうございますです。
>>864
私どーしようもない阿呆なので
微分形式の引き戻しとは
何なのかすらわからないんで
そこから教えていただきたいんですが
何卒よろしく
簡単に言えば代入
868132人目の素数さん:01/11/12 23:16
851わかりません
869132人目の素数さん:01/11/12 23:18
>>868
図の線が太いのが原因。
ちゃんと、傾きとか調べて、図の格子点を通るかどうかチェックしてみよう!
>>868
面積1の細長い平行四辺形が正方形にシフト
871132人目の素数さん:01/11/13 01:23
f(n)=∫[0,1]1/√(1+x^n)dx

このとき、limf(n) n→∞ を求めよ。

教えてください。
872名無し:01/11/13 01:39
873132人目の素数さん:01/11/13 02:18
あーっと・・ちょっと教えてほしい問題があるんですが・・。

0°≦θ<360°のとき、方程式4sin^2θ-4cosθ-5+a=0の解の個数を、定数aの値によって分類せよ。
>>873
(sinθ)^2-cosθ=0
ならわかる?
875132人目の素数さん:01/11/13 02:45
>>859
そですか。ども^2。
876通りすがりの者:01/11/13 04:31
>>871
むかしここらの板で解いた記憶があるのだが。
はさみうちで、
1≧1/√(1+x^n)≧1-x^n/2(0≦x≦1)
で解けたはず。
877 ◆P/Pe9sxI :01/11/13 04:44
>>873
cosθ=x とすれば 与式左辺=-2*x^2-4*x-3+a となります。
0°≦θ≦360°より-1≦x≦1です。
-2*x^2-4*x-3+a=0 を 2*x^2+4*x+3=a と変形しておいて、
y=2*x^2+4*x+3 (-1≦x≦1) と y=a との共有点がどうなるかを
考えてみましょう。
878 ◆pvySbQO2 :01/11/13 07:00
>>628
nを1,2,3,4,6以外の正の整数とする。
n次の対称群S(n)を1以上n以下の整数の置換全体とし
fをS(n)の自己同型写像とする。
互換(p,q)は位数が2なのでf((p,q))は位数が2になるので
互いに素な1個以上の互換の積になる。

(p,q)(p)(q)=(p,q)
(p,q)(p,q)=(p)(q)
(p,q)(p,r)(q)=(p,q,r)
(p,q)(p,r)(q,s)=(p,s,q,r)
から互換に位数2の置換を掛けたものの位数は
1,2,3,4,6のどれかになるので
f((p,q))に位数2の置換を掛けたものの位数は
1,2,3,4,6のどれかになる。

 (p,q)(r,s)(t)(p)(q,r)(s,t)
=(p,r,t,s,q)
から互換が2個以上あり動かさない元が1個以上あると
位数2の置換を掛けたものの位数は5の倍数になることがあり
 (p,q)(r,s)(t,u)(v,w)(p)(q,r)(s,t)(u,v)(w)
=(p,r,t,v,w,u,s,q)
から互換が4個以上あると
位数2の置換を掛けたものの位数は8の倍数になることがあるので
f((p,q))は互換になる。

(p,q)(p,q)=(p)(q)
(p,q)(p,r)=(p,q,r)
(p,q)(r,s)=(p,q)(r,s)
なので互換a,bについて
abの位数が3。<=>a=(p,q),b=(p,r)と表せる。
となる。
879 ◆pvySbQO2 :01/11/13 07:00
(1,2)(1,3)の位数は3なので
f((1,2))f((1,3))の位数は3になるので
f((1,2))=(x(1),x(2))
f((1,3))=(x(1),x(3))
と表せる。
4≦i≦nとするとf((1,i))は
f((1,2)),f((1,3))との積の位数が3なので
(x(1),x(i))か(x(2),x(3))になる。
f((1,i))=(x(2),x(3))とすると
4≦j≦n,j≠iとするとf((1,j))は
f((1,2)),f((1,3)),f((1,i))との積の位数が3なるが
そのようなものは存在しないので
f((1,i))=(x(1),x(i))となる。

1≦i≦nとなるiに対してx(i)をiに移す置換をaとすると
f((1,i))=(x(1),x(i))=a(1,i)a^(−1)となり
任意の置換cは(1,i)(2≦i≦n)の積で表せるので
f(c)=aca^(−1)となる。

よってa∈S(n)に対してg(a)を
g(a)(c)=aca^(−1)(c∈S(n))
となるS(n)の自己同型写像とすると
S(n)の自己同型写像は全てg(a)と表せる。

g(a)=g(b)とすると任意の置換cに対して
aca^(−1)=bcb^(−1)となり
(b^(−1)a)c=c(b^(−1)a)となる。
もしb^(−1)aが恒等置換でないとすると可換にならない置換があるので
b^(−1)aは恒等置換になりa=bとなる。

よってgはS(n)からS(n)の自己同型写像群への全単射であり
 (g(a)g(b))(c)
=a(bcb^(−1))a^(−1)
=(ab)c(ab)^(−1)
=g(ab)(c)
なのでgはS(n)からS(n)の自己同型写像群への同型写像になるので
S(n)の自己同型写像群はS(n)になる。

同じ方法でnを9以上の整数とすると
n次の交代群の自己同型写像群はS(n)になることが示せます。
880132人目の素数さん:01/11/13 10:29
a bが任意の定数のとき(a≠b)2次方程式
3(a-b)x^2+6bx-a-2b=0は少なくとも0と1トの間に解が1つあることを示せ
という問題なのですが

解答では 次のように書いています

f(x)=3(a-b)x^2+6bx-a-2bとさだめ
f(0)f(1)=(-a-2b)(2a+b)<0のとき 0<x<1 に少なくとも1つの解があり
条件をみたす
f(0)=0のとき
f(x)=-3bx(3x-2) となり  0<x=2/3<1 となる解2/3をもつので 条件をみたす
f(1)=0のとき
f(x)=3a(x-1)(3x-1)となり      0<x=1/3<1となる解をもつので条件をみたす
以下略…

こうなるのですが 少し疑問があります それは最初の方に『f(0)f(1)=(-a-2b)(2a+b)<0のとき 0<x<1 に少なくとも1つの解があり条件をみたす』と書いてありますが この書き方が少しおかしいのじゃないかって
事です
だって f(0)f(1)<0⇔0と1との間に少なくとも1つ解がある  が成立するのだから 『どのような条件の
とき
f(0)f(1)=(-a-2b)(2a+b)<0が成立するのか』こそ示さなければならない事柄だと思うのです
それを さも自明かのように
『f(0)f(1)=(-a-2b)(2a+b)<0のとき』と書いてしまうのは少し問題があるんじゃないでしょうか?
 つまりこの書き方だと 『この色が青のとき 青なのだよ』と説明されているのと同じではないか?
と思うのです そのような書き方ではなくて『この色は赤でも黄色でも無い 青と断定せざるをえない
よってこのとき この色は青だよ』という風にかかなくてはいけないとおもうのです

だから 『題意より a bは任意なので(-a-2b)(2a+b)<0をみたすa bは 必ず存在する
よって(-a-2b)(2a+b)<0のときf(0)f(1)<0 となるので このとき 0と1の間に
解をもつ』と書くのが 正しいのではないか とおもうのですが どうなんでしょうか?

私の考えは正しいでしょうか?
うひゃひゃ、高校以降の解答はそんなに親切じゃないのだよ(笑)。
(-a-2b)(2a+b)<0になる条件は自分で求めるのじゃよ。
882132人目の素数さん:01/11/13 11:21
なんか変じゃない?別にf(0)f(1)<0となるa bなんてなくてもこの段階では
単に長ったらしく無意味な事を書いただけでしょ。かならずそうなる訳でもなし。
883132人目の素数さん:01/11/13 11:41
>882  なにいってんの!任意なんだから必ず f(0)f(1)<0になる
a bは存在するでしょーが!
>>880
評価は「以下略…」次第。なんで略すかねえ。
885132人目の素数さん:01/11/13 11:48
>883  なにいってんの!以下略…
886132人目の素数さん:01/11/13 12:29
無意味に長い文章。YOHOOにもマルチポスト。マスマニアの予感。
887悩める新聞紙:01/11/13 12:58
小学4年生の宿題に新聞紙3枚を使って1辺が1mの正方形を作るのですが、ただし、ハサミ等は使わない。(切ってはダメ!!)
888132人目の素数さん:01/11/13 13:04
難しいなあ>887 だれかわかる?
889パァ:01/11/13 16:46
x+y=3,x二乗+y二乗=8のとき

(1)xy

ワカラナイ…だれか解説して…
890にゃ=ん?:01/11/13 17:03
x^2+y^2=(x+y)^2-2xy

∴2xy=(x+y)^2-(x^2+y^2)

∴xy=(3^2-8)/2=1/2
891132人目の素数さん:01/11/13 17:07
>>887-888
自作自演で難しがられても問題の趣旨がサッパリわからんのだが。

     ここらへんが正方形
         ↓
         ____
       /|     /|
     /  |    / |
   /    |   /   |
  | ̄ ̄ ̄ ̄ ̄|    |
  |       | 聞 |
  |       |新  |
  |  新聞  |   |
  |       |  /
  |       |/
    ̄ ̄ ̄ ̄ ̄
ってことじゃダメなのか?
892ポケット:01/11/13 17:07
楕円
(x/a)^2+(y/b)^2=1
の各点での曲率をどうやって求めれば良いか教えて下さい。
893にゃ=ん?:01/11/13 17:11
>>887
新聞紙の一辺って何センチ?
>>878
ついに対称群の答えがでた。じゃあ、おれは交代群やってみよ。
補題 g∈G=A(n)が長さ3の循環置換⇔g≠e,g^3=e,gはC_G(g)の中心。
これ簡単。
補題 長さ3の循環置換g,hに対しx∈S(n)の元でg=xhx^(-1)とかける。
これも簡単。この2つから
補題 φ∈AutA(n)、1≦k≦n-2にたいしx∈S(n)を
xφ(k,k+1,k+2)x^(-1)=(k,k+1,k+2)ととれる。
をしめせば
命題 S(n)→AutA(n)を自然な準同型とする。これは全射である。
がいえる。こっからAutA(n)はすぐもとまる。
というか標準的な有限群論の教科書ならのってんじゃないの。鈴木みっちーの本とか。
>>894
訂正。最初の補題はn=6では成立しないので別途証明必要。
>>894-895
n=6の場合は
補題 g∈G=A(6)が長さ3の循環置換
⇔g≠e,g^3=e,gはどんなクラインの4群(に同型な部分群)の中心化群にもふくまれない。
を利用すればいい。
>>894-896
ごめんn=7のばあいもだめだった。
補題 g∈G=A(7)が長さ3の循環置換 ⇔g≠e,g^3=e,C_G(g)はC_3×A_4
も追加。
>>896
イパーイリソスツカテゴメン。テイセイ。スマソデス。
⇔g≠e,g^3=e,gはどんなクラインの4群(に同型な部分群)の正規化群にもふくまれない。
デシタ。
899628:01/11/13 20:32
みなはんありがとうございます
漏れもn≠6で、中心化群の位数を考えることにより
互換は互換、長さ3の巡回置換は長さ3の巡回置換に写せて、そこから
φ∈Aut(S(n)),Aut(A(n))が内部自己同型になり、S(n)と同型になることは示せました
あとはn=6でてこづっているんだけど、896を参考にしてやってみます!
900628:01/11/13 20:38
あ、S(6)はどうなるの?
>>900
A(n)はS(n)の特性部分群(⇔任意の自己同型で保存される)から
AutS(n)→AutA(n)が自然に定義されるんじゃないの?
だから>>894
>命題 S(n)→AutA(n)を自然な準同型とする。これは全射である。
がA(n)についていえれば
命題 S(n)→AutS(n)を自然な準同型とする。これは全射である。
もいえると思うんだけど。
902132人目の素数さん:01/11/13 21:52
三角形ABCにおいて、

ABを3:2に分ける点をE
ACを3:4に分ける点をF

とし、BFとCEの交点をPとし、
APとBCの交点をDとする。

このときAP:PDがどうなるのか教えてください!
903質問です:01/11/13 22:08
熱・統計力学の講義でガウス積分がでてきました。
で、レポートで数学的帰納法により、答えを出せというので、
1回微分で1 2回で3 3回で15 4回で105・・・と
増えていってる部分をn回すると。。。というので、
数列をつかってるようなんですけど、
どうやって表現したらよろしいでしょうか。。。
>>902
お化け向きなんだけど。以下ベクトルXYを[XY]と書くとして
Aを始点、[AB]、[BC]をつかう。
>ABを3:2に分ける点をE
から[AE]=3/5[AB]
>ACを3:4に分ける点をF
から[AF]=3/7[AC]
よって直線CE上の点Pの位置ベクトルは
[AP]=s(3/5)[AB]+(1-s)[AC]
直線BF上の点Pの位置ベクトルは
[AP]=t[AB]+(1-t)(3/7)[AC]
[AB][AC]は一次独立だから係数比較して連立方程式つくってとく。
...中略...
s=10/13,t=6/13。∴[AP]=6/13[AB]+3/13[AC]
DはAP上なので
[AD]=k(6/13)[AB]+k(3/13)[AC]
AB上でもあるので
[AD]=l[AB]+(1-l)[AC]
係数比較してまたといて[AD]=(2/3)[AB]+(1/3)[AC]
...
なれてくると[AP]がわかった時点でもう答えはみえるけど。
905パァ:01/11/13 22:19
>>890
なぜ2xyと出せたんですか!?
906132人目の素数さん:01/11/13 22:52
確率変数Xのラプラス変換M_X(t)を
M_X(t)=E(exp(tX))
で定義する時,
M_Y(t)=\sum_{i=1}^{m}{p_i*M_{X_i}(t)} (1)
ただし,\sum_{i=1}^{m}{p_i}=1である。
とき,YとX_i,(i=1,2,\cdots,m)の分布関数の間に,
F_Y(x)=\sum_{i=1}^{m}{p_i*F_{X_i}(x)} (2)
なる関係が成り立ちます。ただし,
F_X(x)=Pr{X\leq x}
です。
で、証明はわかるんですが、(1)⇒(2)が成立する
直感的な解釈は出来ますか?
907132人目の素数さん:01/11/13 22:55
複素数zの関数 f(z)=1/(z^2*sin(z))
について、f(z)の留数をすべて求めよ。  よろしくお願いします。
なにげに900超えてるな。
>>907
f(z)は有理型関数。
“lim[z→α](z-α)f(z)=cのときf(z)の点αでの留数はc”
をつかえばz=0以外の点(z=nπ)の留数はすぐでる。
g(z)=z/sinzとおくとf(z)=g(z)/z^3でg(z)はz≠nπ(n≠0)
で正則な関数。そこで
g(z)/z^3のz=0での留数
=g(z)/z^3のz=0での-1次のローラン展開の係数
=g(z)のz=0での2次のローラン展開の係数=2次のテーラー展開の係数
それはg(z)を2回微分してz=0代入すればわかる。
910132人目の素数さん:01/11/13 23:32
>>909
g(z)を2回微分してz=0って代入できるんですか?
911132人目の素数さん:01/11/13 23:40
>904
すみません・・・。リア厨なんで、3〜6行目しか理解できません・・・。
最後の行は「なるほど!そうなのか!」って感じなんですけど・・・。

お願いします。
912おかma:01/11/13 23:57
>>911
-------------------------------------------------------------
>>904の一部
>ABを3:2に分ける点をE
から[AE]=3/5[AB]
>ACを3:4に分ける点をF
から[AF]=3/7[AC]
よって直線CE上の点Pの位置ベクトルは
[AP]=s(3/5)[AB]+(1-s)[AC]
直線BF上の点Pの位置ベクトルは
[AP]=t[AB]+(1-t)(3/7)[AC]
[AB][AC]は一次独立だから係数比較して連立方程式つくってとく。

^^^^^^^^^^^^^^^^^^^^^^^^^^^^^^^^^^^^^^^^^^^^^^^^^^^^^^^^^^^^^^^^^^^^^^^^^^
 EP:PC=(1−s):s (ただし0<s<1)
 BP:PF=(1−t):t (ただし0<t<1)
としているのは分かるかしら?

s、tという文字はは勝手に比の文字として決めただけよ。
913132人目の素数さん:01/11/14 00:04
>912

うー・・・・なんとなく・・・わかるような気がします。
ベクトルってのがよくわかりませんが・・・。
914おかma:01/11/14 00:18
>>913
AP↓を下のように2つの式で表しているのよ。

AP↓=s・AE↓+(1−s)・AC↓
AP↓=t・AB↓+(1ーt)・AF↓

上の2つの式に
 AE↓=(3/5)・AB↓
 AF↓=(3/7)・AC↓    の2つを代入したら

AP↓=s・(3/5)・AB↓  +  (1−s)・AC↓
AP↓=     t・AB↓+(1−t)・(3/7)・AC↓
 ̄ ̄ ̄ ̄ ̄ ̄ ̄ ̄ ̄ ̄ ̄ ̄ ̄ ̄ ̄ ̄ ̄ ̄ ̄ ̄   ここまで分かるかしら?
915902:01/11/14 00:39
>914

なんとなくわかりますです。
ところで矢印はなんですか?
916おかma:01/11/14 00:47
>>915
矢印の方が分かりやすくないかしら?
ベクトルAPを
『  →
  AP 』と書くのは面倒でしょ。だから 『AB↓』 と書いた方がわかりやすいと思ったのよ。

でもよく考えたら 『↓AP』 と書いた方がいいわね。
『↓』を『ベクトル』って読むと決めておいたら、『↓AB』は『ベクトルAB』と読めるでしょ。

『↓』は「した」を変換したらでるわ。
>>898-899
まだまちがってた。n=6の場合が一番むづい。(123)と(123)(456)が中心化群、
正規化群等で区別しにくい。
n=6のときA(6)のクライン4群に同型な部分群はH={e,(12)(34),(13)(24),(14)(23)}
と共役なものとH={e,(12)(34),(12)(56),(34)(56)}に共役なもののいづれかで
それぞれN_G(H)/Hが3次巡回群であるか3次対称群であるかで区別できる。
そして長さ3の循環置換は前者のタイプの正規化群にしかはいらないことで
(123)型と(123)(456)型を区別できる。もっとあざやかなんないかな?
918902:01/11/14 00:55
>916

ベクトルについては、まだ厨房なのでよくわからんのですが、

> AP↓=s・(3/5)・AB↓  +  (1−s)・AC↓

は成り立つものなんですか?
あぁ、sのとり方次第で成り立つのか・・・。

こんなアホに丁寧に教えてくださってありがとうございますです・・・。
919おかma:01/11/14 01:13
>918
↓AP=s・↓AE+(1−s)・↓AC    (←これが成り立つ事は教科書でも読めば分かるわ。ここでは説明しづらいわね。)
上の式に
↓AE=(3/5)・↓AB   (←これは分かるでしょうね)
を当てはめれば下の式が成り立つ事が分かるはずね。

> AP↓=s・(3/5)・AB↓  +  (1−s)・AC↓
この式自体の意味は考えない方がいいわよ。「sが〜になったら・・・」とか考えると混乱しちゃうわ。
920おかma:01/11/14 01:22
とにかく解答の続きを書くと
ーーーーーーーーーーーーーーーーーーーーーーーーーーーーーーーーーーーーーーーーー
↓AP=s・(3/5)・↓AB  +  (1−s)・↓AC ・・・@
↓AP=     t・↓AB+(1−t)・(3/7)・↓AC ・・・A
 ̄ ̄ ̄ ̄ ̄ ̄ ̄ ̄ ̄ ̄ ̄ ̄ ̄ ̄ ̄ ̄ ̄ ̄ ̄ ̄
この次に何をするかと言うと、上の@とAの式の ○・↓AB と □・↓AC の○と□が等しくなるのよ。

言い換えると
┌ (3/5)s=t
└ (1−s)=(3/7)(1−t)
の2式が成り立つと言うことね。

この次は上の方の式のtを下の式に当てはめたら
s=10/13、t=6/13 となるのよ。
ーーーーーーーーーーーーーーーーーーーーーーーーーーーーーーーーーーーーーーーーー

この問題はここまで理解できるようになればOKよ。
まぁあせること無いからゆっくり考えなさいな。
分からないようなら学校の先生に図を描きながら説明してもらったほうがいいわね。
921132人目の素数さん:01/11/14 01:24
問題:nを3以上の自然数とするとき、
2^n > 2n
を証明せよ。
-----------------------------------------------------------
この問題の回答を見ると、

n=3のとき、左辺>右辺で成立。
n=k(k:3以上の自然数)のとき
2^k > 2が成り立つと仮定すると
2^k × 2 > 4k
ここで
4k−2(k+1)= 2k−2 > 0 (k≧3より)
∴2^(k+1) > 4k >2(k+1)
∴2^(k+1) > 2(k+1)
(以下略)
---------------------------------------------------
となっているんですが、
2^k × 2 > 4k
↑の式から↓の式への手順がよくわかりません。
4k−2(k+1)= 2k−2 > 0 (k≧3より)

よろしくお願いします。
922902:01/11/14 01:30
>920

どうもありがとうございました!
なんとか理解できましたです!
923おかma:01/11/14 01:39
>>921
これは、結局は
 2^(k+1) > 2(k+1) (・・・@式とするわ)
を示したいのよね。

そうしたら、解答の上のほうで
2^k × 2 > 4k とあるわね。 これは 2^(k+1)>4k と同じことでしょ。(この式は絶対に成り立つと仮定しているの。)

だったら
4k は 2(k+1) より大きい事を示せば上の@式が成り立つ事を示せるのよ。

4k − 2(k+1) = 2k−2 > 2・3−2 = 2 >0
となるから@式が成り立つの。

そして n=k+1のときも絶対に成り立つから
2^n>2n (n:3以上のすべての自然数) が成り立つ事を示せるのよ。
924132人目の素数さん:01/11/14 01:40
>>921
2^k × 2 > 4k
から4k−2(k+1)= 2k−2 > 0
を導出してはいない。

k≧3の時に4k−2(k+1)= 2k−2 > 0が
成り立つといっているだけだ。
925132人目の素数さん:01/11/14 01:49
>>923
>>924
なるほど、よくわかりました!!
涙出るほどありがたいです。
厨房にレスして頂き感謝です。
926132人目の素数さん:01/11/14 02:03
ちょっと気になりましたので・・・ひとつご意見願います。
代数体KのQ上の整基底β_1,…,β_nをとると、Kの任意の整数αは
α=a_1β_1+…+a_nβ_n
という有理整係数線形結合で表せる。(但し、[K:Q]=n)
これって証明とかが必要なものでしょうか?
自明と流しても問題ないのでしょうか?
>>926
言葉が分からないんだけれどK=Qで
1=a×2となる整数aがないから
成り立たないんじゃないですか。
928132人目の素数さん:01/11/14 02:44
この問題が解けません。
http://cgi.mytoday.de/%7Eimg12/img/97.jpg
929132人目の素数さん:01/11/14 03:32
>926
一般にO(Kの整数環)はZ^nに同型で、その主張は正しいが
自明ではないと思う.
証明は「整数論」東大出版とかにのってたと思う.
930132人目の素数さん:01/11/14 04:49
>>928
この寒空の下を集団でフルチンで歩ける為には
写真のメンバーが帝京大学ラグビー部であることが必要条件である。
同時に帝京大学ラグビー部であれば、この寒空の下を集団でフルチンであるける。
よって十分性も示された。
931ななし:01/11/14 05:50
質問すまそ。適当なスレが見つからなかったので・・・
ベクトルバンドルって、多様体で出てくる、法線ベクトルや
ベクトル場を、位相空間に拡張したような物と思って良いの?
932 ◆pvySbQO2 :01/11/14 08:00
>>892
曲線上の点Aで曲線に垂直に交わる直線と点Bで曲線に垂直に交わる直線の交点を
Oとすると曲線はほぼOを中心とし半径をOAとする円で近似できます。
この円の半径が大きいと曲がり具合が小さく
この円の半径が小さいと曲がり具合が大きいので
半径の逆数で曲がり具合をあらわそうと曲率ができました。
だから点Aでの曲率は
 lim_{B−>A}(1/OA)
=lim_{B−>A}(AOB/AB)
で曲線がtによって(x(t),y(t))と表されているときは
a=dx/dt
b=dy/dt
c=d^2x/dt^2
d=d^2y/dt^2
とすると曲率は
(ad−bc)/(a^2+b^2)^(3/2)
となります。

(x/a)^2+(y/b)^2=1の場合は
x=a・cos(t),y=b・sin(t)として計算すれば
 ab/(a^2・sin^2(t)+b^2・cos^2(t))^(3/2)
=a^4・b^4/(b^4・x^2+a^4・y^2)^(3/2)
となります。
933132人目の素数さん:01/11/14 08:28
はぁ。困っています。
S^2={(x,y,z)|x^2+y^2+z^2=1}
(1)f(x,y,z)=zはS^2上のC^∞関数であることを示せ。
(2)f(x,y,z)=x^2+y^2はS^2上のC^∞関数であることを示せ。
どうか何卒・・・。
934厨房な高校留学生 ◆BIvPkp0k :01/11/14 09:29
三角形を作図していて思ったのですが、
どうして、一番大きな内角と、最も長い一辺は、
絶対に隣り合わないんですか?
935132人目の素数さん:01/11/14 10:03
>>933
定義に従ってやればいい。
>>917
よかった。このスレまだいきのこってた。>>917はうそ。>>898-899のままであってる。
A(6)の4グループは{e,(12)(34),(34)(56),(12)(56)}型しかないや。ゴメソ。
>>934
隣り合わない辺を延長して最も長い一辺と同じ長さになるように
二等辺三角形を作ると、その三角形の内角の和が180度より大きくなる。
>>926
ごめん。君前に“整基底”ってなに?って聞いてた子だね。そのときいいかげんな
レスつけた。R代数Sの整基底をもつとはSがR加群として自由R加群で
そのR加群としての基底が整基底。
((EX))K=Q,L=Q(√3)のときKの整数環R=Z,Lの整数環はS=Z[(1+√3)/2]。
このとき整基底としては{1,(1+√3)/2}などをとることができる。
一般に代数体とその整数環、およびその代数拡大とその整数環をとれば
つねにこのような整基底をとれる。
>>936
マタマタトリケシマス。>>917ガセイカイデス。スマソ。
>>934
正弦定理 a:b:c=sinA:sinB:sinC を使えば楽に示せる。
何にせよ、>>934のようなことを自力で発見するのは大変よいことだと思いまふ。
941928:01/11/14 13:07
>938
あ、整基底の定義がそれなんや、じゃ、自明やな、すまん>926
942132人目の素数さん:01/11/14 15:08
すいません、突然。

仕事でどうしても今日中に知りたいことがありまして。

「ハッシュ関数」について文系出身のぼくでも理解できるように、
超超判りやすく教えて。お願い。

厳密じゃなくてもよいです。同じく文系の上司がなんとなくごまか
せる程度の解説でよいです。
お願いします。
943にゃ=ん?:01/11/14 15:45
>>905
これが分からないのなら
これはこういう風にやるものとして暗記した方がよいと思います。

しかし、一応説明
>>890の一番上の式は
「公式:(x+y)^2=x^2+y^2+2xy」の「2xy」を移項するとできます。
「x+y」と「x^2+y^2」を利用して「xy」を求めようとして作った式です。
>>937
>三角形の内角の和が180度より大きくなる
すみませんが、よく分かりませんでした。

>>940
>正弦定理 a:b:c=sinA:sinB:sinC
調べてみます。
945132人目の素数さん:01/11/14 15:49
Nという形をした部品があって、ある法則にしたがって結合していくと
Nの集合が、またNと同じような形になって…

というフラクタルはありませんか。いちばん簡単なのは単なる
三角形や四角形だと思うけど…
群Gで、x^2=xのときGは可換群になるよね。
じゃあ、x^3=xのときは?

x^n=xのときGが可換になるnって求められる?
n≧3のときは非可換になるのかな?(予想)
947132人目の素数さん:01/11/14 16:46
群って、有限群で考えていると思うが
>>946
生成元が1個だけなら、任意のnで巡回群になるが。
948132人目の素数さん:01/11/14 17:06
>>942
具体的な例で示そう。

例えばとあるデータベース(例:英単語の辞書)を作るとする。
全ての単語をまとめて1つのデータベースにすると検索が大変。
そこでいくつかに分類して登録すれば検索速度は速くなる。
その分類手法として、単語を何らかの方法で限定された範囲の数値に変換する。

例えばa=1,b=2,...,z=26として単語中の全ての文字を足し、
合計を50で割った余り、とすれば、全ての単語は0〜49に変換される。
ball=2+1+12+12=27 → 27
play=16+12+1+25=54 → 4
こうすればデータベースを50グループで管理することができ、
検索効率は大きく向上する。

このように変換する関数をハッシュ関数という。

こんな感じで。
949946:01/11/14 17:31
>>947
いや、可換になるか非可換になるかを知りたいのね。
950数学ひで:01/11/14 17:32
中学生の僕が知っていて自慢できる公式や定理などを教えてください
951数学ひで:01/11/14 17:35
パップスギュルダンの定理の定理って何ですか?
使い方などを教えてください。
>>950
Atiyah-Singerの指数定理
953質問です:01/11/14 17:45
次のような問題を解くプログラムを作りたいのですが、
どのようにしたら良いのかわかりません。
方針だけでもよいので、教えてください。

問題:
箱がN個あり、各箱には必ずMin個以上、Max個以下の玉を入れる。
玉は、白玉P個、赤玉Q個、青玉R個がある。
ただし、赤玉と青玉は一緒に入れることはできない。
このとき、可能な入れ方を少なくとも1つ求めたい。
(できれば、各箱にはMinに近い個数を入れたい)

式は、
箱iに入れる玉の総数ni,白玉数pi,赤玉数qi,青玉数ri,とすると
1≦i≦N
Σpi=P
Σqi=Q
Σri=R
Σni=N
pi+qi+ri=ni
Min≦ni≦Max
qi*ri=0
このとき、pi,qi,riを求めよ、
だと思うのですが...
954132人目の素数さん:01/11/14 17:55
>>953
ではモンテカルロでどうぞ。
955私も質問です:01/11/14 18:01
次のような問題を効率的に解くアルゴリズムを知りたいのですが。

node node 得点
--------------------
a → b 100
a → c 50
b → a 98
b → c 49
c → a 70
c → b 99

表から、得点が最大となるような多分木を作成せよ。
ただし、nodeをそれぞれ一回ずつ使用し、
矢先は子の方に向いているものとする。
956りゅーりゅー:01/11/14 18:50
変化の割合を求める式

Y=aX2乗の式で、
Xはc〜d

a(c+d)を証明方法を教えて下さい。
957132人目の素数さん:01/11/14 19:13
だれか>>956を翻訳してくれ
958132人目の素数さん:01/11/14 19:16
>>956
(変化の割合)=(yの増加量)/(xの増加量)
=(ad^2-ac^2)/(d-c)
=a(d^2-c^2)/(d-c)
=a(d+c)(d-c)/(d-c)
=a(d+c)
=a(c+d)
以上。
959パァ:01/11/14 19:44
>>943

ナルホド!ありがとうございました。
そういう公式にすれば…
数学って難しいですねぇ…
━━━━━━━━━━━━━━━━━━━━━━━━━━━━━━

             新しいスレッドが出来ましたので
     新たに質問をする方はこちらでして頂けると嬉しいですわ
      (スレッド立てるのが遅れてしまいまして申し訳ありません)

         ◆ わからない問題はここに書いてね 16 ◆
    http://cheese.2ch.net/test/read.cgi/math/1005735838/l50/


━━━━━━━━━━━━━━━━━━━━━━━━━━━━━━
961KARL ◆gjHKPQSQ :01/11/14 23:26
>>934
中学生時代を思い出しました。「チャート式幾何」なつかしいなあ(涙)
「より長い辺に相対する角はより短い辺に相対する角より大きい」とその逆
「より大きい角に相対する辺はより小さい角に相対する辺より大きい」
とりあえずこの2つの定理だけ書いておきます。モノグラフシリーズ「幾何学」
(清宮俊雄)など初等幾何の参考書(教科書)を見るとちゃんと証明が書いてある
と思います。
962132人目の素数さん:01/11/14 23:53
>946
1の根をつけたしてるだけなので、可換
963ポケット:01/11/15 17:12
>>932
有難う御座います。
964高校生の数学:01/11/15 19:37
(1)4点(0,0,0)、(0,1,−1)、(−1,2,0)、(1,2,5)
   を通る球の方程式を求めよ。   <3次元図形の方程式>

(2)AB=2、BC=√3+1、CD=√2、∠ABC=60°、∠BCD=75°である四角形
   ABCDの面積を求めよ。   <三角比>

(3)初項から第m項までの和が初項から第n項までの和に等しい数列がある。この数列の
   初項からm+n項までの和を求めよ。ただし、m≠nとする。  <等差数列>

(4)初項が1、第2項が(1+2)、第3項が(1+2+3)、・・・・、第r項が(1+2+3+・・・・+r)
   である数列の初項から第n項までの総和を求めよ。   <等比数列>


   みなさんなら一瞬で説いてしまう問題ですが、解法、回答のほう、どうかお願いします。
966フエルメール:01/11/15 23:36
質問です!!もともと英語で書かれていたので英語の文章も・・・
英語版
Recall that every rational number q∈Q can be written
uniquely in the form q=a/b where a∈Z, b∈N, and
a and b are relatively prime. Consider the function
f:R→R defined by
f(x)= 0 if x is irrational,
f(x)= 1/b if x is rational and x=a/b as above
Prove that f is continuous at every irrational point,
but f is discountinuous at every rational point.
日本語訳(多分ですけど・・・)
q=a/bと言う形で書かれる有理数q∈Qを思い返してみよう。た
だしa∈Z,b∈Nで、aとbは互いに素である。ここで下のように
定義される関数f:R→Rを考えてみよう。
f(x)= 0 (xが無理数のとき)
f(x)= 1/b (xが上のx=a/bような有理数のとき)
fが全ての無理数の点で連続であり、すべての有理数の点で連
続でないことを証明せよ。
・・・と言う問題です。どうしても解けませんでした。
どうかよろしくお願いします!
967132人目の素数さん:01/11/15 23:43
>>966
日本語約
「形式q=の中ですべての有理数q∈Qをユニークに
書くことができることを思い出す、1つの/b、どこで、
a∈Z、b∈N、そして1つの、そしてb、比較的第1です。
関数fを考慮する:xが不合理な場合にf(x)=0によって
定義されたR→R、xが合理的なx=である場合、f(x)=1/b、
1つの/b、として、の上に、fがすべての不合理なポイント
で連続的であることを証明する、しかし、fはすべての
合理的なポイントでdiscountinuousです。」

http://www.excite.co.jp/world/text/
の翻訳サイト参照。
>>966
無理数 x で連続であること:
任意のε>0 に対し,a/b s.t 1/b>ε のうちで
x にもっとも近いものと x との距離δとすると
∀y s.t. |x-y|<δ f(y)<ε がなりたつ。

有理数 x で不連続であること:
x に収束する無理数列 {xn} をとるとき
lim f(xn)=0≠f(x)
969フエルメール:01/11/16 01:00
う・・・む、むずかしいですね。なんとか理解できるように
がんばってみます!ありがとうございました!
━━━━━━━━━━━━━━━━━━━━━━━━━━━━━━

             新しいスレッドが出来ましたので
     新たに質問をする方はこちらでして頂けると嬉しいですわ

         ◆ わからない問題はここに書いてね 16 ◆
    http://cheese.2ch.net/test/read.cgi/math/1005735838/l50


━━━━━━━━━━━━━━━━━━━━━━━━━━━━━━
971?@?@:01/11/16 14:42
1/(X*X*X+1)


この積分の仕方を教えてください
972ちむ教の信者:01/11/19 22:03
f(x)=x^2-2ax+a+1

f(n)<0を満たすnがちょうど1つ存在するように
aの範囲を求めよ。

これは、範囲の限定で、超難問だそうなんですけど、
問題をよんでも<f(n)<0を満たすnがちょうど1つ存在するように
aの範囲を求めよ。
の意味及び解法がわかません。
解が1つ存在するって、x軸にってことですか?
973132人目の素数さん:01/11/19 22:09
>>972
nは整数とかの条件ないの?
974ちむ教の信者の母:01/11/19 23:09
>>973
わかりきったことをいちいち聞かずに
簡潔に教えてやってください。お願いします。
975975:01/11/20 12:12
sin(1/x)がx→0で極限を持たないことをうまく説明できません。
お願いします。
976ちむ教の信者:01/11/20 19:31
数列1,11,111・・・の一般項はどう求めるの?
また、和はどうするの?
━━━━━━━━━━━━━━━━━━━━━━━━━━━━━━

             新しいスレッドが出来ましたので
     新たに質問をする方はこちらでして頂けると嬉しいですわ

         ◆ わからない問題はここに書いてね 16 ◆
    http://cheese.2ch.net/test/read.cgi/math/1005735838/l50


━━━━━━━━━━━━━━━━━━━━━━━━━━━━━━
>>976
a_[n+1]=a_[n]+10^ n 、  a_1=1
(a_[n+1])/10^(n+1)=(1/10)(a_[n])/10^n+1/10
b_[n+1]=1/10b_n+1/10
b_[n+1]-1/9=1/10(b_[n]-1/9)
979質問です:01/12/02 01:14
四元数群H(Z)={1,-1,i,-i,j,-j,k,-k}に対して
(1) H(Z)の共役類への分解を与えよ
(2) 中心Z(H(Z))を求めよ

っていう問題なんですけどよくわからないんですぅ。
分かる方どうか詳しく教えてください。よろしくお願いしますm(_ _)m
以下の最新のスレッドで質問してくださいね。

◆ わからない問題はここに書いてね 17 ◆
http://cheese.2ch.net/test/read.cgi?bbs=math&key=1006859798
981質問です:01/12/03 05:46
不定積分

∫(4x^5−1)(x^5+x+1)^(−2) dx

の解き方教えてください。お願いします。
982ken_m:01/12/05 03:47
えーと
先輩から出された質問!(解けなかった・・)
9*1=1
9*2=18
9*3=27
・・・
1の位と十の位を足すと9になるのはなぜ?
983132人目の素数さん:01/12/05 04:38
>>982
すげえ。ほんとになるじゃん。
>>982
9*k=10*(k-1)+(10-k)
(k-1)+(10-k)=9
985132人目の素数さん
来年の予想問題/
2002!は0がいくつ続くか?